[obm-l] Re: [obm-l] Sequência

2014-11-19 Por tôpico Ralph Teixeira
Vou supor que suas sequencias comecam do indice 1, e nao do indice 0.

1) Dado k fixo, tome Y_n=X_(n+k-1) (n=1,2,3,...)

2) Esse negocio de "formula explicita" eh mais vago do que parece.

X_n = { 1, se n=3k+1,
  { 0, se n=3k+2 ou n=3k+3
(onde k=0,1,2,3,...)

eh uma formula explicita e facil de ler, apesar de ser definida por partes.

Mas tem gente que quer uma formula em uma linha soh, sem "partes"... Bom,
ha opcoes, como por exemplo

X_n = |sin((n-1).pi/3).2raiz(3)/3| (n=1,2,3,...)

ou, se eu puder usar a funcao "mod" (resto na divisao inteira):

X_n = 1- [(n-1)^2] mod 3

Mas serah que estas formulas sao realmente melhores que a primeira? Eu,
pessoalmente, acho a primeira bem mais facil de entender. Alias, mais facil
ainda de entender eh dizer que X_n={1,0,0,1,0,0,1,0,0,...}, com periodo de
repeticao 3. :) :) :)

Abraco, Ralph.

2014-11-19 20:19 GMT-02:00 Eduardo Henrique :

> Pessoas, estou com as seguintes dúvidas:
> 1) Dada uma sequencia {Xn} de números reais, como eu posso criar uma nova
> sequencia partindo do 'k-ésimo' termo da sequencia anterior? (No caso, eu
> quero que a nova sequência tenha o primeiro termo igual ao k-ésimo termo
> duma sequencia anterior dada, e que o segundo termo da nova sequência seja
> igual ao (k+1)-ésimo termo e assim por diante).
>
> 2) Ache uma fórmula explicita para a sequência {1,0,0,1,0,0,1,0,0,1,0...}.
>
> Att.
> Eduardo
>
> --
> Esta mensagem foi verificada pelo sistema de antivírus e
> acredita-se estar livre de perigo.
>

-- 
Esta mensagem foi verificada pelo sistema de antiv�rus e
 acredita-se estar livre de perigo.



[obm-l] Re: [obm-l] Extração de Determinantes

2014-11-17 Por tôpico Ralph Teixeira
Bom, para extrair um determinante mxm você tem que escolher m colunas e m
linhas. Tem C(n,m) maneiras de fazer a primeira coisa, e C(n,m) de fazer a
segunda. Então, no total, são C(n,m)^2 menores que podem ser extraídos da
sua matriz.

Entendi corretamente o que "extrair" significava?

Abraço,
Ralph

2014-11-14 15:40 GMT-02:00 Walter Tadeu Nogueira da Silveira <
wtade...@gmail.com>:

> Amigos,
>
> Há algum resultado que permita saber quantos determinantes de ordem 2, 3,
> etc podem ser extraídos de uma matriz quadrada de ordem n?
> Creio que o desenvolvimento por Laplace deve indicar, mas não consegui uma
> generalização. Agradeço uma sugestão.
>
> Abs
>
>
>
> --
> Esta mensagem foi verificada pelo sistema de antivírus e
> acredita-se estar livre de perigo.

-- 
Esta mensagem foi verificada pelo sistema de antiv�rus e
 acredita-se estar livre de perigo.



[obm-l] Re: [obm-l] Indução

2014-11-17 Por tôpico Ralph Teixeira
Seja P(n): o banco pode pagar a quantia de n reais.

Então:
P(8) é verdadeira: 8=3+5
P(9) é verdadeira: 9=3+3+3
P(10) é verdadeira: 10=5+5

Agora, se P(k) é verdadeira, então P(k+3) também é.
De fato, basta pagar k reais da maneira que é possível, e adicionar uma
nota de $3.

Por indução, P(n) vale para todo n>=8.

---///---

Essa foi uma indução de "passo 3". Se você quiser converter isso numa
indução de "passo 1", use:
Q(n): o banco pode pagar n, n+1 e n+2 reais.

Então:
i) Q(8) é verdadeira (vide P(8), P(9) e P(10) acima).
ii) Se Q(k) é verdadeira, Q(k+1) também é.
(Pois se pode pagar k, k+1 e k+2, então obviamente pode pagar k+1 e k+2.
Para pagar k+3, pague k e ponha uma nota de 3.)

Por indução, Q(n) é verdadeira para todo n>=8.

Abraço,
Ralph

2014-11-15 9:19 GMT-02:00 marcone augusto araújo borges <
marconeborge...@hotmail.com>:

> Em um país longinquo, a moeda local é o cruzeiro.Neste país um banco tem
> uma quantidade ilimitada
> de cédulas de 3 e 5 crzeiros.Prove, por indução, que o banco pode pagar
> uma quantidade qualquer(inteira)
> de cruzeiros, maior que 7
>
>
> --
> Esta mensagem foi verificada pelo sistema de antivírus e
> acredita-se estar livre de perigo.
>

-- 
Esta mensagem foi verificada pelo sistema de antiv�rus e
 acredita-se estar livre de perigo.



[obm-l] Re: [obm-l] Problema de análise

2014-11-12 Por tôpico Ralph Teixeira
Hmmm... Deu vontade de olhar para g(x)=n.ln[f(x)] + m ln[f(-x)], cuja
derivada é g'(x)=n.f'(x)/f(x) - m. f´(-x)/f(-x). Ou seja, a condição pedida
passaria a ser g´(c)=0.

Como g(0)=0 independentemente de m e n, basta achar um outro ponto d onde
g(d)=0 para usar um Rolle. Ou seja, você quer mostrar que
h(x)=ln[f(-x)]/ln[f(x)] =-m/n para algum x=d... Isto é, você quer achar um
racional negativo na imagem de h.

Parece que lim(x->0) h(x) = -1? Então deve ser possível arrumar um
intervalo qualquer em volta de x=0 onde h(x) é contínua, e portanto ela
deve assumir outros valores racionais (se ela fosse constante, seria -1,
também serve; o problema mesmo é se f(x)=1 em uma montanha de pontos, o que
faz h nem existir, tem que analisar isto à parte).

Tem um monte de furinhos nessas ideias, mas acho que dá para fechar o
problema por esse caminho?

Abraço,
Ralph

2014-11-12 0:07 GMT-02:00 Artur Costa Steiner :

> Oi amigos,
>
> Ainda não consegui resolver este não. Alguém pode colaborar?
>
> Suponhamos que a função real f seja contínua e positiva em  em [-1,
> 1], diferenciável em (0, 1) e que f(0) = 1. Mostre que existem c em (-1, 1)
> e inteiros positivos m e n tais que
>
> m f(c) f'(-c) = n f(-c) f'(c)
>
> Obrigado.
>
> Artur
>
>
>
> --
> Esta mensagem foi verificada pelo sistema de antivírus e
> acredita-se estar livre de perigo.

-- 
Esta mensagem foi verificada pelo sistema de antiv�rus e
 acredita-se estar livre de perigo.



Re: [obm-l] problema real - off topic

2014-11-08 Por tôpico Ralph Teixeira
Ah, olha soh: as combinacoes lineares de 38 e 56 (com coeficientes
inteiros) abaixo de 300 sao:
Sem usar 56: 38, 76, 114,152,190, 228, 266;
Com um 56: 56, 94, 132, 170, 208, 246, 284;
Com dois 56: 112,150,188, 226, 264;
Com tres 56: 168, 206, 244, 282;
Com quatro 56: 224, 262, 300;
Com cinco 56: 280.

Em particular, de 100 voce soh consegue "usar" no maximo 94; de 120,
tira-se no maximo 114; de 200 soh sai 190; e de 300 talvez de para tirar
300.

Entao:

i) De 200x100 tira-se no maximo 190x94, se tanto. Mas 190x94/(56x38)=8.4,
entao de 200x100 soh se tiram no maximo 8 chapas. Como 200x100/(56x38) =
9.4, o desperdicio serah pelo menos 1.4/8 = 17.5%.
ii) De 200x120 tira-se no maximo 190x114. Mas 190x114/(56x38)=10.18, entao
no maximo 10 chapas. Mas 200x120/(56x38) = 11.3. O desperdicio seria pelo
menos 1.3/10 = 13%.
iii) De 300x100 tira-se no maximo 300x94. Mas 300x94/(56x38)=13.25, maximo
de 13 chapas. Como 300x100/(56x38) = 14.1, desperdicio de 1.1/14 = 7.8% no
minimo.
iv) De 300x120 tira-se no maximo 300x114/(56x38)=16.07 chapas. Note que
300x120/(56x38)=16.92, entao o desperdicio seria aquele que eu tinha citado
de 0.92/16 = 5.7%.

Este ultimo eh menor que todos os outros, e a gente CONSEGUIU uma maneira
de realiza-lo, entao ele eh o melhor possivel (se eu nao errei conta)!

Abraco, Ralph.




2014-11-08 23:38 GMT-02:00 Ralph Teixeira :

> Hmmm... Acho que eh um tiquinho mais complicado, se a gente levar em conta
> misturas de orientacoes.
>
> Por exemplo: voce poderia pegar a chapa 300x120 e dividi-la em 224x120 +
> 76x120. A primeira vira (4x56)x(3x38) = 12 chapas, e a segunda vira
> (2x38)x(2x56)=4 chapas. Entao em teoria eh possivel fazer **16** chapas a
> partir de uma 300x120. Digo "em teoria" porque em uma direcao os cortes
> ficaram JUSTISSIMOS, sem um milimetro de sobra -- nao sei o quanto esta
> precisao seria problematica no seu caso.
>
> Neste caso, o desperdicio seria de 1952cm^2 por 16 chapas -- desperdicio
> medio de 122cm^2 por chapa, ou seja, apenas uns 5.7%. Ficou bem razoavel.
>
> Serah que tem algum jeito ainda melhor?
>
> Abraco, Ralph.
>
> P.S.: Achei que a medida certa era "desperdicio por chapa" pois imagino
> que voce vai fazer INUMERAS chapas, certo?
>
>
>
> 2014-11-08 22:09 GMT-02:00 saulo nilson :
>
>
>> Preciso cortar chapas de
>> 38cm x 56cm
>>
>> e gostaria de saber qual dos tamanhos de chapa abaixo seria o melhor (ou
>> seja, menor perda)
>> 200cm x 100cm
>> 200cm x 120cm
>> 300cm x 100cm
>> 300cm x 120cm
>>
>> 168*76 sobra 24*168+100*32=7232cm^2
>> 168 *114 sobra 6*168+120*32=4848
>>  280*76 sobra 20*100+24*280=8720
>> 280*114 sobra 120*20+280*6=4080
>> 300 *120 e melhor pois tem menos perda
>>
>> 2014-11-08 12:06 GMT-02:00 Rogerio Ponce :
>>
>>> Ola' Hermann,
>>> escolha uma das chapas de 120cm de largura.
>>>
>>> Se for a de 200cm de comprimento, a divisao do comprimento por 5 (e da
>>> largura por 2) gera retangulos de 40cmx60cm.
>>> Portanto voce obtera' 10 pedacos do tamanho desejado.
>>>
>>> Se for a de 300cm, a divisao do comprimento por 5 (e da largura por 3)
>>> tambem gera retangulos de 60cmx40cm.
>>> Neste caso, voce obtera' 15 pedacos do tamanho desejado.
>>>
>>> O rendimento e' o mesmo, e a decisao devera' ser funcao do total de
>>> pedacos de que voce precisa.
>>>
>>> Repare que, ao considerar o total, talvez mesmo uma chapa com menor
>>> rendimento seja mais apropriada.
>>> Exemplo: se voce precisa de apenas dois pedacos de 38x56, use a chapa de
>>> 100x200.
>>>
>>> []'s
>>> Rogerio Ponce
>>>
>>> 2014-11-07 18:54 GMT-02:00 Hermann :
>>>
>>>  Meus amigos estou com um problema real de "otimização"
>>>>
>>>> Preciso cortar chapas de
>>>>  38cm x 56cm
>>>>
>>>> e gostaria de saber qual dos tamanhos de chapa abaixo seria o melhor
>>>> (ou seja, menor perda)
>>>> 200cm x 100cm
>>>> 200cm x 120cm
>>>> 300cm x 100cm
>>>> 300cm x 120cm
>>>>
>>>> Se alguém puder me ajudar agradeço muito
>>>> Abraços
>>>> Hermann
>>>>
>>>> --
>>>> Esta mensagem foi verificada pelo sistema de antivírus e
>>>> acredita-se estar livre de perigo.
>>>>
>>>
>>>
>>> --
>>> Esta mensagem foi verificada pelo sistema de antivírus e
>>> acredita-se estar livre de perigo.
>>>
>>
>>
>> --
>> Esta mensagem foi verificada pelo sistema de antivírus e
>> acredita-se estar livre de perigo.
>>
>
>

-- 
Esta mensagem foi verificada pelo sistema de antiv�rus e
 acredita-se estar livre de perigo.



Re: [obm-l] problema real - off topic

2014-11-08 Por tôpico Ralph Teixeira
Hmmm... Acho que eh um tiquinho mais complicado, se a gente levar em conta
misturas de orientacoes.

Por exemplo: voce poderia pegar a chapa 300x120 e dividi-la em 224x120 +
76x120. A primeira vira (4x56)x(3x38) = 12 chapas, e a segunda vira
(2x38)x(2x56)=4 chapas. Entao em teoria eh possivel fazer **16** chapas a
partir de uma 300x120. Digo "em teoria" porque em uma direcao os cortes
ficaram JUSTISSIMOS, sem um milimetro de sobra -- nao sei o quanto esta
precisao seria problematica no seu caso.

Neste caso, o desperdicio seria de 1952cm^2 por 16 chapas -- desperdicio
medio de 122cm^2 por chapa, ou seja, apenas uns 5.7%. Ficou bem razoavel.

Serah que tem algum jeito ainda melhor?

Abraco, Ralph.

P.S.: Achei que a medida certa era "desperdicio por chapa" pois imagino que
voce vai fazer INUMERAS chapas, certo?



2014-11-08 22:09 GMT-02:00 saulo nilson :

>
> Preciso cortar chapas de
> 38cm x 56cm
>
> e gostaria de saber qual dos tamanhos de chapa abaixo seria o melhor (ou
> seja, menor perda)
> 200cm x 100cm
> 200cm x 120cm
> 300cm x 100cm
> 300cm x 120cm
>
> 168*76 sobra 24*168+100*32=7232cm^2
> 168 *114 sobra 6*168+120*32=4848
>  280*76 sobra 20*100+24*280=8720
> 280*114 sobra 120*20+280*6=4080
> 300 *120 e melhor pois tem menos perda
>
> 2014-11-08 12:06 GMT-02:00 Rogerio Ponce :
>
>> Ola' Hermann,
>> escolha uma das chapas de 120cm de largura.
>>
>> Se for a de 200cm de comprimento, a divisao do comprimento por 5 (e da
>> largura por 2) gera retangulos de 40cmx60cm.
>> Portanto voce obtera' 10 pedacos do tamanho desejado.
>>
>> Se for a de 300cm, a divisao do comprimento por 5 (e da largura por 3)
>> tambem gera retangulos de 60cmx40cm.
>> Neste caso, voce obtera' 15 pedacos do tamanho desejado.
>>
>> O rendimento e' o mesmo, e a decisao devera' ser funcao do total de
>> pedacos de que voce precisa.
>>
>> Repare que, ao considerar o total, talvez mesmo uma chapa com menor
>> rendimento seja mais apropriada.
>> Exemplo: se voce precisa de apenas dois pedacos de 38x56, use a chapa de
>> 100x200.
>>
>> []'s
>> Rogerio Ponce
>>
>> 2014-11-07 18:54 GMT-02:00 Hermann :
>>
>>  Meus amigos estou com um problema real de "otimização"
>>>
>>> Preciso cortar chapas de
>>>  38cm x 56cm
>>>
>>> e gostaria de saber qual dos tamanhos de chapa abaixo seria o melhor (ou
>>> seja, menor perda)
>>> 200cm x 100cm
>>> 200cm x 120cm
>>> 300cm x 100cm
>>> 300cm x 120cm
>>>
>>> Se alguém puder me ajudar agradeço muito
>>> Abraços
>>> Hermann
>>>
>>> --
>>> Esta mensagem foi verificada pelo sistema de antivírus e
>>> acredita-se estar livre de perigo.
>>>
>>
>>
>> --
>> Esta mensagem foi verificada pelo sistema de antivírus e
>> acredita-se estar livre de perigo.
>>
>
>
> --
> Esta mensagem foi verificada pelo sistema de antivírus e
> acredita-se estar livre de perigo.
>

-- 
Esta mensagem foi verificada pelo sistema de antiv�rus e
 acredita-se estar livre de perigo.



Re: [obm-l] Problema de pilhas

2014-11-07 Por tôpico Ralph Teixeira
Uma maneira que satisfaz as condições do enunciado é com 30 pilhas:

1,3,5,7,9,...,59

Ao dividir qualquer pilha em duas, tem que aparecer um ímpar menor, então
haverá repetição.

Agora temos que mostrar que não há maneira mais eficiente que esta...
Suponha, por contradição, que você conseguiu uma partição com menos pilhas,
digamos x1(2m+1)

Mas existem pelo menos m maneiras de quebrar a pilha xm:
1+(xm-1),2+(xm-2),...,m+(xm-m). Note que xm-m>m+1, então realmente todas
essas maneiras são válidas. Portanto, tem que haver uma pilha de tamanho em
{1,xm-1}, outra em {2,xm-2}, etc. Mas você só tinha m-1 pilhas menores que
xm! Então haverá uma maneira de quebrar a pilha xm em dois pedaços menores
distintos que não estão presentes!

Assim, mostramos que não há maneira de distribuir as pedras em 29 pilhas
(ou menos), e portanto a maneira mais eficiente possível tem 30 pilhas.

Abraço,
  Ralph

P.S.: Note-se que até onde sabemos poderia haver OUTRAS maneiras com 30
pilhas, não provamos que aquela maneira é única.


2014-11-02 14:08 GMT-02:00 Mariana Groff :

> Boa Tarde,
> Alguém poderia, por favor, me auxiliar neste problema?
>
> Devemos distribuir 900 pedras em k pilhas, de modo que sejam satisfeitas
> as condições a seguir:
> (i) todas as pilhas têm quantidades distintas de pedras;
> (ii) se dividirmos uma das pilhas em duas pilhas não vazias, as k+1 pilhas
> resultantes não mais terão quantidades distintas de pedras.
> Ache o menor valor possível de k.
>
> Obrigada,
> Mariana
>
> --
> Esta mensagem foi verificada pelo sistema de antivírus e
> acredita-se estar livre de perigo.

-- 
Esta mensagem foi verificada pelo sistema de antiv�rus e
 acredita-se estar livre de perigo.



Re: [obm-l] Problema Legal

2014-10-20 Por tôpico Ralph Teixeira
E a ideia do Pedro também resolve o caso de 5 números distintos a:

> Boa tarde!
>
> Não havia me apercebido, mas por sorte não muda nada.
>
> Pois, como os números são distintos, se ordenarmo-los, a > b > c >d e as
> somas s1 > s2 > s3 >= s4 > s5 > s6.
>
> Como os números são distintos a + b = s1, a + c = s2, b+d = s5 e c + d =
> s6.
>
> logo poderemos formar um sistema: Ax = b onde A é a matriz abaixo.
> |  1  1  0  0  |
> |  1  0  1  0  |
> |  0  1  0  1  |
> |  0  0  1  1  |
>
> e bT = [ s1,s2,s5,s6]
>
> que pode facilmente ser transformada na matriz
>
> |  1  1  0  0  |
> |  0  1 -1  0  |
> |  0  0  1 -1  |
> |  0  0  0  2  |
>
> Como posto de A = 4 e dim (v) = 4 ==> din (N) = 0 ==> não existe solução
> ou a solução é única. Mas novamente como existe pelo menos uma solução a
> escolhida por A, a solução é única.
>
> R; Não é possível o jogador escolher quatro números que tornem impossível
> o jogador B ganhar.
>
>
> Saudações,
> PJMS.
>
>
>
> Em 20 de outubro de 2014 14:56, Ralph Teixeira 
> escreveu:
>
> Hmmm... mas cuidado: o problema não parece informar que somas correspondem
>> a que combinações das variáveis, então tem um pouco mais do que um sistema
>> de equações aí.
>>
>> Então o problema agora é o seguinte: seja s=(s1, s2, s3, ..., s6) o vetor
>> de somas do lado direito do seu sistema. Você consegue mostrar que, ao
>> permutar as somas, não aparecem soluções novas? Ou seja, que permutações
>> dessas 6 somas levam a permutações da solução antiga OU a um sistema
>> impossível?
>>
>> Abraço,
>>Ralph
>>
>> 2014-10-20 9:16 GMT-02:00 Pedro José :
>>
>> Boa tarde!
>>>
>>> (a)   Ax=b| 1 1 00 | |a|| r|
>>>  |10100 |  |b|   |s|
>>>  | 1 1 00 | |c|  =   |t |
>>>  | 1 1 00 | |a|  | r |
>>>
>>>
>>> Trabalhando a matriz A sem alterar seu posto, 2a = 1a - 2a; 3a = -3a +
>>> 1a -2a; 4a = 4a - 2a + 2 . 3a teremos a matriz A'
>>>
>>> |  1  1  0  0  |
>>> |  0  1 -1  0  |
>>> !  0  0  0 -1  |
>>> |  0  0  0  -2 |
>>> |  0  1  0  1  |
>>> |  0  0  1  1  |
>>>
>>> è fácil perceber que as 4 primeiras linhas são linearmente
>>> idenpendentes, logo posto (A) = 4 ==> dim(Im(f)) = 4
>>>
>>> f: V --> W ==> dim (V) = dim (N) + dim (Im(f)) 4 = din(N) + 4 ==> dim
>>> (N) = 0; logo ou só há uma solução ou é impossível.
>>> Mas como há a solução, que seria a escolha do jogador A. O sistema tem
>>> solção única.
>>>
>>> R; Não é possível o jogador escolher quatro números que tornem
>>> impossível o jogador B ganhar.
>>>
>>> Fazendo o item 2, também é facil mostrar que o posto da matriz A seria 5
>>> e novamente a resposta é a mesma.
>>>
>>>
>>>
>>> Em 17 de outubro de 2014 09:08, benedito  escreveu:
>>>
>>>> *Problema para o Nível  I  - (De uma lista de problemas para
>>>> treinamento da OMA)*
>>>>
>>>> (a)Dois jogadores, A e B, disputam o seguinte jogo:
>>>>
>>>> · O jogador A escolhe 4 números naturais distintos e escreve
>>>> num papel todas as somas de dois desses números (são 6 números)
>>>>
>>>> · O jogador B ganha se encontra os 4 números escolhidos por A;
>>>> caso contrário, ganha o jogador A.
>>>>
>>>>   O jogador A pode escolher os 4 números para que seja impossível B
>>>> ganhar?
>>>>
>>>> (b)   No mesmo jogo descrito em (a), mas agora o jogador A escolhe 5
>>>> números naturais distintos e escreve as 10 somas de dois dos números.
>>>> Novamente, determinar se o jogador A pode escolher os 5 números para que
>>>> seja impossível o jogador  B ganhar.
>>>>
>>>>
>>>>
>>>>
>>>> --
>>>> <http://www.avast.com/>
>>>>
>>>> Este email está limpo de vírus e malwares porque a proteção do avast!
>>>> Antivírus <http://www.avast.com/> está ativa.
>>>>
>>>>
>>>> --
>>>> Esta mensagem foi verificada pelo sistema de antivírus e
>>>> acredita-se estar livre de perigo.
>>>>
>>>
>>>
>>> --
>>> Esta mensagem foi verificada pelo sistema de antivírus e
>>> acredita-se estar livre de perigo.
>>
>>
>>
>> --
>> Esta mensagem foi verificada pelo sistema de antivírus e
>> acredita-se estar livre de perigo.
>>
>
>
> --
> Esta mensagem foi verificada pelo sistema de antivírus e
> acredita-se estar livre de perigo.
>

-- 
Esta mensagem foi verificada pelo sistema de antiv�rus e
 acredita-se estar livre de perigo.



Re: [obm-l] Problema Legal

2014-10-20 Por tôpico Ralph Teixeira
Hmmm... mas cuidado: o problema não parece informar que somas correspondem
a que combinações das variáveis, então tem um pouco mais do que um sistema
de equações aí.

Então o problema agora é o seguinte: seja s=(s1, s2, s3, ..., s6) o vetor
de somas do lado direito do seu sistema. Você consegue mostrar que, ao
permutar as somas, não aparecem soluções novas? Ou seja, que permutações
dessas 6 somas levam a permutações da solução antiga OU a um sistema
impossível?

Abraço,
   Ralph

2014-10-20 9:16 GMT-02:00 Pedro José :

> Boa tarde!
>
> (a)   Ax=b| 1 1 00 | |a|| r|
>  |10100 |  |b|   |s|
>  | 1 1 00 | |c|  =   |t |
>  | 1 1 00 | |a|  | r |
>
>
> Trabalhando a matriz A sem alterar seu posto, 2a = 1a - 2a; 3a = -3a + 1a
> -2a; 4a = 4a - 2a + 2 . 3a teremos a matriz A'
>
> |  1  1  0  0  |
> |  0  1 -1  0  |
> !  0  0  0 -1  |
> |  0  0  0  -2 |
> |  0  1  0  1  |
> |  0  0  1  1  |
>
> è fácil perceber que as 4 primeiras linhas são linearmente  idenpendentes,
> logo posto (A) = 4 ==> dim(Im(f)) = 4
>
> f: V --> W ==> dim (V) = dim (N) + dim (Im(f)) 4 = din(N) + 4 ==> dim (N)
> = 0; logo ou só há uma solução ou é impossível.
> Mas como há a solução, que seria a escolha do jogador A. O sistema tem
> solção única.
>
> R; Não é possível o jogador escolher quatro números que tornem impossível
> o jogador B ganhar.
>
> Fazendo o item 2, também é facil mostrar que o posto da matriz A seria 5 e
> novamente a resposta é a mesma.
>
>
>
> Em 17 de outubro de 2014 09:08, benedito  escreveu:
>
>> *Problema para o Nível  I  - (De uma lista de problemas para treinamento
>> da OMA)*
>>
>> (a)Dois jogadores, A e B, disputam o seguinte jogo:
>>
>> · O jogador A escolhe 4 números naturais distintos e escreve num
>> papel todas as somas de dois desses números (são 6 números)
>>
>> · O jogador B ganha se encontra os 4 números escolhidos por A;
>> caso contrário, ganha o jogador A.
>>
>>   O jogador A pode escolher os 4 números para que seja impossível B
>> ganhar?
>>
>> (b)   No mesmo jogo descrito em (a), mas agora o jogador A escolhe 5
>> números naturais distintos e escreve as 10 somas de dois dos números.
>> Novamente, determinar se o jogador A pode escolher os 5 números para que
>> seja impossível o jogador  B ganhar.
>>
>>
>>
>>
>> --
>> 
>>
>> Este email está limpo de vírus e malwares porque a proteção do avast!
>> Antivírus  está ativa.
>>
>>
>> --
>> Esta mensagem foi verificada pelo sistema de antivírus e
>> acredita-se estar livre de perigo.
>>
>
>
> --
> Esta mensagem foi verificada pelo sistema de antivírus e
> acredita-se estar livre de perigo.

-- 
Esta mensagem foi verificada pelo sistema de antiv�rus e
 acredita-se estar livre de perigo.



Re: [obm-l] Problema

2014-10-18 Por tôpico Ralph Teixeira
Sejam a e c os dois lados mencionados, e d a diagonal. Note que a área A do
quadrilátero satisfaz (pense dois triângulos, um com lados a e d, outro com
lados c e d):

A <= da/2 + dc/2 = d(16-d)/2

Mas esta última expressão é no máximo 32, que só ocorre quando d=8. Então
tem que valer a igualdade 2A= da+dc, ou seja, o ângulo desta diagonal com
os lados a e c têm de ser retos.

Agora uma figura e um Pitágoras mostram que a outra diagonal tem de valer
8√2.

Abraço, Ralph.
On Oct 16, 2014 12:45 PM, "Mauricio Barbosa"  wrote:

> Boa tarde amigos,
> alguém poderia me ajudar com o problema:
> Em um quadrilátero convexo de área 32cm2, a soma dos comprimentos de dois
> lados opostos mais uma diagonal é 16 cm. Determine os valores possíveis
> para a outra diagonal.
> Obrigado!!!
>
> --
> Esta mensagem foi verificada pelo sistema de antivírus e
> acredita-se estar livre de perigo.

-- 
Esta mensagem foi verificada pelo sistema de antiv�rus e
 acredita-se estar livre de perigo.



[obm-l] Re: [obm-l] Intervalo no qual f é crescente

2014-10-13 Por tôpico Ralph Teixeira
Eu nao chequei, mas aqui estah uma possibilidade de resposta, pp.13-19:
http://scholarworks.gsu.edu/cgi/viewcontent.cgi?article=1043&context=math_theses

2014-10-13 19:39 GMT-03:00 Amanda Merryl :
> Oi amigos,
>
> Vamos analisar a seguinte afirmação:
>
> Suponhamos que a função real f seja contínua no intervalo [a, b] e que f(a) < 
> f(b). Existe então um subintervalo de [a, b] no qual f é crescente.
>
> Embora isto aparentemente seja verdade, me garantiram que é falso, mas não 
> tenho um contra exemplo. Alguém pode ajudar? Ou a afirmação é mesmo 
> verdadeira? Se for, a prova parece difícil.
>
> Vamos agora substituir contínua por diferenciável. Pelo teorema do valor 
> médio, existe então u em (a, b) tal que f'(u) = (f(b) - f(a)/(b - a) > 0. Se 
> admitirmos que f' é contínua, então existe um subintervalo I de [a, b] 
> contendo u no qual f' é positiva, o que implica que f seja estritamente 
> crescente em I. Logo, a afirmação torna-se verdadeira. Na realidade, para 
> tanto basta admitir que f' é contínua em algum u com f'(u) > 0. E ao menos um 
> deles existe
>
> Mas, e se tudo que assumirmos é que f é diferenciável?
>
> Obrigada
>
> Amanda
>
>
>
> --
> Esta mensagem foi verificada pelo sistema de antivírus e
>  acredita-se estar livre de perigo.
>
>
> =
> Instru�ões para entrar na lista, sair da lista e usar a lista em
> http://www.mat.puc-rio.br/~obmlistas/obm-l.html
> =

-- 
Esta mensagem foi verificada pelo sistema de antiv�rus e
 acredita-se estar livre de perigo.


=
Instru��es para entrar na lista, sair da lista e usar a lista em
http://www.mat.puc-rio.br/~obmlistas/obm-l.html
=


[obm-l] Re: [obm-l] Função O(x)

2014-09-05 Por tôpico Ralph Teixeira
Ajuda?

http://en.wikipedia.org/wiki/Big_O_notation


2014-09-05 21:06 GMT-03:00 João Sousa :

> Pessoal, alguém poderia me indicar um material em português, ou mesmo
> explicar aquela função O(x) que aparece em algumas explicações na
> matemática.
>
> Estou fazendo um curso de estatística  e vejo frequentemente essa função.
> Como na fórmula de Stirling para a aproximação de fatorial.
>
> n! = n^n  exp(-n) sqrt(2 pi n)[1+ O(n^-1)]
>
> Desde já fico muito grato pela atenção.
>
> João
>
> --
> Esta mensagem foi verificada pelo sistema de antivírus e
> acredita-se estar livre de perigo.
>

-- 
Esta mensagem foi verificada pelo sistema de antiv�rus e
 acredita-se estar livre de perigo.



[obm-l] Re: [obm-l] Re: [obm-l] RES: [obm-l] Problema da Olimpiada de Matemática de Moscou

2014-09-03 Por tôpico Ralph Teixeira
Ah, eh verdade, dah para acelerar MUITO notando que:

S(x) = x (mod 9)

Entao x+S(x)+S(S(x)) = 3x (mod 9)

Isto eh, x+S(x)+S(S(x)) eh sempre divisivel por 3 -- e portanto nunca pode
ser 1993.

Abraco,
 Ralph


2014-09-03 19:42 GMT-03:00 Mauricio de Araujo 
:

> não tem solução!! hehehe
>
>
> 2014-09-03 19:07 GMT-03:00 Albert Bouskela :
>
>> Olá!
>>
>>
>>
>> A melhor solução é pelo “cheiro”
>>
>>
>>
>> 1) x>1899 | 1899+S(1899)+SS(1899)=1935<<1993
>>
>> 2) x>1959 | 1959+S(1959)+SS(1959)=1989<1993
>>
>> 3) S≥16 (x=1960) e SS≥2 (S=20)
>>
>> 4) x≤1993-16-2=1975
>>
>> 5) 1960≤x≤1975
>>
>> 6) Agora é no braço…
>>
>> 7) Mas há uma surpresa no final!
>>
>>
>> --
>>
>> *Albert Bouskelá*
>>
>> bousk...@ymail.com
>>
>>
>>
>> *De:* owner-ob...@mat.puc-rio.br [mailto:owner-ob...@mat.puc-rio.br] *Em
>> nome de *Mauricio de Araujo
>> *Enviada em:* quarta-feira, 3 de setembro de 2014 11:36
>> *Para:* obm-l@mat.puc-rio.br
>> *Assunto:* [obm-l] Problema da Olimpiada de Matemática de Moscou
>>
>>
>>
>> Seja S(x) a soma dos dígitos de um inteiro positivo x.
>>
>>
>>
>> Resolver: x + S(x) + S(S(x)) = 1993.
>>
>>
>>
>> --
>>
>> Abraços
>>
>>
>> oɾnɐɹɐ ǝp oıɔıɹnɐɯ
>>
>>
>>
>>
>> --
>> Esta mensagem foi verificada pelo sistema de antiv?s e
>> acredita-se estar livre de perigo.
>>
>>
>> --
>> Esta mensagem foi verificada pelo sistema de antivírus e
>> acredita-se estar livre de perigo.
>>
>
>
>
> --
> Abraços
>
> oɾnɐɹɐ ǝp oıɔıɹnɐɯ
>
>
> --
> Esta mensagem foi verificada pelo sistema de antivírus e
> acredita-se estar livre de perigo.

-- 
Esta mensagem foi verificada pelo sistema de antiv�rus e
 acredita-se estar livre de perigo.



[obm-l] Re: [obm-l] Problema da Olimpiada de Matemática de Moscou

2014-09-03 Por tôpico Ralph Teixeira
Claramente, x<=1993.
Então S(x)<=1+9+9+9=28,
e portanto S(S(x))<=1+9=2+8=10.
Portanto, 1993-38=1955<=x<=1993, isto é, x="19ab" onde 38<="ab"<=93.

Então reestimo S(x)=1+9+a+b entre 1+9+4+0 e 1+9+8+9, isto é, em [14,27],
e portanto S(S(X)) entre 2+0 e 1+9, isto é, em [2,10]
Portanto, x está entre 1993-37 e 1993-16, isto é, x está em [1966,1977].

Então reestimo... Ah, chega de reestimar: experimente logo todos os 12
números ali dentro rapidinho e termine o problema. :)

Abraço,
  Ralph


2014-09-03 11:36 GMT-03:00 Mauricio de Araujo 
:

> Seja S(x) a soma dos dígitos de um inteiro positivo x.
>
> Resolver: x + S(x) + S(S(x)) = 1993.
>
> --
> Abraços
>
> oɾnɐɹɐ ǝp oıɔıɹnɐɯ
>
>
> --
> Esta mensagem foi verificada pelo sistema de antivírus e
> acredita-se estar livre de perigo.

-- 
Esta mensagem foi verificada pelo sistema de antiv�rus e
 acredita-se estar livre de perigo.



[obm-l] Re: [obm-l] Re: [obm-l] Combinatória - escadas

2014-08-18 Por tôpico Ralph Teixeira
Hmm... Mas N(0)=1, certo? Entao fico com:

N(3) = N(2)+N(1)+N(0) = 2+1+1 = 4
N(4) = N(3)+N(2)+N(1) = 4+2+1 = 7
N(5) = N(4)+N(3)+N(2) = 7+4+2 = 13
N(6) = 24

A sequencia eh 1,1,2,4,7,13,24,44,81,... ou seja os numeros de Tribonacci
, porque a OEIS eh genial!

Abraco,
 Ralph

2014-08-18 16:15 GMT-03:00 Mauricio de Araujo :
> Pense assim, ele está no sexto degrau.. para se chegar ao sexto degrau ou
> ele veio do quinto, ou do quarto ou terceiro degrau...
>
> assim, o total de maneiras de se chegar no sexto degrau, N(6) será igual a
> N(5)+N(4)+N(3)...
>
> N(3) = N(2)+N(1)+N(0) = 2+1+0 = 3
> N(4) = N(3)+N(2)+N(1) = 3+2+1 = 6
> N(5) = N(4)+N(3)+N(2) = 6+3+2 = 11
>
> N(6) = 20...
>
> acho que é isso..
>
>
>
> 2014-08-18 15:37 GMT-03:00 Marcos Xavier :
>>
>> s degraus para chegar em casa. Como tem a
>
>
>
>
>
> --
> Abraços
>
> oɾnɐɹɐ ǝp oıɔıɹnɐɯ
>
>
> --
> Esta mensagem foi verificada pelo sistema de antivírus e
> acredita-se estar livre de perigo.

-- 
Esta mensagem foi verificada pelo sistema de antiv�rus e
 acredita-se estar livre de perigo.



Re: [obm-l] Primos entre si

2014-08-09 Por tôpico Ralph Teixeira
Suponho que a e b sejam distintos... Entao suponho b>a. Tome n=p-a, onde p
eh um primo maior que ambos a e b.
On Aug 8, 2014 8:01 PM, "marcone augusto araújo borges" <
marconeborge...@hotmail.com> wrote:

> Mostre que existem infinitos n tais que a + n e b + n são primos entre si
>
>
> --
> Esta mensagem foi verificada pelo sistema de antivírus e
> acredita-se estar livre de perigo.
>

-- 
Esta mensagem foi verificada pelo sistema de antiv�rus e
 acredita-se estar livre de perigo.



Re: [obm-l] mdc(a^n - 1, a^m - 1) = a^d - 1

2014-07-09 Por tôpico Ralph Teixeira
Hmmm Eu acho que o seguinte eh verdadeiro:

Lema: Considere a seguinte iteracao: dado o conjunto {x,y} com x>y>0,
troque-o por {x,x-y}. Eu afirmo que voce pode repetir esta iteracao
ateh ficar com o conjunto unitario {d} onde d=mdc{x_original,
y_original}.
Dem.: Pense como funciona o algoritmo para encontrar m.d.c., mas ao
inves de dividir x por y para achar x=qy+d, subtraia y de x (q vezes)
ateh ficar com d.

Agora, supondo n>m, qualquer divisor comum de a^n-1 e a^m-1 tem que
dividir a diferenca (a^n-a^m)=a^m (a^(n-m)-1), supondo n>m. Como a^m
eh primo com a^m-1, concluo que ele tem que ser divisor de a^(n-m)-1.

Alias, VAI E VOLTA: supondo n>m, b eh divisor comum de a^n-1 e a^m-1
SE, E SOMENTE SE, b eh divisor comum de a^m-1 e a^(n-m)-1.

Ou seja, os divisores comuns dessas expressoes nao mudam ao perfazer a
operacao de trocar {m,n} por {m,n-m}. Itere esta ideia e voce vai
chegar que b tem que ser divisor de a^d-1 onde d=mdc{m,n}. Entao nao
ha nada maior mesmo.

Abraco,
 Ralph


2014-07-09 14:42 GMT-03:00 Arthur Max :
> oi
>
> Em 08/07/14, Artur Costa Steiner escreveu:
>> De nada!
>>
>> Podemos concluir de bate pronto que, dentre os divisores comuns de a^m - 1 e
>> a^n - 1 que sejam da forma a^r - 1, o maior 茅 a^d - 1. Mas n茫o sei pode
>> haver um divisor comum > a^ d - 1 que n茫o seja da forma a^r - 1. Vou
>> analisar mais.
>>
>> Artur Costa Steiner
>>
>>> Em 08/07/2014, 脿s 09:04, Pedro Chaves  escreveu:
>>>
>>> Muito obrigado, caro Artur, pela demonstra莽茫o do teorema abaixo:
>>> Teorema:
>>> Sendo a, n e m inteiros positivos, com a> 1, a^n - 1 divide a^m - 1 se, e
>>> somente se, n divide m.
>>>
>>> Bem... usando-se esse teorema, seria poss铆vel demonstrar que o
>>> mdc(a^n- 1, a^m - 1)= a^d - 1, sendo d = mdc(m, n)?
>>>
>>> Abra莽os do pedro Chaves!
>>> ___
>>>
>>>
>>>
>>> --
>>> Esta mensagem foi verificada pelo sistema de antiv铆rus e
>>> acredita-se estar livre de perigo.
>>>
>>>
>>> =
>>> Instru莽玫es para entrar na lista, sair da lista e usar a lista em
>>> http://www.mat.puc-rio.br/~obmlistas/obm-l.html
>>> =
>>
>> --
>> Esta mensagem foi verificada pelo sistema de antiv铆rus e
>>  acredita-se estar livre de perigo.
>>
>>
>> =
>> Instru锟矫礶s para entrar na lista, sair da lista e usar a lista em
>> http://www.mat.puc-rio.br/~obmlistas/obm-l.html
>> =
>>
>
> --
> Esta mensagem foi verificada pelo sistema de antiv韗us e
>  acredita-se estar livre de perigo.
>
>
> =
> Instru珲es para entrar na lista, sair da lista e usar a lista em
> http://www.mat.puc-rio.br/~obmlistas/obm-l.html
> =

-- 
Esta mensagem foi verificada pelo sistema de antiv�rus e
 acredita-se estar livre de perigo.


=
Instru��es para entrar na lista, sair da lista e usar a lista em
http://www.mat.puc-rio.br/~obmlistas/obm-l.html
=


Re: [obm-l] Mais uma de diferenciabilidade

2014-07-04 Por tôpico Ralph Teixeira
Estou pensando em algo com o seguinte espirito (mas tem que examinar
todos os detalhes e ver se funciona mesmo)!

1. Suponha que f'(a) NAO EH L. Entao existe alguma sequencia (que,
passando uma subsequencia se necessario, pode ser tomada monotona --
vou supor spdg decrescente) z_n -> a (com z_n <>a) tal que lim
(f(z_n)-f(a)) / (z_n-a) nao eh L.
2. Se a sequencia dos numeros (f(z_n)-f(a))/(z_n-a) for ilimitada,
passe outra sub para que ele o limite dela seja +Inf ou -Inf; se for
limitada, ela tem que ter um ponto de acumulacao que nao eh L, entao
passe uma subsequencia para que o limite seja um numero A diferente de
L.

Em suma, neste momento temos uma sequencia z_n->a decrescente tal que
lim (f(z_n)-f(a)) / (z_n-a) = A <> L.

3. Ideia: tome y_n=z_n. Agora, para CADA y_n fixo, vamos escolher x_n
MUITO PERTO de a, tal que f(x_n) esteja MUITO MUITO PERTO de f(a).
Assim, teremos algo do tipo
[f(y_n) - f(x_n)]/[y_n-x_n] ~~~ [f(y_n) - f(a)]/[y_n-a] ~~~ A, que
estaria **longe** de L. Pronto, isto seria uma contradicao frente aa
hipotese dada!

Vejamos os detalhes, pelo menos no caso em que A eh finito: vou
denotar B_n=[f(y_n) - f(a)]/[y_n-a] e D=|A-L|>0.
i) Primeiro passe outra subsequencia de forma a garantir que |B_n  -
A| < D/4. Isto eh para garantir que este negocio estah realmente longe
de L (e eh possivel porque o limite de B_n eh A quando n->Inf, entao
eh soh cortar o comeco da sequencia e deixar um rabo conveniente).
ii) Agora, para um y_n fixo, note que lim (x->a) (f(y_n) - f(x))/(y_n
- x) = B_n. Entao, para x suficientemente proximo de a, temos
|(f(y_n)-f(x)) / (y_n-x) - B_n | |A-L| - |B_n-A| - |[f(y_n) -
f(x_n)]/[y_n-x_n] - B_n| > D - D/4 - D/4 > D/2
e portanto o limite desta fracao ali na esquerda nao serah L, absurdo.


(Agora falta fazer um raciocinio analogo no caso em que A=+-Inf! Mas
tenho certeza que sai, deve ateh ser mais facil do que esse que eu
fiz.)

Abraco, Ralph.

2014-07-04 21:35 GMT-03:00 Merryl :
> Boa noite amigos
>
> Obrigada a todos pela ajuda naquele outro problema.
>
> Gostaria de ajuda com este aqui. Já pensei mas não consegui provar.
>
> Seja f:I --> R contínua no ponto a do intervalo aberto I. Suponhamos que
> para todas sequências (x_n) e (y_n) em I tais que
>
> (x_n) seja crescente e convirja para a
>
> (y_n) seja decrescente e convirja para a
>
> x_n < a < y_n para todo n
>
> exista um mesmo real L para o qual convirjam os quocientes ((f(y_n) -
> f(x_n))/(y_n - x_n)).
>
> Mostre que f é diferenciável em a e que f'(a) = L
>
> Eu tentei partir o quociente acima em duas partes, escrevendo-o como
>
> (f(y_n) - f(a))/(y_n - a) (y_n - a)/(y_n - x_n) - (f(x_n) - f(a))/(x_n - a)
> (x_n - a)/(y_n - x_n)
>
> Mas como a hipótese é que f é só contínua em a, não se assume
> diferenciabilidade, isto não permite chegar a f'(a) = L.
>
> Obrigada
>
> --
> Esta mensagem foi verificada pelo sistema de antivírus e
> acredita-se estar livre de perigo.

-- 
Esta mensagem foi verificada pelo sistema de antiv�rus e
 acredita-se estar livre de perigo.


=
Instru��es para entrar na lista, sair da lista e usar a lista em
http://www.mat.puc-rio.br/~obmlistas/obm-l.html
=


[obm-l] Re: [obm-l] RE: [obm-l] Re: [obm-l] Re: [obm-l] Re: [obm-l] Este limite é igual a f'(a) ?

2014-06-28 Por tôpico Ralph Teixeira
De fato, se f for derivavel em uma vizinhanca de a, a gente pode usar o TVM
e obter:

f(a+g(x))-f(a+h(x)) / (g(x)-h(x))= f'(c(x))

para x suficientemente proximo de 0, onde c(x) eh algum numero entre a+g(x)
e a+h(x). Tomando x->0, tem-se g(x) e h(x)->0 e portanto c(x)->a. Se f'(x)
for continua em a, entao f'(c(x))->f'(a).

Isso inclui f polinomial e muitos outros casos tipicos. Por isso que o
contra-exemplo teve de ser tao chato.

Abraco, Ralph.


2014-06-28 22:43 GMT-03:00 Artur Costa Steiner :

> Acho que sim. Uma forma um pouco diferente de provar a mesma coisa.
>
> Creio que, se L = 1 ou o limite em o de g/h não existir, cada caso tem que
> ser analisado individualmente. Eu analisei uns casos com L = 1, casos
> simples, porque para g e h complicadas pode ficar quase que intratável.
> Cheguei de fato a f'(a), mas não tenho nenhuma prova de que isto seja
> sempre verdade. Por exemplo, com f(x) = sen(x), g(x) = x, h (x) = e^x - 1 e
> a = 0, o limite g/h é 1 e o do quociente q é f'(a). Mas, claro, isso não
> prova nada que seja geral.
>
> Se f for polinomial, então o limite do quociente sempre será f'(a) para
> qualquer real a, mesmo que L = 1 ou que o limite de g/h não exista nos
> reais expandidos.
>
> Artur
>
> Enviado do Yahoo Mail para iPad
>
>  --
> * From: * Pacini Bores ;
> * To: * ;
> * Subject: * [obm-l] Re: [obm-l] Re: [obm-l] Re: [obm-l] Este limite é
> igual a f'(a) ?
> * Sent: * Sun, Jun 29, 2014 12:08:35 AM
>
>Será que  eu poderia ver também o que o Artur concluiu, como abaixo?
>
>  Para L diferente de 1?
>
> ( vou escrever sem o x, para facilitar).
>
> O limite pedido  pode ser escrito como :
>
> lim{[ (f(a+g)-f(a))/g][g/h] - [( f(a+h)-f(a))/h]}/(g/h-1) = (f´(a).L -
> f´(a))(L-1)= f´(a).
>
>
> E para L=1, ficaríamos ainda sem condições de levantar  o símbolo de
> indeterminação oo/oo.
>
> Abraços
>
> Pacini
>
>
>
> Em 26 de junho de 2014 15:50, Ralph Teixeira  escreveu:
>
>> Pois é. Em espírito, a minha ideia foi tomar a+g(x) MUITO PERTO de
>> a+h(x), de forma que aquele quociente está muito mais para f'(a+g(x))
>> (ou f´(a+h(x)), sei lá, eles estão MUUUITO PERTO um do outro) do que
>> para f'(a). Como o Artur disse, é importante no exemplo que g(x)~h(x)
>> (MUUUITO PERTO), o que fica bem traduzido por lim g/h=1.
>>
>> Enfim, se f´(x) não for contínua, f'(a+g(x)) não precisa se aproximar
>> de f'(a) quando g(x) vai para 0, e daí vem o problema todo -- minha
>> função não é C1.
>>
>> 2014-06-26 15:08 GMT-03:00 Artur Costa Steiner :
>> > Se lim (x --> 0) g(x)/h(x) = L <> 1 no sistema dos reais expandidos,
>> então a
>> > resposta é sim.
>> >
>> > Utilizando a notação o, a diferenciabilidade de f em a implica que, para
>> > todo h tal que a + x permaneça no domínio de f, tenhamos
>> >
>> > f(a + h) = f(a) + f'(a) h + o(h), sendo o uma função contínua em 0 tal
>> que
>> > o(0) = 0 e tal que o(h)/h --> 0 quando h --> 0.  Com isto, o seu
>> quociente
>> > de Newton generalizado q torna-se
>> >
>> > q(x) = (f(a) + f'(a) g(x) + o(g(x)) - (f(a) + f'(a) h(x) +
>> o(h(x)))/(g(x) -
>> > h(x)) =
>> >
>> > f'(a) + (o(g(x) - o(h(x))/(g(x) - h(x))
>> >
>> > Suponhamos que lim ( x --> 0) g(x)/h(x) = L em R, diferente de 1. Então,
>> > existe uma vizinhança deletada de 0 na qual h não se anula e, dividindo
>> o
>> > numerador e o denominador por h(x), seu quociente de Newton generalizado
>> > pode então ser escrito como
>> >
>> > q(x) = f'(a) + (o(g(x))/g(x) g(x)/h(x) - o(h(x)/h(x))/(g(x)/h(x) - 1)
>> >
>> > Assim,
>> >
>> > lim ( x --> 0) q(x) = f'(a) + (0 . L - 0)/(L - 1) = f'(a) + 0 = f'(a)
>> >
>> > Bateu!!
>> >
>> > Se L = + ou - oo, então lim (x -->0) g(x)/h(x) = 0. Divindindo agora em
>> cima
>> > e em baixo por g(x), que não se anula em uma vizinhança deletada de 0,
>> um
>> > raciocínio similar ao anterior mostra que bate de novo com f'(a).
>> >
>> > Mas pode acontecer que L = 1 ou que, quando x --> 0, não exista lim
>> > g(x)/h(x). E agora, José?
>> >
>> > Eu fiz uns testes com L = 1 e bateu, mas não sei se é sempre verdade
>> não. O
>> > raciocínio acima não serve para L = 1, pois o denominador tende a 0. E
>> se o
>> > limite de g(x)/h(x) não existir em 0,  a coisa parece ainda mas
>> complicada.
>

[obm-l] Re: [obm-l] Re: [obm-l] Este limite é igual a f'(a) ?

2014-06-26 Por tôpico Ralph Teixeira
Pois é. Em espírito, a minha ideia foi tomar a+g(x) MUITO PERTO de
a+h(x), de forma que aquele quociente está muito mais para f'(a+g(x))
(ou f´(a+h(x)), sei lá, eles estão MUUUITO PERTO um do outro) do que
para f'(a). Como o Artur disse, é importante no exemplo que g(x)~h(x)
(MUUUITO PERTO), o que fica bem traduzido por lim g/h=1.

Enfim, se f´(x) não for contínua, f'(a+g(x)) não precisa se aproximar
de f'(a) quando g(x) vai para 0, e daí vem o problema todo -- minha
função não é C1.

2014-06-26 15:08 GMT-03:00 Artur Costa Steiner :
> Se lim (x --> 0) g(x)/h(x) = L <> 1 no sistema dos reais expandidos, então a
> resposta é sim.
>
> Utilizando a notação o, a diferenciabilidade de f em a implica que, para
> todo h tal que a + x permaneça no domínio de f, tenhamos
>
> f(a + h) = f(a) + f'(a) h + o(h), sendo o uma função contínua em 0 tal que
> o(0) = 0 e tal que o(h)/h --> 0 quando h --> 0.  Com isto, o seu quociente
> de Newton generalizado q torna-se
>
> q(x) = (f(a) + f'(a) g(x) + o(g(x)) - (f(a) + f'(a) h(x) + o(h(x)))/(g(x) -
> h(x)) =
>
> f'(a) + (o(g(x) - o(h(x))/(g(x) - h(x))
>
> Suponhamos que lim ( x --> 0) g(x)/h(x) = L em R, diferente de 1. Então,
> existe uma vizinhança deletada de 0 na qual h não se anula e, dividindo o
> numerador e o denominador por h(x), seu quociente de Newton generalizado
> pode então ser escrito como
>
> q(x) = f'(a) + (o(g(x))/g(x) g(x)/h(x) - o(h(x)/h(x))/(g(x)/h(x) - 1)
>
> Assim,
>
> lim ( x --> 0) q(x) = f'(a) + (0 . L - 0)/(L - 1) = f'(a) + 0 = f'(a)
>
> Bateu!!
>
> Se L = + ou - oo, então lim (x -->0) g(x)/h(x) = 0. Divindindo agora em cima
> e em baixo por g(x), que não se anula em uma vizinhança deletada de 0, um
> raciocínio similar ao anterior mostra que bate de novo com f'(a).
>
> Mas pode acontecer que L = 1 ou que, quando x --> 0, não exista lim
> g(x)/h(x). E agora, José?
>
> Eu fiz uns testes com L = 1 e bateu, mas não sei se é sempre verdade não. O
> raciocínio acima não serve para L = 1, pois o denominador tende a 0. E se o
> limite de g(x)/h(x) não existir em 0,  a coisa parece ainda mas complicada.
> Como o Ralph esquematizou, acho que nestes casos pode dar qualquer coisa. O
> limite de q pode ser f'(a) como outro valor, como pode nem existir.
>
> Abraços
>
> Artur
>
>
>
> Em terça-feira, 24 de junho de 2014, Merryl  escreveu:
>>
>> Boa noite, amigos. Gostaria de ajuda com isto,
>>
>> Seja f uma função de R em R, diferenciável em a. Sejam g e h funções
>> contínuas em 0 tais que g(0) = h(0) = 0. Suponhamos que exista uma
>> vizinhança deletada de 0 na qual g - h não se anule. Então, é verdade que
>>
>> lim (x --> 0) [f(a + g(x)) - f(a + h(x))]/[g(x) - h(x)]  = f'(a) ?
>>
>> Não tenho certeza. Este limite tem que existir? Se existir, é de fato
>> f'(a)?
>>
>> Obrigada
>>
>> Amanda
>>
>> --
>> Esta mensagem foi verificada pelo sistema de antivírus e
>> acredita-se estar livre de perigo.
>
>
> --
> Esta mensagem foi verificada pelo sistema de antivírus e
> acredita-se estar livre de perigo.

-- 
Esta mensagem foi verificada pelo sistema de antiv�rus e
 acredita-se estar livre de perigo.


=
Instru��es para entrar na lista, sair da lista e usar a lista em
http://www.mat.puc-rio.br/~obmlistas/obm-l.html
=


[obm-l] Re: [obm-l] Re: [obm-l] Este limite é igual a f'(a) ?

2014-06-26 Por tôpico Ralph Teixeira
Pois é. Em espírito, a minha ideia foi tomar a+g(x) MUITO PERTO de
a+h(x), de forma que aquele quociente está muito mais para f'(a+g(x))
(ou f´(a+h(x)), sei lá, eles estão MUUUITO PERTO um do outro) do que
para f'(a). Como o Artur disse, é importante no exemplo que g(x)~h(x)
(MUUUITO PERTO), o que fica bem traduzido por lim g/h=1.

Enfim, se f´(x) não for contínua, f'(a+g(x)) não precisa se aproximar
de f'(a) quando g(x) vai para 0, e daí vem o problema todo -- minha
função não é C1.

2014-06-26 15:08 GMT-03:00 Artur Costa Steiner :
> Se lim (x --> 0) g(x)/h(x) = L <> 1 no sistema dos reais expandidos, então a
> resposta é sim.
>
> Utilizando a notação o, a diferenciabilidade de f em a implica que, para
> todo h tal que a + x permaneça no domínio de f, tenhamos
>
> f(a + h) = f(a) + f'(a) h + o(h), sendo o uma função contínua em 0 tal que
> o(0) = 0 e tal que o(h)/h --> 0 quando h --> 0.  Com isto, o seu quociente
> de Newton generalizado q torna-se
>
> q(x) = (f(a) + f'(a) g(x) + o(g(x)) - (f(a) + f'(a) h(x) + o(h(x)))/(g(x) -
> h(x)) =
>
> f'(a) + (o(g(x) - o(h(x))/(g(x) - h(x))
>
> Suponhamos que lim ( x --> 0) g(x)/h(x) = L em R, diferente de 1. Então,
> existe uma vizinhança deletada de 0 na qual h não se anula e, dividindo o
> numerador e o denominador por h(x), seu quociente de Newton generalizado
> pode então ser escrito como
>
> q(x) = f'(a) + (o(g(x))/g(x) g(x)/h(x) - o(h(x)/h(x))/(g(x)/h(x) - 1)
>
> Assim,
>
> lim ( x --> 0) q(x) = f'(a) + (0 . L - 0)/(L - 1) = f'(a) + 0 = f'(a)
>
> Bateu!!
>
> Se L = + ou - oo, então lim (x -->0) g(x)/h(x) = 0. Divindindo agora em cima
> e em baixo por g(x), que não se anula em uma vizinhança deletada de 0, um
> raciocínio similar ao anterior mostra que bate de novo com f'(a).
>
> Mas pode acontecer que L = 1 ou que, quando x --> 0, não exista lim
> g(x)/h(x). E agora, José?
>
> Eu fiz uns testes com L = 1 e bateu, mas não sei se é sempre verdade não. O
> raciocínio acima não serve para L = 1, pois o denominador tende a 0. E se o
> limite de g(x)/h(x) não existir em 0,  a coisa parece ainda mas complicada.
> Como o Ralph esquematizou, acho que nestes casos pode dar qualquer coisa. O
> limite de q pode ser f'(a) como outro valor, como pode nem existir.
>
> Abraços
>
> Artur
>
>
>
> Em terça-feira, 24 de junho de 2014, Merryl  escreveu:
>>
>> Boa noite, amigos. Gostaria de ajuda com isto,
>>
>> Seja f uma função de R em R, diferenciável em a. Sejam g e h funções
>> contínuas em 0 tais que g(0) = h(0) = 0. Suponhamos que exista uma
>> vizinhança deletada de 0 na qual g - h não se anule. Então, é verdade que
>>
>> lim (x --> 0) [f(a + g(x)) - f(a + h(x))]/[g(x) - h(x)]  = f'(a) ?
>>
>> Não tenho certeza. Este limite tem que existir? Se existir, é de fato
>> f'(a)?
>>
>> Obrigada
>>
>> Amanda
>>
>> --
>> Esta mensagem foi verificada pelo sistema de antivírus e
>> acredita-se estar livre de perigo.
>
>
> --
> Esta mensagem foi verificada pelo sistema de antivírus e
> acredita-se estar livre de perigo.

-- 
Esta mensagem foi verificada pelo sistema de antiv�rus e
 acredita-se estar livre de perigo.


=
Instru��es para entrar na lista, sair da lista e usar a lista em
http://www.mat.puc-rio.br/~obmlistas/obm-l.html
=


[obm-l] Re: [obm-l] Série de Taylor

2014-06-26 Por tôpico Ralph Teixeira
Oi, João.

Bom, você já deve ter feito:

a) sin(x^2)=SUM (-1)^n.x^(4n+2))/(2n+1)! = x^2 -x^6/3! +x^10/5!
-x^14/7!... para todo x real (o somatório começa em n=0)
b) Podemos integrar séries de Potência termo-a-termo, então
Int (0 a x) sin(u^2) du = SUM (-1)^n.x^(4n+3)/[(4n+3).(2n+1)!] = x^3/3
- x^7/(7.3!) +x^11/(11.5!) - x^15/(15.7!) para todo x real
c) Botando x=1, vem que a integral pedida é
A= SUM (-1)^n.x^(4n+3)/[(4n+3).(2n+1)!] = 1/3 - 1/(7.3!) +1/(11.5!)
-1/(15.7!) +...

Agora:
d) Isto é uma série alternada! Se os termos da série forem
decrescentes em módulo (que é o caso aqui) e forem para 0 (idem), dá
para ver que, quando você trunca a série, a diferença entre a série
truncada e a série completa é, em módulo, NO MÁXIMO, o primeiro termo
descartado! Como 15.7! = 75600 > 1, você pode parar no terceiro
termo. Ou seja, a resposta é 1/3-1/42+1/1320 com erro menor que
1/75600.

Abraço, Ralph

P.S.: Para provar o que eu falei sobre a série alternada: suponha a
série é alternada, com termos decrescentes em módulo indo para 0,
(s.p.d.g, suponha que o primeiro termo é positivo). Sendo s1, s2, ...
,sn as somas parciais, e L o limite da série, é fácil ver que
0:
> Alguém pode me ajudar na seguinte questão?
>
> Ache uma aproximação para Integral (0 10^(-4)
>
> Eu achei a expansão de Taylor dessa integral, mas não consegui achar (e
> provar) um erro que fosse menor que 10^(-4)
> Tem como alguém me dar uma ajuda?
>
> []'s
> Joao
>
> --
> Esta mensagem foi verificada pelo sistema de antivírus e
> acredita-se estar livre de perigo.

-- 
Esta mensagem foi verificada pelo sistema de antiv�rus e
 acredita-se estar livre de perigo.


=
Instru��es para entrar na lista, sair da lista e usar a lista em
http://www.mat.puc-rio.br/~obmlistas/obm-l.html
=


[obm-l] Re: [obm-l] Este limite é igual a f'(a) ?

2014-06-26 Por tôpico Ralph Teixeira
Hmmm, acho que se f nao for de classe C1, e resposta eh "nao necessariamente".

Afinal, tome f(x)=x^2.sin(1/x) e a=0. Dah para mostrar que f'(0)=0, certo?

Agora para k=1,2,3,..., tome xk=1/(2kpi+pi/2) e yk=1/(2kpi). Entao
f(xk)=xk^2 e f(yk)=0 (no grafico fica claro -- os x_k correspondem aos
"picos" de f(x) e os y_k sao as raizes de f "logo em seguida").

Pois bem, (f(yk)-f(xk)) / (yk-xk) = -xk^2 / (yk-xk) nao vai para 0
quando k->+Inf (faca a conta!), apesar de termos xk->0 e yk->0 quando
k->+Inf.

Entao tomando g(x)=x/(2pi+pi.x/2) e h(x)=x/2pi ou algo assim, vai dar
tudo errado -- sempre que x=1/k, estamos na situacao acima, onde o seu
limite NAO DAH 0, portanto o limite que voce pede NAO DAH 0 (acho que
nao existe).

Outra opcao seria tomar uma funcao escada g(x) que assume os valores
da forma x_k, enquanto h(x) eh uma funcao escada assumindo apenas os
valores y_k. Acertando os detalhes, dah para fazer ambas serem
continuas e irem para 0 quando x->0 (tipo, faca g(x)=x_k se
1/k:
> Boa noite, amigos. Gostaria de ajuda com isto,
>
> Seja f uma função de R em R, diferenciável em a. Sejam g e h funções
> contínuas em 0 tais que g(0) = h(0) = 0. Suponhamos que exista uma
> vizinhança deletada de 0 na qual g - h não se anule. Então, é verdade que
>
> lim (x --> 0) [f(a + g(x)) - f(a + h(x))]/[g(x) - h(x)]  = f'(a) ?
>
> Não tenho certeza. Este limite tem que existir? Se existir, é de fato f'(a)?
>
> Obrigada
>
> Amanda
>
> --
> Esta mensagem foi verificada pelo sistema de antivírus e
> acredita-se estar livre de perigo.

-- 
Esta mensagem foi verificada pelo sistema de antiv�rus e
 acredita-se estar livre de perigo.


=
Instru��es para entrar na lista, sair da lista e usar a lista em
http://www.mat.puc-rio.br/~obmlistas/obm-l.html
=


Re: [obm-l] Limite por l'Hospital

2014-06-23 Por tôpico Ralph Teixeira
Ah, o Saulo fez de outro jeito que funciona. Mas acho que tem um
sinalzinho trocado aqui:

lny=(nln(1+1/n)-1)/(1/n)
lny=(ln(1+1/n)   **-**1/(1+n))/(-1/n^2)


2014-06-23 2:12 GMT-03:00 saulo nilson :
> lim (n -> inf) (1+1/n)^(n²) e^(-n)
> =lim(1+1/n)^n^2* e^-n
> y=lim(1+1/n)^n^2
> lny=limn^2ln(1+1/n) -n
> lny=oo*0-oo
> lny=limn(nln(1+1/n))-1)
> lny=(nln(1+1/n)-1)/(1/n)
> lny=(ln(1+1/n)+1/(1+n))/(-1/n^2)=0/0
> lny=(-1/n*1/(n+1)-1/(n+1)^2)/2/n^3=
> lny=-n^2/2(n+1)*(2n+1)/(n+1))=-limn^2(2n+1)/2(n+1)^2=-oo
> y=e^-00
> y=0
>
>
>
> 2014-06-23 0:43 GMT-03:00 Ralph Teixeira :
>>
>> Vamos ver o ln disso, que eh:
>>
>> g(x)=x^2.ln(1+1/x)-x = x^2 (ln(1+1/x)-1/x) = (ln(1+1/x)-1/x) / (x^(-2))
>>
>> Quando x->+Inf, isto aqui eh uma indet. do tipo 0/0. Note como eu
>> deixei o ln o mais sozinho possivel, por que agora L'Hopital vai
>> simplificar as coisas (se o ln ficar "misturado" com outras coisas,
>> ele nao some na derivada):
>>
>> lim (x->+Inf) g(x) = lim (x->+Inf) ((-1/x^2)(1/(1+1/x))-1/x^2) /
>> (-2x^(-3)) = lim (x->+inf) (-1/2)(x/(x+1)) = -1/2
>>
>> Entao, se eu nao errei conta, o limite original eh e^(-1/2).
>>
>> Abraco,
>>Ralph
>>
>> 2014-06-23 0:17 GMT-03:00 João Maldonado :
>> > Fala galera, tem como alguém me dar uma ajuda no seguinte limite? Faz
>> > uma
>> > horta que estou tentando calcular e não sai.
>> >
>> > lim (n -> inf) (1+1/n)^(n²) e^(-n)
>> >
>> > []'s
>> > Joao
>> >
>> > --
>> > Esta mensagem foi verificada pelo sistema de antivírus e
>> > acredita-se estar livre de perigo.
>>
>> --
>> Esta mensagem foi verificada pelo sistema de antivírus e
>>  acredita-se estar livre de perigo.
>>
>>
>> =
>> Instruções para entrar na lista, sair da lista e usar a lista em
>> http://www.mat.puc-rio.br/~obmlistas/obm-l.html
>> =
>
>
>
> --
> Esta mensagem foi verificada pelo sistema de antivírus e
> acredita-se estar livre de perigo.

-- 
Esta mensagem foi verificada pelo sistema de antiv�rus e
 acredita-se estar livre de perigo.


=
Instru��es para entrar na lista, sair da lista e usar a lista em
http://www.mat.puc-rio.br/~obmlistas/obm-l.html
=


Re: [obm-l] Limite por l'Hospital

2014-06-22 Por tôpico Ralph Teixeira
Vamos ver o ln disso, que eh:

g(x)=x^2.ln(1+1/x)-x = x^2 (ln(1+1/x)-1/x) = (ln(1+1/x)-1/x) / (x^(-2))

Quando x->+Inf, isto aqui eh uma indet. do tipo 0/0. Note como eu
deixei o ln o mais sozinho possivel, por que agora L'Hopital vai
simplificar as coisas (se o ln ficar "misturado" com outras coisas,
ele nao some na derivada):

lim (x->+Inf) g(x) = lim (x->+Inf) ((-1/x^2)(1/(1+1/x))-1/x^2) /
(-2x^(-3)) = lim (x->+inf) (-1/2)(x/(x+1)) = -1/2

Entao, se eu nao errei conta, o limite original eh e^(-1/2).

Abraco,
   Ralph

2014-06-23 0:17 GMT-03:00 João Maldonado :
> Fala galera, tem como alguém me dar uma ajuda no seguinte limite? Faz uma
> horta que estou tentando calcular e não sai.
>
> lim (n -> inf) (1+1/n)^(n²) e^(-n)
>
> []'s
> Joao
>
> --
> Esta mensagem foi verificada pelo sistema de antivírus e
> acredita-se estar livre de perigo.

-- 
Esta mensagem foi verificada pelo sistema de antiv�rus e
 acredita-se estar livre de perigo.


=
Instru��es para entrar na lista, sair da lista e usar a lista em
http://www.mat.puc-rio.br/~obmlistas/obm-l.html
=


[obm-l] Re: [obm-l] Máximo e Contagem

2014-06-10 Por tôpico Ralph Teixeira
De novo, você vai ter que dizer explicitamente o que quer dizer por
"alternadas".

Acho que o significado mais formal de "alternada" significa UM sim, UM
não, UM sim, UM não... Mas as pessoas usam esta palavra com outros
significados -- o mais comum é "não necessariamente consecutivas".
Então HHMH teria 3 homens em posições alternadas -- é assim que você
quer?

2014-06-10 16:42 GMT-03:00 jamil silva :
> Você foi designado para organizar uma sala de reuniões de forma que haja em
> cada fileira,
>
> um máximo de quatro pessoas do mesmo sexo em cadeiras vizinhas, ou três, em
> cadeiras
>
> alternadas. As fileiras podem ter o mesmo número de pessoas, desde que a
> disposição
>
> entre os sexos seja diferente. Qual o número máximo de pessoas que você
> poderá acomodar,
>
> nestas condições ?
>
>
> --
> Esta mensagem foi verificada pelo sistema de antivírus e
> acredita-se estar livre de perigo.

-- 
Esta mensagem foi verificada pelo sistema de antiv�rus e
 acredita-se estar livre de perigo.


=
Instru��es para entrar na lista, sair da lista e usar a lista em
http://www.mat.puc-rio.br/~obmlistas/obm-l.html
=


[obm-l] Re: [obm-l] Re: [obm-l] Par ou Ímpar ? --- Contagem e combinatória

2014-06-08 Por tôpico Ralph Teixeira
Em geral, em esportes, o pessoal faz a regra com "x" consecutivas ou
"x+y" alternadas; eles costumam usar a palavra "alternadas" para
significar apenas "em qualquer ordem, nao necessariamente
consecutivas". Mas aqui fica estranho, nao? O que significa
"alternado" neste enunciado? Por exemplo, no caso aaaba voce conta que
Ana venceu 3 alternadas (digamos, 1a, 3a e 5a) ou nao (apenas dois
blocos de vitorias)?

Vou supor que voce estah pensando em "blocos de vitorias".
Consequencia: no caso aaaba, Ana preferiria PERDER a 2a, tornar isso
ababa, e assim vencer a partida!?!? Eh estranho, porque qualquer regra
que incentive alguem a fazer "gol contra" eh estranha para mim... Mas
vamos lah, vou resolver pensando assim mesmo.

a) A pior hipotese para uma partida seria aaabbbaaabbba, terminando em
13 lances. De fato, com 13 lances quaisquer, alguem tem que ganhar
pelo menos 7. Se quem ganhou essas 7 nao o fez em 3 blocos de
vitorias, entao sao apenas 2 blocos ou menos, e portanto pelo menos um
bloco terah 4 vitorias consecutivas. Ou seja, em 13, certamente vai
ter acabado.

b) Analogamente, o pior caso eh A, com 25
partidas. De fato, com 25 alguem ganhou pelo menos 13. Se nao ha 4
"blocos" (para fazer as 4 vitorias alternadas), entao sao apenas 3;
como 3x4=12<13, pelo menos um dos blocos terah pelo menos 5 partidas
consecutivas.

(Em suma, no pior caso seriam 25x13=325 lances de par ou impar. Nao
parece muito divertido :) :) :) )

c) Voce diz, as chances de vitoria de Ana versus as de Beatriz? Se
supusermos que cada **lance** tem probabilidade 50% de vitoria para
cada uma, por simetria, SIM, ambas tem a mesma chance de ganhar um
lance, uma partida ou o campeonato.

Abraco, Ralph.

2014-06-08 3:34 GMT-03:00 jamil silva :
> Sim, de zero a cinco dedos.
>
>
> Em 8 de junho de 2014 00:18, Listeiro 037 
> escreveu:
>
>>
>>
>> Cada mão de uma concorrente pode colocar de zero a cinco dedos numa
>> partida?
>>
>>
>> Em Sat, 7 Jun 2014 23:01:39 -0300
>> jamil silva  escreveu:
>>
>> > Ana e Beatriz disputam um campeonato de Par ou Ímpar,
>> > com as seguintes regras:
>> >
>> > Uma partida consiste numa série de lances com três acertos
>> > alternados, ou quatro consecutivos. O campeonato termina quando
>> > ocorrer quatro vitórias alternadas, ou cinco consecutivas.
>> >
>> > .
>> >
>> > .
>> >
>> > Responda:
>> >
>> > I) Qual o número máximo de lances de uma partida ?
>> >
>> > II) Qual o número máximo de partidas possíveis em um campeonato ?
>> >
>> > III) As chances de vitória são iguais ?
>> >
>> > .
>> > .
>> > .
>> > .
>> >
>> > Utilize as seguintes convenções:
>> >
>> > a = Lance em que Ana acerta
>> >
>> > b = Lance em que Beatriz acerta
>> >
>> > A = Partida em que a vitória é de Ana.
>> >
>> > B = Partida em que a vitória é de Beatriz.
>> >
>> > X = Campeonato de Ana
>> >
>> > Y = Campeonato de Beatriz
>> >
>> > .
>> > .
>> >
>> > Alguns exemplos de partidas:
>> >
>> >  = aaba = A
>> >
>> > bbab =  = B
>> >
>> > .
>> > .
>> > .
>> >
>> > Alguns exemplos de campeonatos:
>> >
>> > X = 
>> >
>> > Y = 
>> >
>>
>> --
>> Esta mensagem foi verificada pelo sistema de antivírus e
>>  acredita-se estar livre de perigo.
>>
>>
>> =
>> Instru�ões para entrar na lista, sair da lista e usar a lista em
>> http://www.mat.puc-rio.br/~obmlistas/obm-l.html
>> =
>
>
>
> --
> Esta mensagem foi verificada pelo sistema de antivírus e
> acredita-se estar livre de perigo.

-- 
Esta mensagem foi verificada pelo sistema de antiv�rus e
 acredita-se estar livre de perigo.


=
Instru��es para entrar na lista, sair da lista e usar a lista em
http://www.mat.puc-rio.br/~obmlistas/obm-l.html
=


Re: [obm-l] perguntinhas simples

2014-06-04 Por tôpico Ralph Teixeira
Bom, eu estou com o Bernardo: ele mostrou que a expressao vale 9x-2.
Isto nao dah um valor fixo -- depende de qual das 3 raizes voce
escolhe (e todas sao reais, e feias). Entao nao eh possivel que o
problema tenha uma resposta numerica unica A (se tivesse, teriamos
x=(A+2)/9, e isso eh impossivel pois ha 3 valores distintos de x).
Tambem voto em algum erro tipografico do livro ou prova donde veio o
problema.

Por exemplo, se fosse "dado que x^2+1/x^2=1, calcule x^3+1/x^3", a
resposta seria 0, pois (x^6+1)/x^3=((x^2+1)/x).((x^4-x^2+1)/x^2) e a
segunda expressao eh 0.

Ou aqui outra opcao: "Dado que x+1/x^2=3, calcule A=x^3+1/x^3+9/x^2".

Esse problema seria legal! Como A=(x^3-3x^2+1)(x^3+3x^2+9x+1)/x^3 +
25, a resposta seria 25.

Abraco,
   Ralph

2014-06-04 10:28 GMT-03:00 Hermann :
> É isso mesmo a resposta é zero, pelo visto é complicada paca, né!?
>
> abraços
> Hermann
> - Original Message - From: "Listeiro 037"
> 
> To: 
> Sent: Wednesday, June 04, 2014 4:17 AM
> Subject: Re: [obm-l] perguntinhas simples
>
>
>
> Errata:
>
> Corrigindo, y^6 - 3y^2 - 1 (troquei o sinal do termo de grau 1)
>
> e percebi que melhor x = y^2 + 1 é usar x = y - 1, fica
>
> (x-1)^3 - 3(x-1) - 1 = 0
>
> se for remanejar a cúbica para a outra forma, apesar do outro método
> ser exato. Mas resolver a cúbica diretamente não ajuda, não é?
>
>
> =
>
>
>
> Bem, partindo de x + 1/x^2 = 3
>
> e chegando a x^3 - 3x^2 + 1 = 0
>
> Por curiosidade, esta se assemelha à "cúbica babilônica", que eles
> resolviam com tabelas (tá lá no Boyer).
>
> e usando a substituição x = y^2 + b, se não me confundi
>
> (y^2 + b)^3 - 3(y^2 + b)^2 + 1 = 0
>
> y^6 + 3by^4 + 3b^2y^2 + b^3 - 3y^4 - 6by^2 - 3b^2 + 1 = 0
>
> y^6 + y^4(3b - 3) + 3y^2(b^2 - 2b) + b^3 - 3b^2 + 1 = 0
>
> escolhendo b = 1 o coeficiente de y^4 some e vira uma "cúbica de
> Cardano" em y^2. (caso esteja tudo ok)
>
> y^6 + y^4(3 - 3) + 3y^2(1 - 2) + 1 - 3 + 1 = 0
>
> y^6 + 3y^2(1 - 2) - 1 = 0
>
> Pode ser z = y^2 e fica
>
> z^3 + 3z - 1 = 0 (os sinais mudaram, deve ter algo simples com as
> raízes que dispense isso tudo)
>
> Prá mim melhorou em nada. A cúbica de Cardano é um pouco mais manjada.
> Mas por chegar atá aqui eu acho que deve haver uma transformação que
> recaia numa outra equação em que as raízes tenham propriedades mais
> notáveis.
>
>
>
> Em Tue, 3 Jun 2014 09:02:11 -0300
> Bernardo Freitas Paulo da Costa  escreveu:
>
>> 2014-06-02 22:20 GMT-03:00 Hermann :
>> > Na 3 vc fez outra questão a minha é Se x+(1/x)^2=3 qual o valor de
>> > x^3+(1/x)^3?
>> > não tem quadrado no primeiro x
>>
>> Bom, na força bruta: x + 1/x^2 = 3 implica que
>>
>> x^3 + 1 = 3x^2
>> 1 + 1/x^3 = 3/x
>>
>> Somando as duas igualdades, vem
>>
>> x^3 + 1/x^3 + 2 = 3x^2 + 3/x = 3x(x + 1/x^2) = 3x * 3
>>
>> Assim, x^3 + 1/x^3 = 9x - 2, e o valor depende de qual das (três)
>> raízes do polinômio você vai escolher.
>>
>> Portanto, eu acho (nessa ordem de "plausibilidade") que
>> - ou tem um quadrado no primeiro x
>> - ou não tem um quadrado no segundo x
>> - ou não era uma questão cuja resposta tem um valor numérico
>> - ou a fórmula com os cubos era mais complicada.
>>
>
>
> --
> Encryption works. Properly implemented strong crypto systems are one of
> the few things that you can rely on. Unfortunately, endpoint security
> is so terrifically weak that NSA can frequently find ways around it. —
> Edward Snowden
>
> --
> Esta mensagem foi verificada pelo sistema de antivrus e
> acredita-se estar livre de perigo.
>
>
> =
> Instrues para entrar na lista, sair da lista e usar a lista em
> http://www.mat.puc-rio.br/~obmlistas/obm-l.html
> =
>
> --
> Esta mensagem foi verificada pelo sistema de antivírus e
> acredita-se estar livre de perigo.
>
> =
> Instru�ões para entrar na lista, sair da lista e usar a lista em
> http://www.mat.puc-rio.br/~obmlistas/obm-l.html
> =

-- 
Esta mensagem foi verificada pelo sistema de antiv�rus e
 acredita-se estar livre de perigo.


=
Instru��es para entrar na lista, sair da lista e usar a lista em
http://www.mat.puc-rio.br/~obmlistas/obm-l.html
=


[obm-l] Re: [obm-l] Re: [obm-l] Re: [obm-l] Fwd: Mudança de base

2014-05-24 Por tôpico Ralph Teixeira
Acho que o problema quer as seguintes observacoes interessantes:

(sqrt(65)-1)(sqrt(65)+1)=65-1=64
e
(sqrt(65)+1)^2=66+2sqrt(65)=2(sqrt(65)+33)

Com essas duas, tudo se arruma. Vou escrever todos os logs em base 2 (e nao
vou escrever a base para ficar mais legivel). Entao:

log(sqrt(65)+33)/log(sqrt(2)/2) = log((sqrt(65)+1)^2/2) / log(2^(-1/2)) =
(2log(sqrt(65)+1) - 1 ) / (-1/2) = -4log(sqrt(65)+1) +2

Mas

log(sqrt(65)+1) = log(64/(sqrt(65)-1)) = 6-m

Entao a resposta eh 4(m-6)+2=4m-22.

Abraco, Ralph.


2014-05-24 12:54 GMT-03:00 saulo nilson :

> log(rq65+33)=x
> x^-1/2=rq65+33
> x^-1/2-34=rq65-1
> log2(x^-1/2-34)=m
> x=(2^m+34)^-2
>
>
> 2014-05-20 23:38 GMT-03:00 terence thirteen :
>
>> Acho que a melhor forma é simplesmente escrever $log_a(b)=ln(b)/ln(a)$.
>> Isso vai te ajudar a ver o que calcular, afinal.
>>
>>
>> Em 18 de maio de 2014 13:33, Marcelo de Moura Costa 
>> escreveu:
>>
>>
>>> Alguém poderia me ajudar nesta?
>>>
>>> Sabe-se que:
>>>
>>> [image: \log_{2}{\left( \sqrt{65}-1 \right)} = m]
>>>
>>> Determine em função de m o valor de:
>>>
>>> [image: \log_{\frac{\sqrt{2}}{2}}{\left(\sqrt{65}+33\right)}]
>>>
>>> Que é uma mudança de base parece óbvio, mas o numerador é que está sendo
>>> o problema, aguardo um retorno, grato.
>>>
>>>
>>>
>>>
>>>
>>> --
>>> Esta mensagem foi verificada pelo sistema de antivírus e
>>> acredita-se estar livre de perigo.
>>>
>>
>>
>>
>> --
>> /**/
>> 神が祝福
>>
>> Torres
>>
>> --
>> Esta mensagem foi verificada pelo sistema de antivírus e
>> acredita-se estar livre de perigo.
>>
>
>
> --
> Esta mensagem foi verificada pelo sistema de antivírus e
> acredita-se estar livre de perigo.

-- 
Esta mensagem foi verificada pelo sistema de antiv�rus e
 acredita-se estar livre de perigo.



[obm-l] Re: [obm-l] Re: [obm-l] Re: [obm-l] Diferença de Quadrados e Equações Diofantinas

2014-05-17 Por tôpico Ralph Teixeira
Ah, racionais... Ok, então, como você disse, o conjunto dos números da
forma n/2 (onde n é inteiro) serve, pois ((k+1)/2)^2-((k-1)/2)^2=k para
todo k. Mas nao sei se ele eh minimal...
On May 17, 2014 4:59 AM, "jamil silva"  wrote:

> Saudações, Ralph !
>
>
> O que quero é um conjunto no qual, além dos inteiros ímpares e inteiros
> pares da
> forma 4n, haja solução também para k = 4n-2.
>
> Por exemplo: p² - q² = 2 não tem solução nos inteiros, mas tem solução nos
> racionais
>
> p = 3/2 e q = 1/2  ou p = 17/12 e q = 1/12   etc.
>
> Considere, então que o que peço é um subconjunto dos Racionais no qual a
> equação
>
> p² - q² = k tenha pelo menos uma solução para todo k Inteiro positivo
>
>
>
>
>
>
> Em 16 de maio de 2014 19:40, Ralph Teixeira  escreveu:
>
>> Depende do que significa "menor"...
>>
>> Por exemplo, considere A={1,2,4,8,13,21,31,45,..}. Este conjunto foi
>> montado assim a partir do terceiro elemento: calcule z_n=menor inteiro
>> positivo que não eh da forma a_i-a_j com i,j> B o conjunto das raízes quadradas dos elementos de A.
>>
>> Eu afirmo que, para todo k inteiro, existem p e q em B com p^2-q^2=k
>> (pois existem x e y em A com x- y=k). Por outro lado, o único jeito de
>> obter z_n como diferença de termos de A é tomando a_(n+1)-a_n, então nenhum
>> subconjunto de B tem a propriedade...
>>
>> Diz-se que B é MINIMAL com relação a esta propriedade... É algo assim que
>> você procura?
>>
>> Abraço, Ralph.
>> On May 16, 2014 2:37 PM, "jamil silva"  wrote:
>>
>>> Qual o menor conjunto ao qual devam pertencer p e q a fim de que
>>> p² - q² = k sempre tenha uma solução não vazia para todo k perten-
>>> -cente aos Inteiros ?
>>>
>>> --
>>> Esta mensagem foi verificada pelo sistema de antivirus e
>>> acredita-se estar livre de perigo.
>>
>>
>> --
>> Esta mensagem foi verificada pelo sistema de antivirus e
>> acredita-se estar livre de perigo.
>
>
>
> --
> Esta mensagem foi verificada pelo sistema de antivírus e
> acredita-se estar livre de perigo.

-- 
Esta mensagem foi verificada pelo sistema de antiv�rus e
 acredita-se estar livre de perigo.



[obm-l] Re: [obm-l] Diferença de Quadrados e Equações Diofantinas

2014-05-16 Por tôpico Ralph Teixeira
Depende do que significa "menor"...

Por exemplo, considere A={1,2,4,8,13,21,31,45,..}. Este conjunto foi
montado assim a partir do terceiro elemento: calcule z_n=menor inteiro
positivo que não eh da forma a_i-a_j com i,j wrote:

> Qual o menor conjunto ao qual devam pertencer p e q a fim de que
> p² - q² = k sempre tenha uma solução não vazia para todo k perten-
> -cente aos Inteiros ?
>
> --
> Esta mensagem foi verificada pelo sistema de antivírus e
> acredita-se estar livre de perigo.

-- 
Esta mensagem foi verificada pelo sistema de antiv�rus e
 acredita-se estar livre de perigo.



[obm-l] Re: [obm-l] ajude-me a comprovar... ou não

2014-05-05 Por tôpico Ralph Teixeira
Hmm, eles perguntam o gráfico de f? Então eu concordo com o gabarito
oficial: o GRÁFICO de f(x,y) corresponde à superfície z=-sqrt(16-x^2-y^2),
que é um subconjunto de z^2=16-x^2-y^2, ou seja x^2+y^2+z^2=16, uma
superfície esférica de centro na origem (0,0,0) e raio 4. Mas não é a
superfície toda não, pois note que z é sempre negativo na equação original.
Então é de fato a superfície inferior do hemisfério de baixo desta esfera.

("Calota esférica" é uma boa maneira de dizer que é só a "casca", não
incluindo o sóldio todo. Mas é o hemisfério sul desta calota, por assim
dizer)

Metade inferior de um círculo seria o gráfico de g(x)=-sqrt(16-x^2), uma
função de uma variável apenas. Como ali tem duas variáveis x e y NO
DOMÍNIO, o gráfico tem que morar em R^3 (x, y e uma terceira variável que
chamei de z), não no plano R^2.


2014-05-05 16:07 GMT-03:00 Fabio Silva :

> Olá caríssimos,
>
> Fiz uma prova e havia uma questão em que:
> "Seja a função f(x,y) = - sqrt(16 - x² - y²) é possível afirmar que:"
>
> Pois bem, a resposta do gabarito dizia que: "Trata-se da CALOTA INFERIOR
> DE UMA ESFERA com centro na origem e diâmetro 4."
>
> Minha resposta é que "Trata-se da METADE INFERIOR DE UM CÍRCULO com centro
> na origem e diâmetro 4."
>
> Se a minha estiver correta, preciso de mais alguma demonstração para abrir
> um recurso.
>
> Obrigado
>
> Fabio MS
>
> --
> Esta mensagem foi verificada pelo sistema de antivírus e
> acredita-se estar livre de perigo.
>

-- 
Esta mensagem foi verificada pelo sistema de antiv�rus e
 acredita-se estar livre de perigo.



Re: [obm-l] resultados racionais.

2014-05-03 Por tôpico Ralph Teixeira
Teoreminha: Se x=k.pi com k racional e 2cos(x) eh racional, entao 2cos(x)
eh inteiro.

(Consequencia: os unicos valores do tipo que voce falou sao -1, -1/2, 0,
1/2 e 1. Tah, voce falou do sinx, mas como cos(x)=sin(pi/2-x), dah no
mesmo.)

Demonstracao: Suponha 2cosx=a/b com a e b inteiros primos entre si e |b|>1.
Entao 2cos(2x)=4(cosx)^2-2=(a^2-2b^2)/b^2 tambem eh racional. Alias, esta
fracao tambem eh irredutivel, pois a e b sao primos entre si, e nao eh
zero, pois raiz(2) eh irracional.

Entao a sequencia 2cosx, 2cos2x, 2cos4x, 2cos8x, etc. terah infinitos
numeros racionais, todos distintos (olha aquele denominador elevado ao
quadrado, portanto aumentando, cada vez que eu dobro o angulo!). Mas isso
eh absurdo, pois se x=pi.p/q, todos aqueles angulos sao da forma pi.n/q com
n e q inteiros, e (exceto congruencia) soh ha 2q angulos deste tipo, a
saber, {0,pi/q,2pi/q,3pi/q,...(2q-1)pi/q)}.

Abraco, Ralph.


2014-05-03 15:56 GMT-03:00 Douglas Oliveira de Lima <
profdouglaso.del...@gmail.com>:

> Ola amigos , estive pensando em sala de aula recentemente, em relação aos
> ângulos do primeiro quadrante e o valores de seus senos, como por exemplo,
> o sen(pi/6)=1/2,
> ai me veio a seguinte duvida, se existem outros ângulos da forma k(pi) com
> k racional que possuem senos racionais bonitinhos entre zero e um, sem ser
> o de 30 graus, e se não como poderíamos provar isso.
> Obs. Ainda nao tive tempo de pensar, mas gostaria de compartilhar a ideia
> com o grupo.
>
> Abs do Douglas Oliveira.
>
> --
> Esta mensagem foi verificada pelo sistema de antivírus e
> acredita-se estar livre de perigo.

-- 
Esta mensagem foi verificada pelo sistema de antiv�rus e
 acredita-se estar livre de perigo.



Re: [obm-l] Subtrair x de y

2014-05-03 Por tôpico Ralph Teixeira
Para mim, "subtrair" = "tirar". Entao, tirar x de y eh y-x, que nem voce
disse.

Abraco, Ralph.


2014-05-03 12:58 GMT-03:00 Ennius Lima :

> Caros Colegas,
>
> Dados os números reais x e y, parece-me que "subtrair x de y" significa
> obter "y - x".
> Já vi, entretanto, quem diga que significa "x - y ".
> Gostaria de saber o que vocês pensam a respeito.
> Desde já, muito obrigado.
> Um abraço do Ennius!
> __
>
> --
> Esta mensagem foi verificada pelo sistema de antivírus e
>  acredita-se estar livre de perigo.
>
>

-- 
Esta mensagem foi verificada pelo sistema de antiv�rus e
 acredita-se estar livre de perigo.



[obm-l] Re: [obm-l] Re: [obm-l] Re: [obm-l] Matemágica - Séries

2014-04-28 Por tôpico Ralph Teixeira
Chegar a resposta certa eh facil. No primeiro video, ela chega a resposta
S=0, depois chega a resposta S=1, e enfim chega a resposta 1/2. Fica a
impressao que a resposta final que ele achou eh a melhor, simplesmente
porque ali que ele escolheu parar. Ele poderia, usando metodos parecidos,
chegar em outros numeros. Por exemplo, ele podia dizer que 2S=1-S usando o
fato de que S=0, entao S=2S=0. Entao ele tiraria S=1/3, que nao faz sentido.

Note-se: existem argumentos em favor de dizer que aquela soma vale 1/2, e
que a outra do outro video vale -1/12 -- mas, que eu saiba, nao sao esses
"simples".


2014-04-28 19:20 GMT-03:00 luiz silva :

> Pois é,
>
> Mas a minha pergunta é : se os axiomas não se aplicam, pq quando usados,
> chegam à mesma resposta que os outros métodos "corretos" (o metodo que vc
> colocou, o uso da função zeta, para o caso dos numeros naturais, etc..)?
>
> Abs
> Felipe
>   Em Segunda-feira, 28 de Abril de 2014 17:36, Ralph Teixeira <
> ralp...@gmail.com> escreveu:
>  Só tem um problema -- os axiomas básicos da aritmética dos números
> naturais NÃO se aplicam a somas infinitas. Na definição mais básica de soma
> infinita, a soma 1-1+1-1+1... simplesmente não existe (a série diverge). A
> soma infinita não é associativa, então o que eles fizeram não está nem um
> pouco certo.
>
> Note-se que HÁ maneiras de REDEFINIR somas infinitas que dão a resposta
> 1/2 (somar "a Cesaro", por exemplo, vide
> http://en.wikipedia.org/wiki/Ces%C3%A0ro_summation)
>
> (Só para ilustrar -- um dos "axiomas básicos" afirma que a soma de números
> positivos dá positivo, o que não é o caso ali.)
>
> Não é que o que está sendo dito ali está **errado**, mas eu
> particularmente não gosto do jeito que eles apresentam as coisas, fazendo
> tudo parecer fácil e simples quando não é. No fundo no fundo, eles sabem
> que estão provocando (porque os comentários do Youtube são a fonte natural
> de comentários lógicos Viu a ironia?).
>
>
>
> 2014-04-28 16:57 GMT-03:00 luiz silva :
>
> Prezados,
>
> Não conhecia esses resultados, mas achei surpreendente e tenho algumas
> questões :
>
> 1) http://www.youtube.com/watch?v=PCu_BNNI5x4
>
> 2) http://www.youtube.com/watch?v=w-I6XTVZXww
>
> 3) http://www.youtube.com/watch?v=0Oazb7IWzbA
>
>
> Sem "nada adicional" (análise complexa), apenas aplicando os axiomas
> básicos da aritmética aos números naturais em uma série infinita, chegamos
> ao resultado correto, sem ter que recorrer a análise complexa.
>
> Porque chega-se ao resultado correto, com estas manipulações básicas, na
> análise real? O que isso significa? Qual o significado ontológico disso
> tudo, quando aplicada a física? Isso por ter algo a ver com Godel ?
>
> Abs
> Felipe
>
>
>
>
> --
> Esta mensagem foi verificada pelo sistema de antivírus e
> acredita-se estar livre de perigo.
>
>
>
> --
> Esta mensagem foi verificada pelo sistema de antiv�us e
> acredita-se estar livre de perigo.
>
>
>
> --
> Esta mensagem foi verificada pelo sistema de antivírus e
> acredita-se estar livre de perigo.
>

-- 
Esta mensagem foi verificada pelo sistema de antiv�rus e
 acredita-se estar livre de perigo.



[obm-l] Re: [obm-l] Matemágica - Séries

2014-04-28 Por tôpico Ralph Teixeira
Só tem um problema -- os axiomas básicos da aritmética dos números naturais
NÃO se aplicam a somas infinitas. Na definição mais básica de soma
infinita, a soma 1-1+1-1+1... simplesmente não existe (a série diverge). A
soma infinita não é associativa, então o que eles fizeram não está nem um
pouco certo.

Note-se que HÁ maneiras de REDEFINIR somas infinitas que dão a resposta 1/2
(somar "a Cesaro", por exemplo, vide
http://en.wikipedia.org/wiki/Ces%C3%A0ro_summation)

(Só para ilustrar -- um dos "axiomas básicos" afirma que a soma de números
positivos dá positivo, o que não é o caso ali.)

Não é que o que está sendo dito ali está **errado**, mas eu particularmente
não gosto do jeito que eles apresentam as coisas, fazendo tudo parecer
fácil e simples quando não é. No fundo no fundo, eles sabem que estão
provocando (porque os comentários do Youtube são a fonte natural de
comentários lógicos Viu a ironia?).



2014-04-28 16:57 GMT-03:00 luiz silva :

> Prezados,
>
> Não conhecia esses resultados, mas achei surpreendente e tenho algumas
> questões :
>
> 1) http://www.youtube.com/watch?v=PCu_BNNI5x4
>
> 2) http://www.youtube.com/watch?v=w-I6XTVZXww
>
> 3) http://www.youtube.com/watch?v=0Oazb7IWzbA
>
>
> Sem "nada adicional" (análise complexa), apenas aplicando os axiomas
> básicos da aritmética aos números naturais em uma série infinita, chegamos
> ao resultado correto, sem ter que recorrer a análise complexa.
>
> Porque chega-se ao resultado correto, com estas manipulações básicas, na
> análise real? O que isso significa? Qual o significado ontológico disso
> tudo, quando aplicada a física? Isso por ter algo a ver com Godel ?
>
> Abs
> Felipe
>
>
>
>
> --
> Esta mensagem foi verificada pelo sistema de antivírus e
> acredita-se estar livre de perigo.

-- 
Esta mensagem foi verificada pelo sistema de antiv�rus e
 acredita-se estar livre de perigo.



[obm-l] Re: [obm-l] Re: [obm-l] Implicação lógica (linguagem)

2014-04-23 Por tôpico Ralph Teixeira
Na minha interpretação ambas as frases têm o mesmo significado...

Agora, vale notar: assim, sem contexto, nenhuma delas afirma que "x=1 é
verdadeiro".

Abraço,
   Ralph


2014-04-23 16:06 GMT-03:00 terence thirteen :

> Não, não está errado. É supérfluo, mas não errôneo.
>
>
> Em 21 de abril de 2014 10:33, Pedro Chaves escreveu:
>
>> Caros colegas da Lista,
>>
>> Constitui erro escrever "x = 1 é verdadeiro => x + 1 = 2 é verdadeiro"?
>>
>> Penso que, pelo significado de implicação lógica, bastaria escrever:
>>
>> "x = 1 => x + 1 = 2" .
>>
>> Abraços do Pedro Chaves!
>>
>> ___
>> --
>> Esta mensagem foi verificada pelo sistema de antivírus e
>>  acredita-se estar livre de perigo.
>>
>>
>> =
>> Instruções para entrar na lista, sair da lista e usar a lista em
>> http://www.mat.puc-rio.br/~obmlistas/obm-l.html
>> =
>>
>
>
>
> --
> /**/
> 神が祝福
>
> Torres
>
> --
> Esta mensagem foi verificada pelo sistema de antivírus e
> acredita-se estar livre de perigo.

-- 
Esta mensagem foi verificada pelo sistema de antiv�rus e
 acredita-se estar livre de perigo.



[obm-l] Re: [obm-l] Minimizar a distância

2014-04-22 Por tôpico Ralph Teixeira
Acho que o enunciado podia dizer se as cidades estao "em linha reta" (isto
eh, na mesma estrada), no plano, no espaco sideral...

Mas acho que o que se quer eh o seguinte: sejam A, B e C as cidades com
100, 200 e 300 estudantes respectivamente (que vamos fingir que sao pontos
do plano). Supondo que nao haja ruas (e que sejam pontos distintos), eu
afirmo que o lugar correto para colocar a escola eh em C mesmo. (Ou, para
ser chato, em qualquer ponto na intersecao dos segmentos AC e BC --
"costuma" ser soh C mesmo)

De fato, a distancia total percorria por todos os alunos serah
100.d(A,C)+200.d(B,C)+300.0. Dado um outro ponto P qualquer do plano (ou do
espaco sideral, o que vem a seguir vale em qualquer espaco metrico com
desigualdade triangular!), temos d(A,C)<=d(A,P)+d(P,C) e
d(B,C)<=d(B,P)+d(P,C). Entao

100.d(A,C)+200.d(B,C) <= 100 d(A,P) + 200.d(B,P) + 300.d(P,C)

Ou seja, o ponto P eh pior. Entao construa em C logo. Bacaninha esse
problema!

(Para ser chato: P empata com C, se estiver em ambos os segmentos AC e BC.
Entao temos dois casos (i) C eh a unica solucao, ou (ii) A,B,C sao
colineares, com A e B de um lado, P no meio e C do outro. Nesse caso (ii),
qualquer ponto que esteja na parte comum dos segmentos AC e BC vale.)

Abraco,
  Ralph


2014-04-20 18:06 GMT-03:00 marcone augusto araújo borges <
marconeborge...@hotmail.com>:

> Três cidadezinhas têm 100,200 e 300 estudantes,respectivamente.Onde se
> deve construir
> uma escola para minimizar a distância total percorrida pelos estudantes
> todos os dias?
>
> --
> Esta mensagem foi verificada pelo sistema de antivírus e
> acredita-se estar livre de perigo.
>

-- 
Esta mensagem foi verificada pelo sistema de antiv�rus e
 acredita-se estar livre de perigo.



Re: [obm-l] Produto de inteiros consecutivos

2014-04-16 Por tôpico Ralph Teixeira
Para n=1, eh obvio.

Vamos ao passo de inducao: suponha que, para qualquer k inteiro,
A=k(k+1)...(k+n-3)(k+n-2) eh multiplo de (n-1)!.

Agora considere B=m(m+1)(m+2)(m+n-1). Como sao n numeros consecutivos,
um deles tem que ser multiplo de n, digamos, x=m+a. Escrevendo em funcao de
x, fica:

B=(x-a)(x-a+1)...(x-1)x(x+1)...(x+n-a-2)(x+n-a-1)

Argumento inicial: se x for o ultimo fator, acabou, pois seriam n-1 numeros
consecutivos (cujo produto eh divisivel por (n-1)!), vezes x, que eh
divisivel por n.

Senao, procederemos a uma expansao escolhida a dedo com relacao ao ultimo
termo.

No primeiro passo, pense (x+n-a-1)=n+(x-a-1) e distribua -- note que eu
escrevi o x-a-1 no INICIO do segundo produto abaixo:
B=(x-a)(x-a+1)...(x+n-a-2).n + (x-a-1)(x-a)(x-a+1)(x+n-a-2)

Fazendo k=(x-a), vemos que o primeiro termo eh divisivel por (n-1)!.n=n!;
vamos abrir o segundo termo pelo mesmo truque, (x+n-a-2)=n+(x-a-2).
B= c1.n! + (x-a-1)(x-a)(x-a+1)(x+n-a-3).n +
(x-a-2)(x-a-1)(x-a)(x+n-a-3)

Agora tome k=(x-a-1) e veja que a segunda parcela da soma eh divisivel por
n!, entao abra o terceiro via (x+n-a-3)=n+(x-a-3):
B = c2.n! + (x-a-2)(x+n-a-4).n + (x-a-3)(x-a-2)(x-a-1)...(x+n-a-4) = ...

e assim por diante. A ideia eh que a cada passo voce expande usando o n do
ultimo termo para gerar uma coisa que tem n!, e "translada" o ultimo fator
do produto para a primeira posicao. Bom, vah fazendo isso ateh que a
parcela que resta da soma seja (x-n+1)(x-n+2)...(x-1)x. Agora, como x eh o
ultimo fator, acabou pelo argumento inicial.

---///---

Essa solucao PARECE fazer inducao soh em n, mas no fundo no fundo eu estou
disfarcadamente fazendo uma inducao em n e k ao mesmo tempo, que nem nas
outras solucoes propostas... :(

Abraco, Ralph.


2014-04-16 19:53 GMT-03:00 Ennius Lima :

> Caros Colegas,
>
> Pode-se demonstrar por indução sobre n (somente sobre n) que o produto de
> n inteiros consecutivos quaisquer é múltiplo do fatorial de n?
> Até agora não consegui nenhuma demonstração assim.
>
> Agradeço-lhes a habitual gentileza.
> Abraços do Ennius Lima.
> _
>
>
> --
> Esta mensagem foi verificada pelo sistema de antivírus e
>  acredita-se estar livre de perigo.
>
>

-- 
Esta mensagem foi verificada pelo sistema de antiv�rus e
 acredita-se estar livre de perigo.



[obm-l] Re: [obm-l] Re: [obm-l] Re: [obm-l] Re: [obm-l] Re: [obm-l] Combinatória

2014-03-18 Por tôpico Ralph Teixeira
Acho sim que esta maneira tem dupla contagem Vou chamar os homens de
xyzt e as mulheres de EFGH.

Entao, voce pode escolher aquela mulher como E, ordenar os outros 7 como
xFyGzHt, e depois inserir a mulher E antes de F de forma a gerar xEFyGzHt,
por exemplo.

Ou voce pode escolher F, ordenar xEyGzHt, e inserir F apos E, ficando com
xEFyGzHt.

Aquele metodo conta ambas as configuracoes, mas note que eh a mesma! E,
naturalmente, isso acontece com varias escolhas...

Para consertar, voce pode decidir que aquela mulher separada pode ser
colocada nas pontas (2 maneiras), ou, se junto de alguma mulher, tem que
ser ANTES da outra (3 maneiras). Entao sao 5 maneiras de colocar a mulher
extra. Agora nao ha dupla contagem: 5.4!.4!.

Abraco,
   Ralph


2014-03-18 9:45 GMT-03:00 Fabio Silva :

> Olá amigos,
>
> Ainda insisto. Pensemos nas oito possibilidades de escolher um lugar para
> aquela mulher. Após isto, devemos pensar em escolher quantas possibilidades
> de mulheres posso colocar na primeira posição posição, na segunda e assim
> sucessivamente. O que daria um total de 4!. O mesmo pensamento seria para
> os homens, sendo igual a 4!.
> Daí, não vi contagem dobrada. E o resultado seria apenas o produto mesmo:
> 8.4!.4!=4608 possibilidades.
>
> Onde estaria a contagem em dobro?
>
> Um abraço
>
> Fabio MS
>
>
>   On Monday, March 17, 2014 10:52 PM, Walter Tadeu Nogueira da Silveira <
> wtade...@gmail.com> wrote:
>  Obrigado a todos. E, sim, Leo, foi engano. Seria C(5,4) formas de
> escolher a posição dos homens.
>
> Abs
>
>
> Em 17 de março de 2014 21:06, Pacini Bores escreveu:
>
> Olá,
> Nas soluções do Kleber e do Fabio, devemos retirar 3.4!.4! ; pois como o
> Leonardo falou, entre os homens os 3.4!.4! foram contado duas vezes.
>
> Abraços
>
> Pacini
>
>
> Em 17 de março de 2014 20:35, Leonardo Maia  escreveu:
>
> Vejo a razão com o Walter (apesar de um typo), e não com o Kleber.
>
> Enxergo "dupla contagem" na solução do Kleber. Notem os dois espaços ao
> redor da 1a. mulher entre as 3 já alocadas, por exemplo. Quando se contam
> as possíveis posições da 4a. mulher, essas duas posições já são
> consideradas entre as 8 possibilidades, correspondendo aos dois possíveis
> ordenamentos de duas mulheres que eventualmente fiquem juntas ali. Depois,
>  DE NOVO esses dois possíveis ordenamentos são contados no 4! das
> mulheres. Overcounting!
>
> Na solução do Walter, os dois fatores 4! estão corretos e devem ser
> multiplicados pelo número de possíveis "entrelaçamentos" das filas de
> homens e mulheres, que é dado pelo número de soluções da equação x1 + x2 +
> x3 + x4 + x5 = 4 onde cada variável só pode valer 0 ou 1 (cada variável
> corresponde ao número de homens na posição de cada espaço _ na solução do
> Walter). São 5, e não C(5,2), tais soluções. O Walter deve ter pensado uma
> coisa e escrito outra, pois o 2880 que julgo correto resulta do 5.
>
> Saudações,
> Leo.
>
>
> On Monday, March 17, 2014, Kleber Bastos  wrote:
>
> Pensei aqui o problema de uma forma diferente:
> Como os homens não podem ficar juntos, temos que ter pelo menos uma mulher
> entre dois homens. Então vamos colocar os 4 homens em fila, sempre com uma
> mulher enrte 2:
> H M H M H M H
>  Para isso precisamos usar 3 mulheres. Isso é o mínimo que temos que ter.
> Mas ainda temos uma mulher para colocar na fila em qualquer lugar. Os
> lugares possíveis para essa última mulher são 8, onde vou colocar os traços:
> _ H _ M _ H _ M _ H _ M _ H _
>  Então temos 8 maneiras diferentes de colocar a última mulher. Além
> disso, podemos trocar os homens de lugar entre si (que pode ser feito de P
> 4 = 4! maneiras) e as mulheres de lugar enter si (que pode ser feito de P4 =
> 4! maneiras).
> Portanto teremos:
> = 8 . 4! . 4!
> = 8 . 24 . 24= 4608
>
> Abraços, Kleber.
> Sent from my iPad
>
> On 17/03/2014, at 19:06, Walter Tadeu Nogueira da Silveira <
> wtade...@gmail.com> wrote:
>
> Amigos,
>
> Na questão: "De quantas maneiras podemos dispor 4 homens e 4 mulheres em
> uma fila, sem que dois homens fiquem juntos?"
>
> Pensei em "amarrar" as mulheres e escolher posições onde os homens
> poderiam ocupar sem ficar dois juntos. Depois permutar homens e mulheres.
>
> _ M _ M _ M _ M _
>
> C(5,2). P4. P4 = 2880 formas diferentes.
>
> O gabarito da questão diz 4608. Mas não concordei com essa resposta.
>
> Alguém poderia ajudar. Muito obrigado.
> --
> Walter Tadeu Nogueira da Silveira
>
>
> --
> Esta mensagem foi verificada pelo sistema de antivírus e
> acredita-se estar livre de perigo.
>
>
> --
> Esta mensagem foi verificada pelo sistema de antivírus e
> acredita-se estar livre de perigo.
>
>
> --
> Esta mensagem foi verificada pelo sistema de antivírus e
> acredita-se estar livre de perigo.
>
>
>
> --
> Esta mensagem foi verificada pelo sistema de antivírus e
> acredita-se estar livre de perigo.
>
>
>
>
> --
> Walter Tadeu Nogueira da Silveira
> http://www.professorwaltertadeu.mat.br
>
>
> --
> Esta mensagem foi verificada pelo sistema de antivírus e
> acre

[obm-l] Re: [obm-l] RE: [obm-l] Re: [obm-l] Número de soluções naturais

2014-03-16 Por tôpico Ralph Teixeira
Isso mostra que sao 201 opcoes para z -- mas cada valor de z tem VARIAS
solucoes em x e y, como voce mesmo mostrou.

Mas dah para continuar o seu raciocinio e matar a questao: voce mostrou
que, dado um z especifico, as solucoes sao da forma y=t e x=1000-5z-2t.
Note que aqui t varia entre 0 e (500-2.5z). Ou seja:

-- Se z=0, ha 501 opcoes para t (de 0 a 500)
-- Se z=1, ha 498 opcoes para t (de 0 a 497)
-- Se z=2, ha 496 opcoes para t (de 0 a 495);
-- Se z=3, ha 493 opcoes para t (de 0 a 492);
...
-- Se z=200, ha 1 opcao para t (de 0 a 0).

Entao o numero de solucoes eh 501+498+496+493++1. Calculando isso, o
problema sai.

Abraco,
Ralph


2014-03-16 10:02 GMT-03:00 Ennius Lima :

> A resolução que enviei através do Pedro Chaves, pois meu e-mail estava
> tendo problemas, parece que está equivocada.
> Pode algum colega me ajudar?
> Grato.
> Ennius Lima
> 
>
>
>
>
>
>
> De: brped...@hotmail.com
> Enviada: Domingo, 16 de Março de 2014 01:54
> Para: obm-l@mat.puc-rio.br
> Assunto: [obm-l] RE: [obm-l] Re: [obm-l] Número de soluções naturais
>
> Bem... acho que são 201 soluções naturais.
> Resolução:
>
> x + 2y = 1000 - 5z
>
> Fixado z, temos uma equação diofantina com duas variáveis.
>
> Uma solução particular:   x = 1000 - 5z   e  y = 0
>
> Solução geral:   x = 1000 - 5z - 2t   (t é inteiro)
>  y= t
>
> Atribuindo-se a z qualquer valor de 0 a 200, pode-se sempre encontrar um t
> no intervalo [0, 500], tal que x esteja no intervalo [0, 1000].
> Portanto, são 201 soluções naturais.
>
> Peço comentários dos colegas.
> Abraços do Ennius!
> __
>
>
>
>
> _
> > Date: Fri, 14 Mar 2014 15:40:13 -0300
> > Subject: [obm-l] Re: [obm-l] Número de soluções naturais
> > From: peterdirich...@gmail.com
> > To: obm-l@mat.puc-rio.br
> >
> > Acho que uma boníssima pedida seria Séries Formais!
> >
> > Vamos tentar achar a série formal cujos expoentes são da forma A+2B+3C,
> > A,B,C>= 0.
> >
> > Acho que uma manipulação algébrica é moleza, algo como
> >
> > 1/((1-x)^3(1+x)(1+x+x^2))
> >
> >
> >
> >
> >
> > Em 5 de março de 2014 20:22, Ennius Lima
> > mailto:enn...@bol.com.br>> escreveu:
> > Caros Colegas,
> >
> > Quantas soluções naturais tem a equação diofantina x + 2y + 5z = 1000?
> >
> > (Incluo o zero entre os números naturais)
> >
> >
> > Desde já, agradeço-lhes a atenção.
> >
> > Ennius Lima
> > __-
> >
> >
> > --
> > Esta mensagem foi verificada pelo sistema de antivírus e
> > acredita-se estar livre de perigo.
> >
> >
> >
> >
> > --
> > /**/
> > 神が祝福
> >
> > Torres
> >
> > --
> > Esta mensagem foi verificada pelo sistema de antiv�rus e
> > acredita-se estar livre de perigo.
> --
> Esta mensagem foi verificada pelo sistema de antiv�rus e
>  acredita-se estar livre de perigo.
>
>
> =
> Instru��es para entrar na lista, sair da lista e usar a lista em
> http://www.mat.puc-rio.br/~obmlistas/obm-l.html
> =
>
> --
> Esta mensagem foi verificada pelo sistema de antivírus e
>  acredita-se estar livre de perigo.
>
>

-- 
Esta mensagem foi verificada pelo sistema de antiv�rus e
 acredita-se estar livre de perigo.



[obm-l] Re: [obm-l] Re: [obm-l] Re: [obm-l] Preciso de uma ajuda para a solução de um problema.

2014-03-12 Por tôpico Ralph Teixeira
Desculpa, eu tive que enviar a mensagem antes de terminar... Ainda há
problemas no que eu falei: tem um n^2/2 naquela expressão do x! Então:

i) Se n for par (n=2k), n^2/2 é inteiro, então n^2-500=4k^2-500 tem que ser
divisor de 125000, isto é k^2-125 é divisor de 31250=2.5^6. Os divisores
são 1, 5, 125, 625, 3125, 15625, seus dobros e os negativos possíveis -1,
-2, -5, -10 e -125 (os outros são negativos demais). Veja quais deles
somados com 125 dão quadrados perfeitos e você acha algumas poucas
possibilidades Estou vendo como possibilidades n=0 huh... tem mais
alguma?

ii) Se n for ímpar, o que eu escrevi estava errado! Agora, 125000/(n^2-500)
tem que ser do tipo "m/2" com m ímpar. Mas não vai dar: n^2-500 é ímpar,
então o numerador de 125000/(n^2-500) é par, então não há "1/2" para juntar
com o do n^2/2.

Deve ter um jeito melhor de fazer o caso "x e n ambos inteiros", mas fica
para outra. ;)

Abraço,
 Ralph


2014-03-12 16:41 GMT-03:00 :

>  Valeu demais Ralph Teixeira.
>
> Em 12.03.2014 16:18, Ralph Teixeira escreveu:
>
> Eu entendi que x não é necessariamente inteiro, mas a expressão tem que
> ser inteira (aliás, um inteiro positivo-ou-nulo, pois é raiz de um troço).
> Então escrevi algo assim
>
> (x^2+1000x)^(1/2)-x = n^2   (onde n pode ser a princípio 0,1,2,3,...)
> então
> (x^2+1000x) = (x+n^2)^2
> então
> 1000x=2xn^2+n^4
> então
> x = n^4 / (1000-2n^2)
>
> Bom, mas não é bem isso não, pois vê-se que x+n^2 tinha que ser >= 0... O
> caso n=0 dá x=0 que não ajuda nem atrapalha a soma, então vou supor logo
> que n é positivo e então:
>
> x+n^2 >=0
> sse
> n^4 / (1000-2n^2) + n^2 >=0
> sse
> n^2 (1000 - n^2) / (1000-2n^2) >=0
>
> Esta desigualdade implica em n n>raiz(1000)=10raiz(10)=31.6 Em suma, n só pode assumir os seguintes
> valores:
> {0,1,2,..., 22} ou {32,33,34,...}
>
> Agora, enfim, com estas condições, tanto x+n^2 quando n são positivos,
> então a equação original
> ((x^2+1000x)^(1/2)-x)^(1/2)=n
> é realmente equivalente a
> x = n^4 / (1000-2n^2)
> o que pode ser confirmando revertendo os passos da dedução original lá em
> cima (os "então" viram "sse").
>
> O que você quer agora é achar a soma:
>
> SOMATÓRIO (n^4 / (1000-2n^2)) onde n varia de 0 a 22, e depois de 32 a
> Infinito.
>
> Mas essa soma diverge, pois o termo geral não vai para zero!
>
> ---///---
>
> Se o enunciado limitar x a ser POSITIVO, então devemos ter 2n^2<1000, isto
> é, ficam apenas os números de 0 a 22. Então a resposta seria:
>
> SOMATÓRIO (n=0 a 22) (n^4 / (1000-2n^2))
>
> Eu tinha esperança de abrir em frações parciais e achar uma soma
> telescópica, mas
>
> n^4 / (1000-2n^2) = -n^2/2 - 250 -125000 / (n^2-500)
>
> Os dois primeiros termos são fáceis de somar de 0 a 22, mas o último não
> fica telescópico não!
>
> ---///---
>
> Enfim, se x tiver que ser INTEIRO, então n^2-500 tem que ser divisor de
> 125000. Argh, sai, mas é horrendo.
>
> Abraço,
>  Ralph
>
>
>
>
>
> 2014-03-12 12:53 GMT-03:00 Bernardo Freitas Paulo da Costa <
> bernardo...@gmail.com>:
>
>> 2014-03-11 23:11 GMT-03:00  :
>> > Olá , boa noite a todos os amigos da lista, recebi recentemente um
>> problema
>> > abaixo.
>> >
>> > Determinar a soma de todos os valores de x tais que
>> > ((x^2+1000x)^(1/2)-x)^(1/2) seja inteiro.
>>
>> x é inteiro?
>>
>> --
>> Bernardo Freitas Paulo da Costa
>>
>> --
>> Esta mensagem foi verificada pelo sistema de antivírus e
>>  acredita-se estar livre de perigo.
>>
>>
>> =
>> Instruções para entrar na lista, sair da lista e usar a lista em
>> http://www.mat.puc-rio.br/~obmlistas/obm-l.html
>> =
>>
>
> --
> Esta mensagem foi verificada pelo sistema de antivírus e
> acredita-se estar livre de perigo.
>
>
>
>
> --
> Esta mensagem foi verificada pelo sistema de antivírus e
> acredita-se estar livre de perigo.
>

-- 
Esta mensagem foi verificada pelo sistema de antivírus e
 acredita-se estar livre de perigo.



[obm-l] Re: [obm-l] Re: [obm-l] Preciso de uma ajuda para a solução de um problema.

2014-03-12 Por tôpico Ralph Teixeira
Eu entendi que x não é necessariamente inteiro, mas a expressão tem que ser
inteira (aliás, um inteiro positivo-ou-nulo, pois é raiz de um troço).
Então escrevi algo assim

(x^2+1000x)^(1/2)-x = n^2   (onde n pode ser a princípio 0,1,2,3,...)
então
(x^2+1000x) = (x+n^2)^2
então
1000x=2xn^2+n^4
então
x = n^4 / (1000-2n^2)

Bom, mas não é bem isso não, pois vê-se que x+n^2 tinha que ser >= 0... O
caso n=0 dá x=0 que não ajuda nem atrapalha a soma, então vou supor logo
que n é positivo e então:

x+n^2 >=0
sse
n^4 / (1000-2n^2) + n^2 >=0
sse
n^2 (1000 - n^2) / (1000-2n^2) >=0

Esta desigualdade implica em nraiz(1000)=10raiz(10)=31.6 Em suma, n só pode assumir os seguintes
valores:
{0,1,2,..., 22} ou {32,33,34,...}

Agora, enfim, com estas condições, tanto x+n^2 quando n são positivos,
então a equação original
((x^2+1000x)^(1/2)-x)^(1/2)=n
é realmente equivalente a
x = n^4 / (1000-2n^2)
o que pode ser confirmando revertendo os passos da dedução original lá em
cima (os "então" viram "sse").

O que você quer agora é achar a soma:

SOMATÓRIO (n^4 / (1000-2n^2)) onde n varia de 0 a 22, e depois de 32 a
Infinito.

Mas essa soma diverge, pois o termo geral não vai para zero!

---///---

Se o enunciado limitar x a ser POSITIVO, então devemos ter 2n^2<1000, isto
é, ficam apenas os números de 0 a 22. Então a resposta seria:

SOMATÓRIO (n=0 a 22) (n^4 / (1000-2n^2))

Eu tinha esperança de abrir em frações parciais e achar uma soma
telescópica, mas

n^4 / (1000-2n^2) = -n^2/2 - 250 -125000 / (n^2-500)

Os dois primeiros termos são fáceis de somar de 0 a 22, mas o último não
fica telescópico não!

---///---

Enfim, se x tiver que ser INTEIRO, então n^2-500 tem que ser divisor de
125000. Argh, sai, mas é horrendo.

Abraço,
 Ralph





2014-03-12 12:53 GMT-03:00 Bernardo Freitas Paulo da Costa <
bernardo...@gmail.com>:

> 2014-03-11 23:11 GMT-03:00  :
> > Olá , boa noite a todos os amigos da lista, recebi recentemente um
> problema
> > abaixo.
> >
> > Determinar a soma de todos os valores de x tais que
> > ((x^2+1000x)^(1/2)-x)^(1/2) seja inteiro.
>
> x é inteiro?
>
> --
> Bernardo Freitas Paulo da Costa
>
> --
> Esta mensagem foi verificada pelo sistema de antivírus e
>  acredita-se estar livre de perigo.
>
>
> =
> Instruções para entrar na lista, sair da lista e usar a lista em
> http://www.mat.puc-rio.br/~obmlistas/obm-l.html
> =
>

-- 
Esta mensagem foi verificada pelo sistema de antivírus e
 acredita-se estar livre de perigo.



[obm-l] Re: [obm-l] Re: [obm-l] Re: [obm-l] Re: [obm-l] Re: [obm-l] Polinômio

2014-03-09 Por tôpico Ralph Teixeira
Hm, cade o enunciado original do Marcone mesmo...?

Ah, aqui: era para provar que NAO EXISTIA P(x) com coeficientes inteiros
tal que blah-blah... Entao, fazemos por contradicao: suponha que HOUVESSE
P(x) com coeficientes inteiros Use a ideia do Nehab, e chegariamos a um
polinomio R(x)=ax^2+bx+c com coeficientes inteiros tal que R(1)=2, R(2)=3 e
R(3)=5. Mas o unico polinomio **quadratico** que serve nao tem coeficientes
inteiros, e temos a nossa contradicao.

Abraco,
 Ralph


2014-03-09 19:33 GMT-03:00 Pacini Bores :

> Desculpe Ralph,
>
> Mas se o termo de maior grau  de P(x) não for inteiro , a divisão dele por
> 1 será um número não inteiro; isso não garante que P(x) tenha coeficientes
> inteiros. Estou errado ?
>
>  O problema não é para provar que os coeficientes de P(x)  são inteiros ?
>
> Poderia esclarecer melhor para mim ?
>
> Abraços
>
> Pacini
>
>
> Em 9 de março de 2014 16:58, Ralph Teixeira  escreveu:
>
>> Contrariando o Nehab, acho que o Nehab tinha razao sim. :) :)
>>
>> Pense no algoritmo da divisao de P(x) por Z(x) -- se o coeficiente do
>> primeiro termo de Z(x) for 1 (eh o caso, Z(x)=(x-1)(x-2)(x-3)), entao soh
>> fazemos subtracoes e multiplicacoes (todas as divisoes sao por 1). Entao
>> certamente o quociente tambem terah coeficientes inteiros.
>>
>> Abraco,
>>  Ralph
>>
>>
>> 2014-03-09 15:50 GMT-03:00 Carlos Nehab :
>>
>> Oi, Bernardo (e demais colegas...)
>>>
>>> Toda razão pras observações do Bernardo!
>>> É ótimo tê-lo no pé da gente. Sempre atento (há décadas - rsrsrs).
>>> Minha suposta solução NÃO resolve o problema proposto pelo Marcone.
>>> Da proxima vez serei menos apressado...
>>>
>>> Obrigado e abraços,
>>> Nehab
>>>
>>> On 08/03/2014 16:19, Bernardo Freitas Paulo da Costa wrote:
>>>
>>>> 2014-03-08 14:41 GMT-03:00 Cláudio Gustavo :
>>>>
>>>>> Num polinômio com coeficientes inteiros, ao se substituírem dois
>>>>> valores quaisquer "a" e "b" do domínio e subtraindo as expressões de p(b) 
>>>>> e
>>>>> p(a) eh possível colocar o fator "b-a" em evidencia. Observando que o 
>>>>> outro
>>>>> fator que multiplica "b-a" continua sendo inteiro, tem-se que
>>>>> (p(b)-p(a))/(b-a) eh inteiro e que b-a divide p(b)-p(a).
>>>>>
>>>> Eu não contestei a sua solução, Cláudio. O meu problema é com a
>>>> solução do Nehab. Continuo sem ver como usar a expressão p(x) =
>>>> (x-1)(x-2)(x-3)Q(x) + ax^2 + bx + c ajuda a resolver a questão. A
>>>> divisão euclidiana que ele faz (conforme a outra mensagem dele na
>>>> lista) não garante que Q(x) tem coeficientes inteiros.
>>>>
>>>> Abraços,
>>>>
>>>
>>>
>>> ---
>>> Este email está limpo de vírus e malwares porque a proteção do avast!
>>> Antivírus está ativa.
>>> http://www.avast.com
>>>
>>>
>>> --
>>> Esta mensagem foi verificada pelo sistema de antivírus e
>>> acredita-se estar livre de perigo.
>>>
>>> 
>>> =
>>> Instruções para entrar na lista, sair da lista e usar a lista em
>>> http://www.mat.puc-rio.br/~obmlistas/obm-l.html
>>> 
>>> =
>>>
>>
>>
>> --
>> Esta mensagem foi verificada pelo sistema de antivírus e
>> acredita-se estar livre de perigo.
>>
>
>
> --
> Esta mensagem foi verificada pelo sistema de antivírus e
> acredita-se estar livre de perigo.

-- 
Esta mensagem foi verificada pelo sistema de antivírus e
 acredita-se estar livre de perigo.



[obm-l] Re: [obm-l] Re: [obm-l] Re: [obm-l] Polinômio

2014-03-09 Por tôpico Ralph Teixeira
Contrariando o Nehab, acho que o Nehab tinha razao sim. :) :)

Pense no algoritmo da divisao de P(x) por Z(x) -- se o coeficiente do
primeiro termo de Z(x) for 1 (eh o caso, Z(x)=(x-1)(x-2)(x-3)), entao soh
fazemos subtracoes e multiplicacoes (todas as divisoes sao por 1). Entao
certamente o quociente tambem terah coeficientes inteiros.

Abraco,
 Ralph


2014-03-09 15:50 GMT-03:00 Carlos Nehab :

> Oi, Bernardo (e demais colegas...)
>
> Toda razão pras observações do Bernardo!
> É ótimo tê-lo no pé da gente. Sempre atento (há décadas - rsrsrs).
> Minha suposta solução NÃO resolve o problema proposto pelo Marcone.
> Da proxima vez serei menos apressado...
>
> Obrigado e abraços,
> Nehab
>
> On 08/03/2014 16:19, Bernardo Freitas Paulo da Costa wrote:
>
>> 2014-03-08 14:41 GMT-03:00 Cláudio Gustavo :
>>
>>> Num polinômio com coeficientes inteiros, ao se substituírem dois valores
>>> quaisquer "a" e "b" do domínio e subtraindo as expressões de p(b) e p(a) eh
>>> possível colocar o fator "b-a" em evidencia. Observando que o outro fator
>>> que multiplica "b-a" continua sendo inteiro, tem-se que (p(b)-p(a))/(b-a)
>>> eh inteiro e que b-a divide p(b)-p(a).
>>>
>> Eu não contestei a sua solução, Cláudio. O meu problema é com a
>> solução do Nehab. Continuo sem ver como usar a expressão p(x) =
>> (x-1)(x-2)(x-3)Q(x) + ax^2 + bx + c ajuda a resolver a questão. A
>> divisão euclidiana que ele faz (conforme a outra mensagem dele na
>> lista) não garante que Q(x) tem coeficientes inteiros.
>>
>> Abraços,
>>
>
>
> ---
> Este email está limpo de vírus e malwares porque a proteção do avast!
> Antivírus está ativa.
> http://www.avast.com
>
>
> --
> Esta mensagem foi verificada pelo sistema de antivírus e
> acredita-se estar livre de perigo.
>
> =
> Instruções para entrar na lista, sair da lista e usar a lista em
> http://www.mat.puc-rio.br/~obmlistas/obm-l.html
> =
>

-- 
Esta mensagem foi verificada pelo sistema de antivírus e
 acredita-se estar livre de perigo.



[obm-l] Re: [obm-l] Problema de lógica

2014-03-05 Por tôpico Ralph Teixeira
Tah errado, eh 5, 10, 2, 9, 8, 4, 6, 7, 3, 1, 0.

Ordem alfabetica.


2014-03-05 16:20 GMT-03:00 Jorge Paulino :

> A sequência a seguir é formada por 10 números:
> 5, 10, 2, 8, 9, 4, 6, , , .
> Os 3 últimos números dessa sequência são, respectivamente,
> A) 1, 3 e 7. B) 1, 7 e 3. C) 3, 1 e 7. D) 7, 1 e 3. E) 7, 3 e 1.
>
> O gabarito é a letra E. Por que?
>
> --
> Esta mensagem foi verificada pelo sistema de antivírus e
> acredita-se estar livre de perigo.
>
> =
> Instruções para entrar na lista, sair da lista e usar a lista em
> http://www.mat.puc-rio.br/~obmlistas/obm-l.html
> =
>

-- 
Esta mensagem foi verificada pelo sistema de antivírus e
 acredita-se estar livre de perigo.



Re: [obm-l] Probabilidade Condicional

2014-02-15 Por tôpico Ralph Teixeira
Olha, o enunciado estah mal escrito... aquelas percentagens tem que ser
melhor definidas: 20% do que? 40% do que? De toda a populacao? De todos os
testes?

Na interpretacao mais literal, seria de todos os testes, e entao os numeros
nao fazem sentido (70% sao doentes; 40% sao falsos positivos, ou seja,
gente saudavel que testou +; soh ai jah tem 110%, e nem contamos os
negativos corretos).

Entao vou na interpretacao usual, que eh:
-- 70% de probabilidade de ter a doenca;
-- SE ELE FOR DOENTE, 20% de chance de dar negativo; ou seja, 20% ***dos
doentes*** recebem - no teste (incorretamente);
-- SE ELE FOR SAUDAVEL; 40% de chance de dar positivo; ou seja, 40% ***dos
saudaveis*** recebem + no teste (erradamente).

Na notacao usual, seria:
Pr(Doente)=0.7
Pr(A-|Doente)=0.2
Pr(A+|Nao Doente)=0.4

Entao monto uma tabela assim
   Doente  Nao Doente Total
A+ 56%   12%68%
A-  14%   18%32%
Total  70%   30%   100%

(Note a ordem da montagem -- comece pelo 70/30; 20% de 70% eh aquele 14%;
40% de 30% eh os 12%; depois complete por somas e diferencas).

Entao, se o cara testou + no Teste A, ele eh um dos 68%; a chance de ele
estar de fato doente eh 56/68.

Para o Teste B, faca o mesmo -- mas  COMO VOCE SABE QUE ELE TESTOU + no
TESTE A, voce comeca deste 56/68. A tabela fica assim

  Doente  Nao Doente Total
B+50.4%   1.2% 51.6%
B-  5.6%  10.8%  16.4%
Total 56%12%   68%

TODO MUNDO NESTA TABELA TESTA + EM A. Entao o numero que voce quer eh
50.4/51.6 ~= 97.67% -- eh a probabilidade do cara ter a doenca, SABENDO que
ele testou + em A e em B.

Abraco,
 Ralph



2014-02-12 19:34 GMT-02:00 João Sousa :

> Gostaria de auxílio para a resolução da questão abaixo.
>
>
>
> Após análise de sintomatologia, um médico estima que seu paciente tenha
> uma determinada doença com probabilidade
>
> de 70%. Para confirmar o diagnóstico inicial, ele pede ao paciente que
> faça um exame tipo A, que dá falso
>
> negativo com probabilidade de 20% e falso positivo com probabilidade de
> 40%. O resultado desse exame dá positivo.
>
> Entretanto, desconfiado com a alta frequência de falso positivo do exame
> tipo A, o médico pede novamente
>
> que o paciente se submeta a um exame tipo B, cujas probabilidades de falso
> positivo e falso negativo são ambas
>
> de 10%, independentemente dos resultados do teste A. Novamente o resultado
> do teste tipo B é positivo.
>
> Qual a probabilidade de que o paciente tenha de fato a doença condicionada
> aos dois resultados dos exames
> tipo A e B?
>
> João
>
> --
> Esta mensagem foi verificada pelo sistema de antivírus e
> acredita-se estar livre de perigo.
>

-- 
Esta mensagem foi verificada pelo sistema de antivírus e
 acredita-se estar livre de perigo.



[obm-l] Re: [obm-l] Congruências

2014-02-04 Por tôpico Ralph Teixeira
Sendo x esse numero, voce descobriu que x-1 eh multiplo de 7, de 11 e de 13.

Como eles sao primos, entao x-1 eh multiplo de 7.11.13 = 1001. Entao voce
tem razao: x deixa resto 1 na divisao por 1001.

Uma generalizacao desta ideia eh o "Teorema Chines do Resto":

http://pt.wikipedia.org/wiki/Teorema_chin%C3%AAs_do_resto

Ou "Teorema do Resto Chines"? :) :) :)
http://en.wikipedia.org/wiki/Chinese_remainder_theorem

Abraco,
Ralph


2014-02-04 marcone augusto araújo borges :

> Determinar o resto da divisão de 300^3000 por 1001
>
>
> Pelos meus cálculos essa potência dividida por 7,por 11
> ou por 13 deixa o mesmo resto 1
> como 7,11 e 13 são primos e 7.11.13 = 1001,posso afirmar
> 300^3000 dividido por 1001 deixa resto 1?
>
> --
> Esta mensagem foi verificada pelo sistema de antivírus e
> acredita-se estar livre de perigo.
>

-- 
Esta mensagem foi verificada pelo sistema de antivírus e
 acredita-se estar livre de perigo.



[obm-l] Re: [obm-l] Re: [obm-l] RE: [obm-l] Re: [obm-l] Limite de uma variável

2014-01-01 Por tôpico Ralph Teixeira
Desculpa, Pacini, mas isto nao faz sentido se voce nao disser algo sobre o
que x significa. A frase que voce escreveu:

"para todo k>0, existe x real tal que 0<|x-a|A) f(x) = L(ou, equivalentemente, lim_(x->A) y=L )
(le-se: "o limite de f(x), quando x tende a A, eh igual a L;ou  "y tende a
L quando x tende a A")

SIGNIFICA

"eh possivel garantir que y=f(x) fique tao perto quanto eu quiser de L,
bastando para tanto que x fique suficientemente proximo de A"
(ou em linguagem mais formal, algo parecido com o que o Kelvin falou ali em
cima).

---///---

Voce quer limites infinitos? Ou limites no infinito? Tah, mudamos um
pouquinho:

lim_(x->A) f(x)=+Inf
SIGNIFICA
"eh possivel garantir que f(x) fique tao grande quanto eu queira, bastando
para tanto que x fique suficientemente proximo de A"
(formalmente: para todo K real, existe delta tal que vale
 |x-A|  f(x)>K)

lim_(x->+Inf) f(x)=L
SIGINIFICA
"eh possivel garantir que f(x) fique tao perto quando eu quiser de L,
bastando para tanto que x seja suficientemente grande"
(para todo eps>0, existe K real tal que vale  x>K ==>
|f(x)-L|

> Olá Pedro,
>
> Podemos definir o que desejas da seguinte forma :" limx =a" , com a real;
>
> " para todo k>0 , existe x  real tal que  0 < |x - a| < k " .
>
> Abraços
>
> Pacini
>
>
> Em 1 de janeiro de 2014 08:06, Pedro Chaves escreveu:
>
> 
>> > Date: Tue, 31 Dec 2013 17:50:20 -0200
>> > Subject: [obm-l] Re: [obm-l] Limite de uma variável
>> > From: kelvinan...@gmail.com
>> > To: obm-l@mat.puc-rio.br
>>
>>  Olá, Kelvin!
>>
>> Muito obrigado!
>>
>> Gostaria, entretanto, de uma definição de limite de uma variável, e não
>> de uma função.
>>
>> Feliz Ano Novo!
>> Pedro Chaves
>> ___
>> >
>>
>>
>> Dada a função ƒ(x) definida no intervalo aberto em torno de a, mas não
>> > necessariamente definida em a, temos que:
>> > Limite é o número L ao qual aproximam-se os valores de ƒ(x), quando x
>> > tende a um número a.
>> > Se, e somente se, existir um número ε> 0, e que para cada ε, existir
>> > um número δ> 0, e qualquer que seja o x, seja válido:
>> > 0 < |x - a| < δ que implica em |ƒ(x) - L| < ε.
>> >
>> >
>> >
>> > Em 31 de dezembro de 2013 17:09, Pedro Chaves
>> > mailto:brped...@hotmail.com>> escreveu:
>> > Qual a definição de limite de uma variável real?
>> >
>> > Feliz 2014 para todos!!!
>> >
>> > Pedro Chaves
>> > _
>> > --
>> > Esta mensagem foi verificada pelo sistema de antivírus e
>> > acredita-se estar livre de perigo.
>> >
>> >
>> >
>> =
>> > Instruções para entrar na lista, sair da lista e usar a lista em
>> > http://www.mat.puc-rio.br/~obmlistas/obm-l.html
>> >
>> =
>> >
>> >
>> > --
>> > Esta mensagem foi verificada pelo sistema de antiv�rus e
>> > acredita-se estar livre de perigo.
>> --
>> Esta mensagem foi verificada pelo sistema de antivírus e
>>  acredita-se estar livre de perigo.
>>
>>
>> =
>> Instru�ões para entrar na lista, sair da lista e usar a lista em
>> http://www.mat.puc-rio.br/~obmlistas/obm-l.html
>> =
>>
>
>
> --
> Esta mensagem foi verificada pelo sistema de antivírus e
> acredita-se estar livre de perigo.
>

-- 
Esta mensagem foi verificada pelo sistema de antiv�rus e
 acredita-se estar livre de perigo.



[obm-l] Re: [obm-l] Re: [obm-l] Não consigo resolver

2013-12-19 Por tôpico Ralph Teixeira
Super dica para a 2: crie angulos z_i com tan(z_i)=y_i. Entao a condicao
passa a ser 0<=tan(z_i-z_j)<=1, ou seja, basta que 0<=zi-zj<=pi/4. Agora,
se voce pegar 5 angulos no circulo trigonometrico, pela casa dos pombos...

Ajudou?

Abraco, Ralph.


2013/12/19 saulo nilson 

> n=p1^ap2^b*p3^c
> em que a, b,c,...e maior do que a soma dos expoentes da decomposiçao dos
> numeros menores que n.
>
>
> 2012/8/9 Heitor Bueno Ponchio Xavier 
>
>> Estou com alguns problemas aqui que não estão saindo e agradeceria
>> bastante ajuda.
>>
>> 01. Encontre todos os números ''n'' naturais tais que n² não seja divisor
>> de n!
>>
>> 02.Prove que dentre quaisquer cinco reais y_1, y_2, y_3, y_4, y_5,
>> existem dois que satisfazem:
>> 0<= (y_i - y_j)/(1+(y_i)(y_j)) <=1.
>>
>
>
> --
> Esta mensagem foi verificada pelo sistema de antivírus e
> acredita-se estar livre de perigo.

-- 
Esta mensagem foi verificada pelo sistema de antivírus e
 acredita-se estar livre de perigo.



Re: [obm-l] off-topic scratch

2013-12-15 Por tôpico Ralph Teixeira
Minha opiniao pessoal: qualquer atividade que melhore a LOGICA
(matematica) dos alunos eh bem-vinda. Meu chute eh que programacao
ajuda muito, mas outros colegas devem ter opinioes mais
bem-informatizadas, quero dizer, informadas. :)

Abraco,
  Ralph

2013/12/15 Hermann :
> OFF-TOPIC SCRATCH
>
> Em 1999 iniciei aulas de geometria com o CABRI num colégio de minha região.
> Realmente na época INOVADOR.
>
> Estava pensando em propor aulas de SCRATCH como um novo auxiliar no estudo
> (incentivo) a matemática.
>
> O Scratch me parece um LOGO hiper melhorado, lembro que na decada de 80
> amigos  achavam o LOGO genial.
>
> MINHA PERGUNTA:
>
> Alguém (além do OBAMA-EUA) concorda que o ensino precoce de programação vai
> contribuir para incentivar o aluno em matemática?
>
> Gostaria REALMENTE da opinião dos colegas do fórum.
>
> Obrigado
>
> Hermann
>
> ps: alguém no fórum usa o SCRATCH ou LOGO ou qq coisa semelhante?
>
> --
> Esta mensagem foi verificada pelo sistema de antivírus e
> acredita-se estar livre de perigo.

-- 
Esta mensagem foi verificada pelo sistema de antivírus e
 acredita-se estar livre de perigo.


=
Instruções para entrar na lista, sair da lista e usar a lista em
http://www.mat.puc-rio.br/~obmlistas/obm-l.html
=


[obm-l] Re: [obm-l] Recorreção - IME 2014

2013-12-11 Por tôpico Ralph Teixeira
Por mim, a primeira parte estah corretissima (a menos que eles queiram que
voce demonstre MA>=MG; pode ser que eles tenham decidido que nao podia usar
isso sem demonstrar).

Para a 2a parte, tem um problema tecnico chato: voce mostrou que, SE HOUVER
UM MAXIMO para p, eh no triangulo equilatero. Mas voce nunca exibiu um
argumento para dizer que este maximo existe... Na minha opiniao, problema
tecnico, importante, mas que nao invalida as ideias que voce usou.

Para mim, certamente vale muito mais que 0,1.

Abraco,
  Ralph


2013/12/11 João Maldonado 

> Boa Tarde pra todo mundo :)
>
> Eu prestei o IME no mês de outubro e recentemente chegou a prova corrigida
> no meu email,
> Eu fiquei com nota 9 em matemática, mas jurava que tinha acertado a última
> questão (pelo menos a letra A), e a nota que eles deram foi 0.1. Eu tenho
> até depois de amanhã pra poder pedir recorreção da questão, queria pedir a
> ajuda de vocês, não sei se o que eu escrevi é válido eu não
>
> Obs. a dupla seta significa se e somente se
>
>
>
> --
> Esta mensagem foi verificada pelo sistema de antivírus e
> acredita-se estar livre de perigo.
>

-- 
Esta mensagem foi verificada pelo sistema de antivírus e
 acredita-se estar livre de perigo.



Re: [obm-l] Raplh Costa Teixeira, pergunda sobre seus livros OFF TOPIC

2013-12-03 Por tôpico Ralph Teixeira
Oi, Joao.



Olha, os nossos livros tendem a cobrir MAIS do que o Ensino Medio
tipicamente cobre. Estes livros foram redigidos pensando num curso com o
mesmo nivel do excelente Colegio Santo Inacio aqui do Rio (bom, voce jah
sabia, considerando que 3 dos autores sao professores de lah). Pelo que eu
me lembro dos programas atuais do IME e do ITA (corrija-me se eu estiver
errado), a gente cobre a materia toda deles (ha tempos eles nao cobram mais
Calculo nem Analise nem Desenho Geometrico, correto?) -- inclusive a
Probabilidade Condicional que quase ninguem faz no Ensino Medio. E ha nos
nossos livros varios exercicios dos vestibulares do IME e do ITA e de nivel
parecido. Entao nosso livro eh apropriado para estudar para IME e ITA sim.

Agora, *suficiente* no sentido de "soh precisa desse livro e de mais nada",
eu diria que nao. Meu palpite eh de que qualquer candidato vai precisar
fazer muitos muitos MUITOS problemas de matematica alem de qualquer *um*
livro para ficar bem preparado para esse vestibular. Nesse sentido ajuda
pegar provas antigas, apostilas de cursinhos e outros materiais mais
focados especificamente nesses concursos -- ou mesmo uma turma IME/ITA de
um curso.

Mas confesso que dos 4 autores eu sou o que estah mais longe desses
vestibulares ultimamente. Talvez outros membros da lista pudessem opinar?

Abraco,
Ralph

-- 
Esta mensagem foi verificada pelo sistema de antivírus e
 acredita-se estar livre de perigo.



Re: [obm-l] Pra quem gosta de geometria plana...

2013-11-04 Por tôpico Ralph Teixeira
Para o (2): pense assim: se a área é máxima, então de todos os pontos da
circunferência que contém C, temos que escolher o mais longe possível do
segmento AB (pois, fixando A e B, este C é o que dá a área máxima). Ou
seja, o segmento AB tem que ser paralelo à tangente ao círculo de C, em C.
Isto significa que OC é perpendicular a AB.

Analogamente, OA é perpendicular a BC e OB é perpendicular a AC. Portanto,
O tem que ser o ortocentro do triângulo ABC.

Abraço,
Ralph


2013/11/4 

>  Olá amigos estou reenviando , caso alguns amigos ainda nao tenham
> recebido!!! pois sao
>
> m uitos emails
>
>  Mensagem original 
>  Assunto: [obm-l] Pra quem gosta de geometria plana... Data: 23.10.2013
> 11:06 De: douglas.olive...@grupoolimpo.com.br Para:  
> Responder
> para: obm-l@mat.puc-rio.br
>
>
>
>
> Olá, boa tarde , preciso da ajuda de vocês em duas questões abaixo
>
>
>
> 1)Dado um quadrilátero convexo ABCD circunscritível, traça-se as diagonais
> AC e BC que se interceptam em E, mostre que o quadrilátero formado pelos
> incentros dos  4 triângulos ABE, BCE, CDE, ADE é inscritível.
>
>
> 2)Dados três círculos concentricos de raios 1cm , 2cm, e 3cm, marca-se um
> ponto em cada circunferência, e com os três pontos A, B e C forma-se um
> triângulo de área máxima, assim o centro do círculo representa qual ponto
> notável do triângulo?
>
> Tentei , mas infelizmente  não obtive muito sucesso
> Qualquer ajuda será bem vinda que ainda estou tentando ...
>
> Att: Douglas Oliveira
>
>
> --
> Esta mensagem foi verificada pelo sistema de antivrus e
> acredita-se estar livre de perigo.
>
>
> --
> Esta mensagem foi verificada pelo sistema de antivírus e
> acredita-se estar livre de perigo.
>

-- 
Esta mensagem foi verificada pelo sistema de antivírus e
 acredita-se estar livre de perigo.



Re: [obm-l] Conjecturas especiosas

2013-10-23 Por tôpico Ralph Teixeira
As seguintes não são bem o que você quer, mas eu as uso quando falo de
indução finita:
i) n^2+n+41 eh primo para n=0,1,2,3,...
ii) A e B são números cujas representações decimais coincidem em seus
dígitos iniciais correspondentes (da esquerda para a direita); no primeiro
digito em que diferem, o digito de A eh menor que o digito de B. Então A=5 (este, que eu saiba,
estah em aberto!)

Se eles souberem Calculo, tem:
i) Se f`(a)>0, então f eh crescente em algum intervalo em volta de x=a.
ii) Suponha que lim (t->0) f(t,mt)=lim(t->0) f(mt,t)=0 para todo m real.
Entao lim ((x,y)->(0,0)) f(x,y)=0.

Ou, que tal:
i) A Terra eh plana (se bem que os pessoal mais culto sabia que ela era
redonda muito antes de Colombo);
ii) O tempo passa da mesma forma para todos (Relatividade?);
iii) Nada pode estar em dois lugares ao mesmo tempo (fótons?);
iv) O universo tem 3 dimensões, 4 se contar o tempo (duvidoso, mas veja
"Teoria das Supercordas");
v) Tudo o que eu vejo deve ser verdade, o meu cérebro não **inventa**
informação visual (vide "ponto cego");
vi) Tudo o que sobe, desce (Voyager?);
vii) Eu existo (talvez essa seja verdade... o que eu acho fascinante do
Descartes não eh a frase em si, mas que ele se PREOCUPOU em tentar
demonstrar que existia!).
viii) Provar coisas em Matemática não eh importante. :) :) :)

Abraco,
Ralph

-- 
Esta mensagem foi verificada pelo sistema de antivírus e
 acredita-se estar livre de perigo.



[obm-l] Re: [obm-l] cálculo curvas parametrizadas

2013-10-03 Por tôpico Ralph Teixeira
Em linhas gerais, sim, concordo.

Mais especificamente, eu inverteria a linguagem e diria que essas derivadas
dão o crescimento de x e y com relação a t (suponho que sua nomenclatura
seja x(t)=t^3 e y(t)=t^2-t); e "plano yox" é um pouco estranho, eu diria
"plano x-y", na orientação usual. Se "para baixo" e "para cima" indicam
ideia geral da direção da velocidade (não apenas VERTICALMENTE para cima ou
para baixo), concordo.

Eu costumo pensar nas coisas juntas: o vetor velocidade é
v(t)=(x'(t),y'(t)); então os sinais de x' e y' em cada ponto dão a ideia
geral da direção ("Quadrante Noroeste", "Quadrante Sudeste", etc.) para
onde a velocidade aponta.

Abraço,
   Ralph


2013/10/3 Hermann 

> **
> Meus amigos, boa tarde, como já disse por diversas vezes, a insegurança
> mata e a ajuda dos colegas do fórum é que me tranquiliza.
>
> Não vou fazer uma pergunta vou fazer uma afirmação e gostaria (muito) da
> opinião de vocês:
>
>
> Tenho uma curva parametrizada tipo (t^3, t^2-t)
>
> se eu estudar o sinal de dy/dt terei o crescimento e decrescimento de t em
> relação a y, visualmente no gráfico yox teríamos o vetor velocidade
> apontando para cima ou para baixo,
>
> e se eu estudar o sinal de dx/dt teriamos o crescimento e decrescimento de
> t em relação a x e o vetor velocidade apontando para a esqueda(-) ou para
> direita(+).
>
> Concordam!?!?!?!?
>
> abraços
> Hermann
>
> --
> Esta mensagem foi verificada pelo sistema de antivírus e
> acredita-se estar livre de perigo.
>

-- 
Esta mensagem foi verificada pelo sistema de antivírus e
 acredita-se estar livre de perigo.



[obm-l] Re: [obm-l] Re: [obm-l] Função periodica

2013-09-16 Por tôpico Ralph Teixeira
Usando MA>=MG, voce mostra que **=x1/x2+x2/x3+...+x(n-1)/xn+xn/x1 >= n para
quaisquer x1,x2,...,xn>0.

Suponha b=T/n. Entao divida a integral em n pedaços, com intervalos 0 a b,
b a 2b, ..., (n-1)b a b. Coloque todas no intervalo 0 a b (tomando y=x na
primeira, y=x-b na segunda, etc.), e voce vai ficar com uma integral de 0 a
b cujo integrando tem a cara de ** acima (onde x1=f(x), x2=f(x+b),...etc.).
Entao a integral é maior ou igual que Int(0 a b) n dx=nb=T.

E se b não for dessa forma? Bom, se for b=mT/n com m e n inteiros voce pode
fazer o mesmo truque integrando de 0 a mT=nb (que são m cópias da integral
original, pois f é periódica de período T)... Você vai acabar mostrando que
m*(Integral original) >= nb=mT usando o mesmo tipo de raciocínio.

Enfim, como a sua integral depende continuamente de b, e a gente acabou de
mostrar que ela vale >=T em todos os b múltiplos racionais de T (que é
denso em R)... acabou.

Abraço,
Ralph


2013/9/16 Bernardo Freitas Paulo da Costa 

> 2013/9/16 Francisco Lage :
> > Alguém pode me ajudar?
> >
> > Seja F : R -> R*+ , uma função continua e periódica de período T , prove
> que
> > (1/T)*inegral(f(x)/f(x+b))dx de 0 até 1 é maior ou igual a T , para todo
> b
> > real
>
> Isso tá meio errado... se f(x) = 1 para todo x, então a integral dá
> 1/T... Não seria "1/T * (integral de 0 até T) >= 1" ?
> --
> Bernardo Freitas Paulo da Costa
>
> --
> Esta mensagem foi verificada pelo sistema de antivírus e
>  acredita-se estar livre de perigo.
>
>
> =
> Instruções para entrar na lista, sair da lista e usar a lista em
> http://www.mat.puc-rio.br/~obmlistas/obm-l.html
> =
>

-- 
Esta mensagem foi verificada pelo sistema de antivírus e
 acredita-se estar livre de perigo.



Re: [obm-l] Irracional entre dois racionais

2013-09-16 Por tôpico Ralph Teixeira
Noto que raiz(x^2+1)+raiz(x^2-1) = 2 / (raiz(x^2+1)+raiz(x^2-1))~~ 2/2x =
1/x. Então meu primeiro palpite é 1/8=0.125.

Mas o problema é saber se isto está acima ou abaixo de 1/8, então quero
saber (para x>1):
raiz(x^2+1)+raiz(x^2-1) > 2x ???

Façamos equivalências:
raiz(x^2+1)+raiz(x^2-1) > 2x
sse
x^2+1+2raiz(x^2+1)raiz(x^2-1)+x^2-1 > 4x^2
sse
raiz(x^4-1) > x^2 = raiz(x^4)

E isto é claramente FALSO (para todo x>1). Então eu concluo que:
raiz(x^2+1)+raiz(x^2-1) > 1/x
E minha resposta é 0.13.

Abraço,
Ralph.


2013/9/16 marcone augusto araújo borges 

> O número raiz(65) - raiz(63) está mais próximo de:
>
> a) 0,12   b) 0,13   c) 0,14   d) 0,15   e) 0,16
>
> --
> Esta mensagem foi verificada pelo sistema de antivírus e
> acredita-se estar livre de perigo.
>

-- 
Esta mensagem foi verificada pelo sistema de antivírus e
 acredita-se estar livre de perigo.



[obm-l] Re: [obm-l] Álgebra(não tá saindo)

2013-09-14 Por tôpico Ralph Teixeira
Escreva a multiplicacao que nem a gente fazia lah na 4a serie:
_6
  x4
6_

Agora vah fazendo a multiplicacao. 6x4=24, entao poe o 4, vai 2.
Mas, se eh 4 ali embaixo, eh 4 do lado esquero do 6. Entao fica algo assim:
46
   x4
64
Agora 4x4=16, mais 2, dah 18. Entao poe o 8 no resultado E TAMBEM DO LADO
DO 4 NA PRIMEIRA LINHA (e vai 1).
___846
   x4
6___84
4x8=32, +1=33. O proximo eh 3. Continue assim achando os digitos da direita
para a esquerda: 5, 1... E entao o proximo eh 6, que PODE ser aquele 6
inicial!

Assim, o menor numero inteiro n eh 153846.

Abraco,
Ralph



2013/9/14 marcone augusto araújo borges 

> Encontre o menor inteiro positivo n que possui as seguintes propriedades:
> I. Em sua representação tem o 6 como último dígito
> II.Se o último dígito(6) é apagado  e colocado na frente dos dígitos
> restantes,o número resultante
> é quatro vezes maior que o número original n
>
> --
> Esta mensagem foi verificada pelo sistema de antivírus e
> acredita-se estar livre de perigo.
>

-- 
Esta mensagem foi verificada pelo sistema de antivírus e
 acredita-se estar livre de perigo.



[obm-l] Re: [obm-l] Números inteiros

2013-09-10 Por tôpico Ralph Teixeira
xy-143x-143y=0
(x-143)(y-143)=143^2=11^2.13^2

Olhando os divisores daquele numero a direita, sai.

Abraco,
   Ralph


2013/9/10 marcone augusto araújo borges 

> Encontre todos os inteiros positivos x e y tais que 1/x + 1/y = 1/143
>
> Eu encontrei y = x^2/(x -143) - x e deu pra ver que
> x = 144 e y = 144*143 satisfaz.Mas foi só.
> Alguém ajuda?
>
>
> --
> Esta mensagem foi verificada pelo sistema de antivírus e
> acredita-se estar livre de perigo.
>

-- 
Esta mensagem foi verificada pelo sistema de antivírus e
 acredita-se estar livre de perigo.



[obm-l] Re: [obm-l] parametrização

2013-09-08 Por tôpico Ralph Teixeira
Voce sempre pode colocar x=r.cost, y=r.sint (onde t=theta) e correr
pro abraco. Ninguem disse que vai ficar bonito, mas funciona. :P

Sua circunferencia, por exemplo, pode ser escrita:

(rcost-2)^2+(rsint-3)^2=1

que voce pode simplificar ou operar um pouco, se desejar (ateh dah
para colocar r em funcao de t resolvendo a quadratica -- duas funcoes,
de fato), mas no fundo eh isso mesmo.

Abraco,
   Ralph

2013/9/8 Hermann :
> Meus amigos, bom dia
>
> Gostaria de uma ajuda de vocês, se possível.
>
> Como escrevo uma elipse centrada na origem em coordenadas polares.
>
> Como escrevo uma circunferência por exemplo  (x-2)^2 +(y-3)^=1 em coordendas
> polares
>
> Gostaria de saber se alguém conhece um bom livro ou bom texto que ensine
> parametrização.
>
> Abraços
> Hermann
>
> e novamente desculpem minha ignor^ncia
>
> --
> Esta mensagem foi verificada pelo sistema de antivírus e
> acredita-se estar livre de perigo.

-- 
Esta mensagem foi verificada pelo sistema de antivírus e
 acredita-se estar livre de perigo.


=
Instruções para entrar na lista, sair da lista e usar a lista em
http://www.mat.puc-rio.br/~obmlistas/obm-l.html
=


[obm-l] Re: [obm-l] Álgebra

2013-09-05 Por tôpico Ralph Teixeira
Mas a sua solucao esta tao boa...

Como abc>0,ninguem pode ser 0.

Ok, suponha a negativo. Como abc>0, um dos outros tem que ser negativo, o
outro positivo. Entao suponha a=-x, b=-y e c=z com x,y,z positivos.

Temos entao z>x+y e xy>z(x+y). Mas entao xy>(x+y)^2, o que contradiz
(x+y)/2>=raiz(xy).

Abraco, Ralph.
On Sep 5, 2013 9:21 AM, "marcone augusto araújo borges" <
marconeborge...@hotmail.com> wrote:

> Sejam a,b e c numeros reais tais que a+b+c > 0,ab+ac+bc > 0 e abc > o
> Prove que a > 0,b > 0 e c > 0.
>
> Seja Ax^3 + Bx^2 + Cx + D = 0 uma equação cujas raizes sao a,b e c.
> Quero mostrar que x nao pode ser negativo
> Pelo enunciado e pelas relações de Girard,B e D tem sinais contrarios ao
> de A e C tem o mesmo sinal de A
> 1) Se A é negativo e x idem,temos os 4 termos positivos e a soma deles nao
> pode ser zero
> 2) Se A é positivo e x negativo,temos os 4 termos negativos e a soma nao
> pode ser zero.
> Alguem mostraria outra solução?
>
>
>
>
>
> --
> Esta mensagem foi verificada pelo sistema de antivírus e
> acredita-se estar livre de perigo.
>

-- 
Esta mensagem foi verificada pelo sistema de antivírus e
 acredita-se estar livre de perigo.



[obm-l] Re: [obm-l] Base e e aproximação

2013-09-01 Por tôpico Ralph Teixeira
Suponho que voce quis dizer:

f(t)=q_0 e^(-kt) = q_0 ((e^(-k)))^t

e

g(t)=q_0 B^tonde   B=(q_f/q_0)^(-1/n)

A primeira eh uma funcao exponencial escrita do jeito usual, com base
e; a de baixo eh algo que voce descobriria partindo das condicoes
g(0)=q_0 e g(-n)=q_f. Eu escrevi exatamente o que voce escreveu,
mudando um pouquinho a notacao, e inventei este "B" para o proximo
paragrafo...

Sim, se voce escrever do jeito que eu escrevi, dah para ver que elas
sao EXATAMENTE a mesma funcao bom, desde que voce ajuste o k
corretamente, a saber, fazendo B=e^(-k), isto eh, k=-ln(B).

Em suma, tem 2 jeitos comuns de escrever funcoes exponenciais:
P(t)=P_0.e^(kt) ou P(t)=P_0.(a^t). Sao exatamente a mesma funcao,
desde que a=e^k! Eu chamo "a" de "fator anual" da funcao P(t) (ou
semestral, ou mensal, dependendo da unidade do t), enquanto "k" eh a
"taxa (relativa) continua anual" da funcao P(t). MESMA funcao, duas
maneiras de escrever.

(O pessoal da Matematica Financeira botaria um 3a jeito,
P(t)=P_0.(1+i)^t, onde i eh a "taxa efetiva de juros anual"... Mas eu
prefiro calcular o fator a=1+i, e esquecer esse i, que eh horrivel de
trabalhar. Na nossa colonia utopica em Marte, os bancos nao vao dizer
a taxa de juros, eles vao dar o ***multiplicador*** "a", e ninguem
jamais vai se enrolar com juros compostos. E "juros nominais" nem vao
existir, porque sao ridiculos e nao significam nada de verdade. :P :P
:P)

Abraco,
   Ralph

2013/8/31 Marco Antonio Leal :
> Boa noite, estudando exponencial, percebi que ao usar o modelo q(x ) = qo .
> e ^-(kx)  para representar o decaimento radioativo, o gráfico da função é
> idêntico ao da função q ( x) = q i ( qi / qf ) ^ t/ n
>
> Gostaria de saber se existe alguma aproximação que torne uma função igual a
> outra e onde encontraria material sobre isso
>
> --
> Esta mensagem foi verificada pelo sistema de antivírus e
> acredita-se estar livre de perigo.

-- 
Esta mensagem foi verificada pelo sistema de antivírus e
 acredita-se estar livre de perigo.


=
Instruções para entrar na lista, sair da lista e usar a lista em
http://www.mat.puc-rio.br/~obmlistas/obm-l.html
=


Re: [obm-l] Probabilidade

2013-09-01 Por tôpico Ralph Teixeira
Nao eh soh probabilidade, eh Teoria dos Jogos. E fica mais dificil
porque dois pistoleiros comecam com D. Francamente! :) :) :) )

Como todos os problemas com jogos sequenciais, tem que pensar de tras
para frente. Primeiro, pense o que ocorre se ficarem soh dois
pistoleiros...

Claramente, eles vao atirar um no outro ateh alguem acertar. Por
exemplo, se ficarem B e Do, com B atirando primeiro... hmm, desenhe
uma arvore de probabilidades. Fica:
a. 25% de chance de B matar Do; senao, dos 75% restantes...
   b. 50% de chance de Do Matar B; dos 50% restantes...
  c. 25% de chance de B matar Do. etc.

Ou seja, a chance de B vencer eh
25%+75%.50%.25%+75%.50%.75%.50%.25%+... Esta eh uma PG infinita, soma
25%/(1-75%.50%)=2/5=40%.

Em geral, no caso XY (onde X comeca) se X tem chance p de acertar o
tiro e Y tem chance q, entao a chance de X ganhar o duelo eh
p/(1-(1-q)(1-p)). Fazendo isso para todos os casos XY (onde X comeca
atirando), chegamos a uma tabela que acho ser...

BDo: B=40%, Do=60%
DoB: Do=80%; B=20%
BDa: B=4/13; Da=9/13
DaB: Da=12/13; B=1/13
DoDa: Do=4/7; Da=3/7
DaDo: Da=6/7; Do=1/7

Se B acertar Do, ele leva tudo para DaB, e tem soh 1/13 de chance; se
ele acertar Da, leva para DoB, e tem 20% de chance. A segunda opcao eh
melhor para B! Entao B vai atirar em Da.

Analogamente, a partir do truelo, Do decide atirar em Da (60%>1/7), e
Da atira em Do (9/13>3/7).

Deixa eu fazer uma pausa aqui: de fato, era meio obvio pelas regras do
problema que cada um vai atirar no "melhor" cara restante. O que nao
era tao obvio eh o seguinte: B talvez preferisse passar a vez! De
fato, ele poderia deixar Do atirar em Da... se Do acerta, vai para BDo
(com **B** comecando!!!), com 40% de chance para B (melhor que 1/5);
se Do erra, nao tem problema, porque agora Da atira em Do -- se
acertar, fica BDa, com 4/13 de chance, ainda melhor que 1/5; se errar,
volta tudo ao inicio! Ou seja, a melhor coisa que B faria seria atirar
para o alto -- mas seu enunciado NAO PERMITE isso (regra 4), entao
este paragrafo eh inutil.

Voltando ao problema original entao, fica assim:
1. 25% de chance de B matar Da (deixando DoB); senao, dos 75% restantes...
   2. 50% de chance de Do matar Da (para BDo); dos 50% restantes...
   3. 75% de chance de Da matar Do (para BDa); dos 25% restantes...
  4. 25% de chance de B matar Da (DoB); senao...

Entao a chance de B vencer eh:
(25%)(20%)+(75%)(50%)(40%)+(75%)(50%)(75%)(4/13)+(75%)(50%)(25%)(25%)(20%)+(75%)(50%)(25%)(75%)(50%)(40%)+...

que eh outra PGs infinita de razao (75%)(50%)(25%) (o primeiro termo
sao as 3 primeiras parcelas da soma). Eu devo ter errado contas com
chance 97.55%, mas achei aqui 596/1885=31.62% de chance de B ganhar
(como eh bom ficar "sob o radar", deu quase 1/3!). Os outros podem ser
calculados de maneira analoga -- aproveite para conferir minhas
contas... :)

Abraco,
  Ralph

2013/9/1 João Maldonado :
> Meu amigo me passou a seguinte questão, que não pude resolver
> O ano era 1872, auge do velho oeste americano, quando numa pequena cidade ao
> sul do Texas, os 3 maiores pistoleiros da época se encontraram Billy, Doolin
> e Dalton, contaminados pelos seus enormes egos, se desentenderam e
> resolveram tirar a história a limpo, naquilo que seria o maior confronto da
> história daquele país.
>
> Apesar de rivais, os 3 pistoleiros eram homens muito honrados, e resolveram
> propor uma série de regras para o duelo:
>
> 1 - eles iriam se posicionar em vértices de um triângulo equilátero, de tal
> forma que a distância entre eles sempre fosse a mesma, assim, nenhum
> pistoleiro alteraria suas chances de acerto durante o duelo.
>
> 2 - Como Billy tinha um índice de acerto menor que o dos outros dois, ele
> seria o primeiro a atirar, depois Doolin e em seguida o destemido e nunca
> antes derrotado Dalton.
>
> 3 - Billy tinha um índice de acerto de 1/4, enquanto Doolin acertava 1 em
> cada dois tiros, e Dalton inacreditáveis 75%.
>
> 4 - Os pistoleiros obrigatoriamente devem atirar em alguém, não podendo
> passar a vez ou errar de propósito.
>
> Suponha que nenhum deles fuja e que é necessário apenas um tiro para um
> pistoleiro morrer e que o duelo só termina quando só restar um sobrevivente,
> calcule a probabilidade de cada um deles vencer supondo que todos adotem uma
> estratégia perfeita.
>
> --
> Esta mensagem foi verificada pelo sistema de antivírus e
> acredita-se estar livre de perigo.

-- 
Esta mensagem foi verificada pelo sistema de antivírus e
 acredita-se estar livre de perigo.


=
Instruções para entrar na lista, sair da lista e usar a lista em
http://www.mat.puc-rio.br/~obmlistas/obm-l.html
=


Re: [obm-l] Contagem(?)

2013-08-31 Por tôpico Ralph Teixeira
Muitos desses problemas do tipo "de quantas maneiras..." sao obscuros
porque nao explicam o que eh "maneira".

Neste caso, "1+2009" e "2009+1" sao a mesma maneira? Se sim, a resposta eh
x; caso contrario, a resposta eh 2x. Talvez esta seja a diferenca entre sua
resposta e o gabarito.

Abraco,
Ralph


2013/8/31 Johann Dirichlet 

>  Em 30-08-2013 21:58, marcone augusto araújo borges escreveu:
>
> De quantas maneiras podemos escrever 2010 como soma de dois
> inteiros positivos primos entre si?
>
>  Eu tirei todas as possíveis parcelas que fossem um múltiplo dos
> divisores de 2010,mas achei 528 e o gabarito dá 264.
>
>
>
> Se x+y=2010 e d|x e d|y, então d|2010. Basta então contar todos os números
> primos com 2010 e menores que 2010 - e dividir o total por 2.
>
> Isto é a infame função phi de Euler!
>
>
> --
> Esta mensagem foi verificada pelo sistema de antivírus e
> acredita-se estar livre de perigo.
>
>
>
> --
> Esta mensagem foi verificada pelo sistema de antivírus e
> acredita-se estar livre de perigo.
>

-- 
Esta mensagem foi verificada pelo sistema de antivírus e
 acredita-se estar livre de perigo.



[obm-l] Re: [obm-l] Re: [obm-l] RE: [obm-l] Re: [obm-l] Teoria dos números

2013-08-30 Por tôpico Ralph Teixeira
Acho que voce pensou em 7^x como multiplicacao - ele quer potencia...:-(
:-(
 On Aug 29, 2013 9:17 PM, "Eduardo Wilner" 
wrote:

> Observe que (1 + 3k , 1 + 7k) , k inteiro, satisfaz a equação diofantina
>
> [ ]'s
>
>
>   --
>  *De:* marcone augusto araújo borges 
> *Para:* "obm-l@mat.puc-rio.br" 
> *Enviadas:* Quinta-feira, 29 de Agosto de 2013 12:18
> *Assunto:* [obm-l] RE: [obm-l] Re: [obm-l] Teoria dos números
>
> 7^x modulo 9 dá 1,7 e 4 e 3^y dá quase sempre 0
> O que interessa  para 7^x modulo 9 é 4,o que ocorre apenas quando x é da
> forma 3.k + 2
> Como x tambem é ímpar,só pode ser da forma 6.n + 5,mas...
>
> --
> Date: Thu, 29 Aug 2013 09:21:24 -0300
> Subject: [obm-l] Re: [obm-l] Teoria dos números
> From: ralp...@gmail.com
> To: obm-l@mat.puc-rio.br
>
> Tente agora modulo 9.
> On Aug 28, 2013 9:50 PM, "marcone augusto araújo borges" <
> marconeborge...@hotmail.com> wrote:
>
> Eu já postei a questão aqui,mas infelizmente não obtive resposta.
> Sei que vão aparecendo outras questões interessantes e por isso peço
> licença para reapresentá-la
>
> Determine todos os inteiros positivos x,y tais que 7^x - 3^y = 4
>
> Claro que x = 1 e y = 1 satisfaz(desconfio que seja a unica solução)
> Eu só consegui concluir que x e y são ímpares,analisando módulo 4.
> Desde já agradeço.
>
>
> --
> Esta mensagem foi verificada pelo sistema de antivírus e
> acredita-se estar livre de perigo.
>
>
> --
> Esta mensagem foi verificada pelo sistema de antivírus e
> acredita-se estar livre de perigo.
>
> --
> Esta mensagem foi verificada pelo sistema de antivírus e
> acredita-se estar livre de perigo.
>
>
>
> --
> Esta mensagem foi verificada pelo sistema de antivírus e
> acredita-se estar livre de perigo.
>

-- 
Esta mensagem foi verificada pelo sistema de antivírus e
 acredita-se estar livre de perigo.



[obm-l] Re: [obm-l] RE: [obm-l] Re: [obm-l] Teoria dos números

2013-08-30 Por tôpico Ralph Teixeira
Ah, droga, bobeei. Nao ajudou tanto quanto eu achava... :-( :-(
On Aug 29, 2013 12:23 PM, "marcone augusto araújo borges" <
marconeborge...@hotmail.com> wrote:

> 7^x modulo 9 dá 1,7 e 4 e 3^y dá quase sempre 0
> O que interessa  para 7^x modulo 9 é 4,o que ocorre apenas quando x é da
> forma 3.k + 2
> Como x tambem é ímpar,só pode ser da forma 6.n + 5,mas...
>
> --
> Date: Thu, 29 Aug 2013 09:21:24 -0300
> Subject: [obm-l] Re: [obm-l] Teoria dos números
> From: ralp...@gmail.com
> To: obm-l@mat.puc-rio.br
>
> Tente agora modulo 9.
> On Aug 28, 2013 9:50 PM, "marcone augusto araújo borges" <
> marconeborge...@hotmail.com> wrote:
>
> Eu já postei a questão aqui,mas infelizmente não obtive resposta.
> Sei que vão aparecendo outras questões interessantes e por isso peço
> licença para reapresentá-la
>
> Determine todos os inteiros positivos x,y tais que 7^x - 3^y = 4
>
> Claro que x = 1 e y = 1 satisfaz(desconfio que seja a unica solução)
> Eu só consegui concluir que x e y são ímpares,analisando módulo 4.
> Desde já agradeço.
>
>
> --
> Esta mensagem foi verificada pelo sistema de antivírus e
> acredita-se estar livre de perigo.
>
>
> --
> Esta mensagem foi verificada pelo sistema de antivírus e
> acredita-se estar livre de perigo.
>
> --
> Esta mensagem foi verificada pelo sistema de antivírus e
> acredita-se estar livre de perigo.
>

-- 
Esta mensagem foi verificada pelo sistema de antivírus e
 acredita-se estar livre de perigo.



[obm-l] Re: [obm-l] Teoria dos números

2013-08-29 Por tôpico Ralph Teixeira
Tente agora modulo 9.
On Aug 28, 2013 9:50 PM, "marcone augusto araújo borges" <
marconeborge...@hotmail.com> wrote:

> Eu já postei a questão aqui,mas infelizmente não obtive resposta.
> Sei que vão aparecendo outras questões interessantes e por isso peço
> licença para reapresentá-la
>
> Determine todos os inteiros positivos x,y tais que 7^x - 3^y = 4
>
> Claro que x = 1 e y = 1 satisfaz(desconfio que seja a unica solução)
> Eu só consegui concluir que x e y são ímpares,analisando módulo 4.
> Desde já agradeço.
>
>
> --
> Esta mensagem foi verificada pelo sistema de antivírus e
> acredita-se estar livre de perigo.
>

-- 
Esta mensagem foi verificada pelo sistema de antivírus e
 acredita-se estar livre de perigo.



Re: [obm-l] O Paradoxo da Flecha - Off Topic

2013-08-26 Por tôpico Ralph Teixeira
2013/8/26 Bernardo Freitas Paulo da Costa :
> Já temos um problema aqui. Ao considerar que "num instante, a flecha
> está parada". Nada disso, ela TEM velocidade, mas observar o movimento
> só faz sentido AO LONGO do tempo, não numa fotografia, e é por isso
> que você não vê a flecha se mexer nesta dita fotografia.

zero!

Oi, Paulo Cesar, eu ia responder... mas, po, o Bernardo expressou
exatamente o que eu penso do assunto nesse paragrafo ai em cima, de
maneira mais clara do que eu seria capaz de dizer. :) :)

Abraco,
Ralph

-- 
Esta mensagem foi verificada pelo sistema de antivírus e
 acredita-se estar livre de perigo.


=
Instruções para entrar na lista, sair da lista e usar a lista em
http://www.mat.puc-rio.br/~obmlistas/obm-l.html
=


Re: [obm-l] geometria analitica, frase do Elon...

2013-08-26 Por tôpico Ralph Teixeira
Dica: voce pode pensar que r^2=x^2+y^2. Entao desenhe no plano rz a
regiao 1+z^2<=r^2<=5 (apenas para r>=0). Como a regiao U nao depende
especificamente de x ou y, mas apenas de r=sqrt(x^2+y^2), a regiao U
serah a regiao que voce desenhou no plano rz, rodada em torno do eixo
z.

Agora tem todo o trabalho de decifrar a regiao 1+z^2<=r^2<=5 -- eh toda sua!

Abraco,
  Ralph

P.S.: Eu descobri um dia desses que, se voce entrar algo como
z=sqrt(x^2+y^2-1) no Google, pelo menos aqui no meu Chrome, ele faz um
grafico 3D da superficie! Legal!

2013/8/25 Hermann :
> No excelente curso dado no IMPA para professores do ensino médio, assisti ao
> de 1996, pelos professores, Elon, falecido Morgado, Wagner e PC, foi falado
> a seguinte frase que depois acabou virando o livro: exame de textos Análise
> de livros para o ensino médio do prof Elon.
>
> o teor da  frase dizia mais ou menos  o seguinte: que o professor na maioria
> das vezes tem como tutor (unicamente) péssimos livros no Brasil...
>
> Meus amigos esse professor aqui, já por diversas vezes foi ajudado nesse
> fórum, como não canso de agradecer.
>
> Só que no momento to precisando muito da ajuda dos amigos, eu preciso
> descobrir um livro que entre outras coisas me ensine a encontrar a fronteira
> do sólido
>
> U={(x,y,z) E R^3/ 1+z^2<=x^2+y^2<=5}
>
> eu lembro que no livro de cálculo do SHEIK tinha isso, mas não to achando
> ele nas minhas bagunças, alguém confirma ou me indica um outro.
>
> Abraços
> Hermann
> ps: me perdoem ter mais ou menos repetido a pergunta, to precisando mesmo.
>
> --
> Esta mensagem foi verificada pelo sistema de antivírus e
> acredita-se estar livre de perigo.

-- 
Esta mensagem foi verificada pelo sistema de antivírus e
 acredita-se estar livre de perigo.


=
Instruções para entrar na lista, sair da lista e usar a lista em
http://www.mat.puc-rio.br/~obmlistas/obm-l.html
=


[obm-l] Re: [obm-l] Questão simples(equações polinomiais?)

2013-08-26 Por tôpico Ralph Teixeira
Eu tenho outra solução também na "marra", mas de outro tipo: se você me dá
uma questão qualquer deste tipo com polinômios não exageradamente
horrorosos, o que eu tento fazer é dividir um polinômio pelo outro:

P(x)=x^8-7x^4+1=Q(x)(x^2+x-1)+R(x)

Algumas contas depois, temos R(x). Botando x=a=raiz de x^2+x-1, vem que
P(a)=R(a), que seria a resposta.

No seu caso, fazendo isso eu descobriria (surpreso!) que x^8-7x^4+1 pode
ser fatorado como (x^2+x-1)(x^6-x^5+2x^4-3x^3-2x^2-x-1), então R(a)=0.

--//--

Outra ideia mais mágica é partir para a fatoração mesmo completando
quadrados de maneira esperta:

x^8-7x^4+1=(x^8+2x^4+1-9x^4)=(x^4+1)^2-(3x^2)^2=(x^4-3x^2+1)(x^4+3x^2+1)

e de novo

x^4-3x^2+1=(x^4-2x^2+1)-x^2=(x^2-1)^2-x^2=(x^2+x-1)(x^2-x-1)

Então, puxa!, um é divisível pelo outro, a resposta é 0... Mas estou
roubando: só suspeitei que isso ia dar certo depois que eu vi a sua
resposta... :) :)

Abraço, Ralph.
On Aug 26, 2013 11:44 AM, "marcone augusto araújo borges" <
marconeborge...@hotmail.com> wrote:

> Seja x um número que satisfaz a equação x^2 + x - 1 = 0,determine
> o valor da expressão x^8 - 7x^4 + 1
>
> Eu fiz ´´na marra´´.
> x^2  = 1 - x (1)
> Calculei x^4 e x^8(elevando ao quadrado)
> usei também x^3 = x -x^2 de (1)
> Encontrei zero como resposta
> Dever existir um modo mais interessante.
> Agradeço por um esclarecimento.
>
>
>
> --
> Esta mensagem foi verificada pelo sistema de antivírus e
> acredita-se estar livre de perigo.
>

-- 
Esta mensagem foi verificada pelo sistema de antivírus e
 acredita-se estar livre de perigo.



Re: [obm-l] Como que faz??

2013-08-25 Por tôpico Ralph Teixeira
Vamos reduzir um pouco as coisas... Botando -x no lugar de x,
descobrimos que f(x)^2=f(-x)^2, isto eh, (f(x)+f(-x))(f(x)-f(-x))=0.
Agora, ambas as expressoes em parenteses sao polinomios, entao o unico
jeito deste produto ser identicamente nulo eh se um dos termos o for.

Em suma: f(x) eh par ou f(x) eh impar. Como f tem grau 13 (ao inves de
"menor ou igual a 13"), f(x) tem que ser impar. Isto nao mata o
problema, mas reduz um bocado o trabalho...

Abraco,
  Ralph

2013/8/25 Eduardo Wilner :
> De uma ou de outra forma dá "bastante equações" mesmo! (seriam quatorze?)
>
>
> [ ]'s
>
> 
> De: Bernardo Freitas Paulo da Costa 
> Para: obm-l@mat.puc-rio.br
> Enviadas: Sexta-feira, 23 de Agosto de 2013 19:45
> Assunto: Re: [obm-l] Como que faz??
>
> 2013/8/23  :
>> Olá , alguns alunos do ensino médio da instituição onde trabalho me deram
>> alguns problemas do site https://brilliant.org/
>>
>> PROBLEMA 1: Dada uma função f:R->R tal que f(2x^2 -1)=2(f(x))^2 -1 e f(x)
>> é
>> um polinômio de grau 13, sendo assim determine o coeficiente de x^5 de
>> f(x).
> Bom, f(x) é dada por 14 coeficientes a_i. A equação que você tem dá um
> monte de condições sobre estes coeficientes: para cada x, tem uma
> condição.
>
> Por exemplo, x = 0 dá f(-1) = 2f(0)^2 - 1, f(-1) é uma soma alternada,
> f(0)^2 é apenas (a_0)^2. Escolhendo um monte de x's, você obtém
> bastante equações, e resolve.
>
> Dá pra fazer (um pouco) menos grotesco, porque você pode escrever (a_0
> + a_1 * x + a_2 * x^2 +  + a_13 * x^13)^2, separando por grau.
> Duas funções polinomiais em R são iguais se e somente se os
> coeficientes forem iguais. Assim, identifique os coeficientes de ambos
> os lados, e parta pro abraço.
>
> Pra entender porque isso sempre dá certo, vale a pena lembrar de
> Álgebra Linear (também se ensina um pouco desse tipo de intuição em
> cursos de "Geometria Algébrica classica").
>
> Abraços
> --
> Bernardo Freitas Paulo da Costa
>
> --
> Esta mensagem foi verificada pelo sistema de antivírus e
> acredita-se estar livre de perigo.
>
>
> =
> Instruções para entrar na lista, sair da lista e usar a lista em
> http://www.mat.puc-rio.br/~obmlistas/obm-l.html
> =
>
>
>
> --
> Esta mensagem foi verificada pelo sistema de antivírus e
> acredita-se estar livre de perigo.

-- 
Esta mensagem foi verificada pelo sistema de antivírus e
 acredita-se estar livre de perigo.


=
Instruções para entrar na lista, sair da lista e usar a lista em
http://www.mat.puc-rio.br/~obmlistas/obm-l.html
=


Re: [obm-l] Monty Hall - Bayes

2013-08-23 Por tôpico Ralph Teixeira
Oi, Felipe.

As seguintes propriedades sao 100% equivalentes (bom, a menos que voce
ponha algum evento de probabilidade 0):
Pr(A inter B)=Pr(A).Pr(B)
Pr(A|B)=Pr(A)
Pr(B|A)=Pr(B)
No caso em que alguma delas (e, portanto, todas) vale(m), dizemos que
os eventos A e B sao independentes.

Entao, isto eh para dizer: nao lembro exatamente mais qual eh o
enunciado do problema... Mas noto que voce soh pode usar que Pr(A
inter B)=Pr(A).Pr(B) se voce **supuser** que A e B sao
independentes... o que eh o mesmo que supor que Pr(A|B)=Pr(A)... o que
nao eh nada obvio e depende do enunciado exato do problema!

Abraco,
Ralph

2013/8/23 luiz silva :
> Pessoal,
>
> Voltando ao assunto do problema de Monty Hall, e o uso de Bayes para sua
> solução :
>
> Calcular a probabilidade do "Juiz" Ganhar, AGORA que sei q a
> carta virada não é premiada :
> P(A) - Probabilidade do Juiz Ganhar;
> P(B) - " " da carta virada não ser premiada;
> P(A/B) - " " do Juiz Ganhar, AGORA q sei que carta q a carta virada não é
> premiada;
> P(A inter B)- Probabilidade do Juiz Ganhar e da carta virada não ser
> premiada;
> P(A) = 2/3
> P(B) = 2/3
> P(A inter B) = 2/3 * 2/3 = 4/9
> P(A/B) = (4/9)/(2/3) = 2/3
>
> Ou seja, a probabilidade do Juiz ganhar continua sendo 2/3, mesmo após a
> conferência. A melhor decisão é trocar a carta com o Juiz
>
> Caso o juiz tivesse descartado uma de suas cartas, sem nenhum dos dois saber
> se a descartada não era premiada, então , nesse caso, a probabilidade de
> ganho dos dois seria 50%.
>
> Abs
> Felipe
>
> --
> Esta mensagem foi verificada pelo sistema de antivírus e
> acredita-se estar livre de perigo.

-- 
Esta mensagem foi verificada pelo sistema de antivírus e
 acredita-se estar livre de perigo.


=
Instruções para entrar na lista, sair da lista e usar a lista em
http://www.mat.puc-rio.br/~obmlistas/obm-l.html
=


[obm-l] Re: [obm-l] Projeto de Desafios Matemáticos - CIENTEC 2014

2013-08-21 Por tôpico Ralph Teixeira
Algumas ideias:

Site com jogos de lógica, especialmente "griddlers": griddlers.net
Se você tiver um Ipad, Iphone ou algo assim, tem um app grátis muito legal
com nome bem despretensioso (Logic Games - Time Killers):
https://itunes.apple.com/us/app/84-logic-games-time-killers/id500243153?mt=8
Bom, e se você quiser algo mais matematicamente profundo, tem o livro do
Berlekamp/Conway em 4 volumes (não é grátis):
http://www.amazon.com/Winning-Ways-Your-Mathematical-Plays/dp/1568811306

Abraço,
 Ralph







2013/8/21 Maikel Andril Marcelino 

> Pessoal, gostaria de ideias de jogos, problemas, enigmas, poisestou
> querendo fazer um projeto e qnto maiis jogos melhor! minhas ideias ate
> agora foram:
>
> sudoku
> enigmas
> riddles
> criptoaritmetica
> cubo magico
> problemas de logica
> jogos com palitos
>
> att,
> maikel andril marcelino
>
> --
> Esta mensagem foi verificada pelo sistema de antivírus e
> acredita-se estar livre de perigo.
>

-- 
Esta mensagem foi verificada pelo sistema de antivírus e
 acredita-se estar livre de perigo.



Re: [obm-l] Problema com o enunciado?(inteiros)

2013-08-20 Por tôpico Ralph Teixeira
507=3*13*13. Tente x=2.
On Aug 20, 2013 3:26 PM, "marcone augusto araújo borges" <
marconeborge...@hotmail.com> wrote:

> Calcule o valor de 3x^2.y^2 tal que x e y são inteiros satisfazendo a
> equação
> y^2 + 3x^2.y^2 = 30x^2 + 517
>
> Eu encontrei y^2 = 10 +507/(3x^2 + 1)
> 3x^2 + 1 deve ser  divisor de 507
> Se eu não estiver enganado,3x^2 +1,com x inteiro,
> só é divisor de 507 se x = 0
> Mas para x = 0,y não é inteiro
> Errei ou há problema no enunciado?
>
> --
> Esta mensagem foi verificada pelo sistema de antivírus e
> acredita-se estar livre de perigo.
>

-- 
Esta mensagem foi verificada pelo sistema de antivírus e
 acredita-se estar livre de perigo.



[obm-l] Re: [obm-l] Re: [obm-l] Re: [obm-l] Re: [obm-l] Uma variação do Problema de Monty Hall

2013-08-14 Por tôpico Ralph Teixeira
Ah, Luiz, mas ai que eu discordo -- o ato de conferencia te traz nova
informacao. Isto altera SIM as probabilidades.

Vamos ao jogo com 3 cartas numeradas de 1 a 3. Eu escolho 1, voce fica com
as outras 2. Quem tiver a carta 3 ganha.

Quando o jogo comeca, sim, a probabilidade de eu ganhar eh apenas 1/3.

Agora, voce escolhe uma de suas cartas aleatoriamente e me mostra. Digamos
que voce mostrou a carta 2: as probabilidades mudam! Nova informacao induz
novas probabilidades! Agora as probabilidades sao 50% para cada um.

Mas note que isto depende IMENSAMENTE de COMO voce escolhe a carta. Supus
ali em cima que voce escolheu sem olhar. Se voce olhou as suas cartas e
escolheu a menor, bom, entao sua chance de ganhar eh 100% (se a menor eh 2,
certamente voce tem a 3). Se voce olhou suas cartas e escolheu a maior,
entao eh 0% para voce e 100% para mim.

Em suma -- no problema dos bodes, eh essencial saber COMO o cara abriu a
porta do bode. Se ele seguiu a regra "nunca abra o carro", eh uma coisa. Se
ele "abriu aleatoriamente e apareceu um bode", eh outra.

Abraco,
Ralph

P.S.: Outro experimento que gosto de fazer com meus alunos: eu jogo 2
moedas de forma que o Joao consiga ver uma delas. Ai eu pergunto pra Maria
que estah lah do outro lado da sala "Qual a probabilidade de ambas serem
"Cara"?". Maria responde 25%; entao eu pergunto pro Joao, que estah vendo
uma cara Ele pensa um pouco e diz "50%" porque na pratica ele soh estah
se perguntando sobre a outra moeda. Entao eu olho as duas e digo "A
probabilidade eh 0%."... Foi a mesma pergunta 3 vezes, com 3 respostas
diferentes, quem estah errado? Resposta: NINGUEM estah errado -- o fato eh
que probabilidades DEPENDEM da informacao que voce tem!


2013/8/14 luiz silva 

> Pense da seginte forma :
>
> Após a escolha inicial, um tem 1/3 de ganhar ou outro2/3. Após isso, vem a
> conferência (cada um olha as suas cartas para verificar quem ganhou).
> Imagine que a pessoa com 2 cartas faça a sua conferência antes da que tem 1
> carta. Ela vira a primeira carta e ve que não é a premiada. As suas chances
> continuam sendo 2/3 de ganhar, pois o ato de verificação (ver se a carta
> premiada está ou não com vc) não altera as probabilidades iniciais, pois o
> espaço amostral continua o mesmo e o as escolhas iniciais também. Agora, se
> após a primeria verificação houver o embaralhjamento e nova escolha, aí
> cada um terá 50% de cahnces, pois houve alteração do espaço amostral e das
> escolhas efetuadas.
>
> Reumo :A verificação não altera a probabilidade; virando-se as duas cartas
> em sequencia ou ao mesmo tempo, a probabilidade de ganho continua sendo 2/3
> para quem escolheu duas cartas e 1/3 para quem escolheu uma carta. Assim,
> trocar a carta é probabilisticamente vantajoso para quem esolheu somente
> uma carta.
>
> Dica : imagine a mesma situação com 100 cartas; uma pessoa com 1 e outra
> com 99; se for conferir uma a uma, vai levar um tempo, mas a chance da
> carta vencedora estar em uma das 99 é 99%. Então, a última carta a ser
> conferida continua dando 99% de chances de ganho para quem escolheu as 99
> cartas.
>
> Abs
> Felipe
>   *De:* Bob Roy 
> *Para:* obm-l@mat.puc-rio.br
> *Enviadas:* Quarta-feira, 14 de Agosto de 2013 10:03
> *Assunto:* [obm-l] Re: [obm-l] Re: [obm-l] Uma variação do Problema de
> Monty Hall
>
>  Olá ,
>
> desculpem, mas fiquei confuso; então quer dizer que ( nas novas condições)
> trocando ou não , ele fica com a chance de ganhar igual a 1/3; é isso ?
> abs
> Bob
>
>
> Em 13 de agosto de 2013 20:56, Ralph Teixeira escreveu:
>
> Bom, no problema original eh importante ressaltar as regras: o
> apresentador nunca abre a porta do espectador nem a porta do carro. Nessa
> nova versao, ele nao tem como seguir a segunda regra. Na nova versao, se
> ele abriu um bode, a probabilidade de cada porta eh 1/2.
> Muito vagamente, funciona assim:
> No problema classico, de cada 900 shows, o apresentador deixa um bode na
> outra porta fechada 300 vezes (sempre que o espectador acerta o carro de
> primeira) e deixa um carro na outra porta fechada 600 vezes.
> Agora, na sua nova versao, de cada 900 shows, o apresentador deixa um bode
> na outra porta fechada 300 vezes e deixa o carro na outra porta fechada 300
> vezes. Nas outras 300, ele abre (bom, ou elimina) o carro!
> Abraco, Ralph.
>  On Aug 13, 2013 7:16 PM, "Jorge Paulino"  wrote:
>
> Lembrando que no problema temos 3 portas, com 2 bodes
> atrás de duas delas e um carro atrás de outra.
> Uma expectador escolhe uma porta (querendo ganhar o carro).
> O apresentador, sabendo o que está atrás delas, abre uma outra,
> mostra um bode e pergunta se o expectador quer continuar com a porta
> escolhida ou mudar para a terceira porta.
> Sabem

[obm-l] Re: [obm-l] Re: [obm-l] Re: [obm-l] Uma variação do Problema de Monty Hall

2013-08-14 Por tôpico Ralph Teixeira
Eh isso mesmo!

PORTA ELIMINADA, MAS NAO ABERTA: Ficando, voce tem 1/3 de chance de ganhar;
trocando, voce tem 1/3 de chance de ganhar. Se voce quiser procurar o
"outro 1/3", esta eh a chance de ele ter eliminado o carro, e ai nao
adianta ficar nem trocar.

PORTA ABERTA: Se ele ABRE a porta eliminada, ganhamos mais um pedacinho de
informacao, entao eh um pouquinho diferente,. Temos dois casos:
i) Se ele ABRE a porta com o carro, entao tanto ficando quanto trocando sua
chance eh zero. Jah era.
ii) Se ele ABRE a porta com um bode (ufa! sorte!), entao tanto ficando
quanto trocando sua chance eh 1/2=50%.

Abraco,
Ralph

P.S.: Do jeito que eu penso, probabilidades dependem TOTALMENTE da
informacao que voce tem. De fato, eu diria que "probabilidade" eh sobre
INFORMACAO e nada mais! Entao nao eh surpreendente que haja 3 valores
diferentes para probabilidades nos ultimos paragrafos -- cada valor eh
baseado numa certa informacao que voce tem (ou nao tem).

2013/8/14 Bob Roy 

> Olá ,
>
> desculpem, mas fiquei confuso; então quer dizer que ( nas novas condições)
> trocando ou não , ele fica com a chance de ganhar igual a 1/3; é isso ?
> abs
> Bob
>
>
> Em 13 de agosto de 2013 20:56, Ralph Teixeira escreveu:
>
>> Bom, no problema original eh importante ressaltar as regras: o
>> apresentador nunca abre a porta do espectador nem a porta do carro. Nessa
>> nova versao, ele nao tem como seguir a segunda regra. Na nova versao, se
>> ele abriu um bode, a probabilidade de cada porta eh 1/2.
>>
>> Muito vagamente, funciona assim:
>>
>> No problema classico, de cada 900 shows, o apresentador deixa um bode na
>> outra porta fechada 300 vezes (sempre que o espectador acerta o carro de
>> primeira) e deixa um carro na outra porta fechada 600 vezes.
>>
>> Agora, na sua nova versao, de cada 900 shows, o apresentador deixa um
>> bode na outra porta fechada 300 vezes e deixa o carro na outra porta
>> fechada 300 vezes. Nas outras 300, ele abre (bom, ou elimina) o carro!
>>
>> Abraco, Ralph.
>> On Aug 13, 2013 7:16 PM, "Jorge Paulino"  wrote:
>>
>>> Lembrando que no problema temos 3 portas, com 2 bodes
>>> atrás de duas delas e um carro atrás de outra.
>>> Uma expectador escolhe uma porta (querendo ganhar o carro).
>>> O apresentador, sabendo o que está atrás delas, abre uma outra,
>>> mostra um bode e pergunta se o expectador quer continuar com a porta
>>> escolhida ou mudar para a terceira porta.
>>> Sabemos que vale à pena mudar, pois a probabilidade
>>> desta nova porta é de 2/3 contra 1/3 da escolhida no início.
>>>
>>> Bem, e se o apresentador não souber em que porta está o carro e,
>>> após escolhida a primeira porta, ele apenas elimina (sem abrir) uma
>>> porta.
>>> Como fica a probabilidade de cada uma das duas portas neste momento?
>>> Vale a pena trocar também?
>>>
>>> Grato,
>>>
>>> Jorge
>>>
>>> --
>>> Esta mensagem foi verificada pelo sistema de antivírus e
>>> acredita-se estar livre de perigo.
>>>
>>> ==**==**
>>> =
>>> Instruções para entrar na lista, sair da lista e usar a lista em
>>> http://www.mat.puc-rio.br/~**obmlistas/obm-l.html<http://www.mat.puc-rio.br/~obmlistas/obm-l.html>
>>> ==**==**
>>> =
>>>
>>
>> --
>> Esta mensagem foi verificada pelo sistema de antivírus e
>> acredita-se estar livre de perigo.
>>
>
>
> --
> Esta mensagem foi verificada pelo sistema de antivírus e
> acredita-se estar livre de perigo.

-- 
Esta mensagem foi verificada pelo sistema de antivírus e
 acredita-se estar livre de perigo.



[obm-l] Re: [obm-l] Dízimas periódicas

2013-08-13 Por tôpico Ralph Teixeira
Oi, Luiz.

Argumento interessante? Que tal...

http://www.mat.puc-rio.br/~nicolau/olimp/obm-l.24/msg00074.html

Abraco, Ralph
On Aug 13, 2013 1:25 PM, "Luiz Antonio Rodrigues" 
wrote:

> Olá, pessoal!
> Tudo bem?
> Na semana passada eu propus a seguinte discussão para os meus alunos: se
> 0,111... + 0,888... = 0,999... então 1/9 + 8/9 = 0,999...   o que implica
> que 1= 0,999...
> Consegui despertar a curiosidade dos alunos, mas muitos deles não
> aceitaram o que acabamos concluindo. Alguém poderia me ajudar com algum
> argumento interessante sobre a estranha conclusão?
> Obrigado e um abraço!
> Luiz
>
> --
> Esta mensagem foi verificada pelo sistema de antivírus e
> acredita-se estar livre de perigo.

-- 
Esta mensagem foi verificada pelo sistema de antivírus e
 acredita-se estar livre de perigo.



[obm-l] Re: [obm-l] Uma variação do Problema de Monty Hall

2013-08-13 Por tôpico Ralph Teixeira
Bom, no problema original eh importante ressaltar as regras: o apresentador
nunca abre a porta do espectador nem a porta do carro. Nessa nova versao,
ele nao tem como seguir a segunda regra. Na nova versao, se ele abriu um
bode, a probabilidade de cada porta eh 1/2.

Muito vagamente, funciona assim:

No problema classico, de cada 900 shows, o apresentador deixa um bode na
outra porta fechada 300 vezes (sempre que o espectador acerta o carro de
primeira) e deixa um carro na outra porta fechada 600 vezes.

Agora, na sua nova versao, de cada 900 shows, o apresentador deixa um bode
na outra porta fechada 300 vezes e deixa o carro na outra porta fechada 300
vezes. Nas outras 300, ele abre (bom, ou elimina) o carro!

Abraco, Ralph.
On Aug 13, 2013 7:16 PM, "Jorge Paulino"  wrote:

> Lembrando que no problema temos 3 portas, com 2 bodes
> atrás de duas delas e um carro atrás de outra.
> Uma expectador escolhe uma porta (querendo ganhar o carro).
> O apresentador, sabendo o que está atrás delas, abre uma outra,
> mostra um bode e pergunta se o expectador quer continuar com a porta
> escolhida ou mudar para a terceira porta.
> Sabemos que vale à pena mudar, pois a probabilidade
> desta nova porta é de 2/3 contra 1/3 da escolhida no início.
>
> Bem, e se o apresentador não souber em que porta está o carro e,
> após escolhida a primeira porta, ele apenas elimina (sem abrir) uma porta.
> Como fica a probabilidade de cada uma das duas portas neste momento?
> Vale a pena trocar também?
>
> Grato,
>
> Jorge
>
> --
> Esta mensagem foi verificada pelo sistema de antivírus e
> acredita-se estar livre de perigo.
>
> ==**==**
> =
> Instruções para entrar na lista, sair da lista e usar a lista em
> http://www.mat.puc-rio.br/~**obmlistas/obm-l.html
> ==**==**
> =
>

-- 
Esta mensagem foi verificada pelo sistema de antivírus e
 acredita-se estar livre de perigo.



[obm-l] Re: [obm-l] Re: [obm-l] Re: [obm-l] equação do 2 grau métodos de sol

2013-08-05 Por tôpico Ralph Teixeira
Uma coisa que eu aprendi eh que quase nenhum pais alem do Brasil chama
esta formula de "Baskara" -- pelo menos nas minhas turmas
internacionais, ninguem reconhece o nome, nem os indianos chamam
assim... Acho que eh "formula quadratica" em varias linguas, mas
fiquem aa vontade para me desmentir -- como eh no resto da America
Latina?

Mas, se eu entendi direito, nao eh esse o problema, neh? O seu aluno
fazia realmente por algum outro metodo, eh isso? Nao consigo imaginar
algo que seja tao geral quanto a formula usual, e que nao seja bem
parecida com ela... Alguem do Peru vai ter que responder... :)

Abraco,
 Ralph

2013/8/5 Hermann :
> Eu acho que vc completou o quadrado e isso é báskara, agradeço, mas o que eu
> desejava saber é que método é ensinado no Peru.
> Diferente de báskara.
>
> - Original Message -
> From: Esdras Muniz
> To: obm-l@mat.puc-rio.br
> Sent: Monday, August 05, 2013 1:01 PM
> Subject: [obm-l] Re: [obm-l] equação do 2 grau métodos de sol
>
> x² - 3x + 5 = 0
> x² - (3x/2) + (3/2)² = -5 + (3/2)²
> (x - 3/2)² = (3/2)² - 5
> 
>
>
> Em 5 de agosto de 2013 12:06, Hermann  escreveu:
>>
>> Dei aula para um peruano que não usava báskara, mas não tive tempo na
>> época.
>>
>> Alguém aqui saberia me explicar outros métodos de se obter a solução da
>> equação (sem báskara, sem S e P)
>>
>>  ax^2+bx+c=0
>>
>> abraços
>>
>> Hermann
>>
>> --
>> Esta mensagem foi verificada pelo sistema de antivírus e
>> acredita-se estar livre de perigo.
>
>
>
>
> --
> Esdras Muniz Mota
> Graduando em Matemática Bacharelado
> Universidade Federal do Ceará
>
> "Se algum dia ele recuou, foi para dar um grande salto"
>
> --
> Esta mensagem foi verificada pelo sistema de antivírus e
> acredita-se estar livre de perigo.
>
>
> --
> Esta mensagem foi verificada pelo sistema de antivírus e
> acredita-se estar livre de perigo.

-- 
Esta mensagem foi verificada pelo sistema de antivírus e
 acredita-se estar livre de perigo.


=
Instruções para entrar na lista, sair da lista e usar a lista em
http://www.mat.puc-rio.br/~obmlistas/obm-l.html
=


[obm-l] Re: [obm-l] solução sistema linear dúvida

2013-07-22 Por tôpico Ralph Teixeira
Se o problema era RESOLVER o sistema, tah certo: o conjunto solucao eh
o conjunto de todos os pontos da forma que voce botou, onde z eh um
real qualquer. Note-se que nao eh a UNICA maneira de escrever a
resposta, mas tah certo.

2013/7/22 Hermann :
> Meus amigos recorro a vocês mais uma vez, perdoem-me.
>
> Dado o sistema linear x+y+z=1 e x-y=2
>
> fiz de propósito o z como parâmetro e cheguei a seguinte solução (3/2 - z/2,
> -1/2 - z/2, z)
>
> tem algum erro? Eu acho que não, mas a insegurança me mata.
>
> obrigado
> abraços
> Hermann
>
>
>
> --
> Esta mensagem foi verificada pelo sistema de antivírus e
> acredita-se estar livre de perigo.

-- 
Esta mensagem foi verificada pelo sistema de antivírus e
 acredita-se estar livre de perigo.


=
Instruções para entrar na lista, sair da lista e usar a lista em
http://www.mat.puc-rio.br/~obmlistas/obm-l.html
=


Re: [obm-l] Primos

2013-07-12 Por tôpico Ralph Teixeira
Pois eh, fico com o PS do Ponce, que demonstra o seguinte "Teorema
Generalizado":

"Se A e B sao dois BLAHS consecutivos, entao A+B nao pode ser o dobro de um
BLAH."

2013/7/12 Rogerio Ponce 

> Ola' Marcos,
> todo primo (maior que 3) e' da forma 6k+1 ou 6k-1, assim como todo impar
> e' da forma 2k+1.
>
> []'s
> Rogerio Ponce
>
> PS: Dizer que a soma de dois primos consecutivos, A e B, seria o dobro de
> um terceiro primo, C,
> e' o mesmo que dizer que C e' a media entre A e B, que necessariamente se
> situa entre A e B (inclusive).
> Como A e B sao consecutivos, C nao pode estar entre eles.
>
> []'s,
> Rogerio Ponce
>
>
> 2013/7/12 Marcos Martinelli 
>
>> Acho que não existe uma fórmula fechada para os primos.
>>
>> Acho que tentamos encontrá-la há um bom tempo... mas sem sucesso, apesar
>> de inúmeras outras portas que foram abertas com a teoria analítica dos
>> números.
>>
>> Em sexta-feira, 12 de julho de 2013, Nehab escreveu:
>>
>>>  Oi, Marcone,
>>>
>>> Números primos são da forma 6k - 1 ou 6k + 1.
>>> Imediato...
>>>
>>> Nehab
>>>
>>> On 11/07/2013 23:16, marcone augusto araújo borges wrote:
>>>
>>> Mostre que a soma de dois primos consecutivos nunca é  o dobro de um
>>> primo
>>>Peço ajuda.
>>>
>>> --
>>> Esta mensagem foi verificada pelo sistema de antivírus e
>>> acredita-se estar livre de perigo.
>>>
>>>
>>>
>>>
>>> --
>>> Esta mensagem foi verificada pelo sistema de antivírus e
>>> acredita-se estar livre de perigo.
>>>
>>
>> --
>> Esta mensagem foi verificada pelo sistema de antivírus e
>> acredita-se estar livre de perigo.
>
>
>
> --
> Esta mensagem foi verificada pelo sistema de antivírus e
> acredita-se estar livre de perigo.
>

-- 
Esta mensagem foi verificada pelo sistema de antivírus e
 acredita-se estar livre de perigo.



[obm-l] Re: [obm-l] correção de enunciado(potencia de base irracional)

2013-06-16 Por tôpico Ralph Teixeira
Ideia: (8+raiz(65))^2012+(8-raiz(65))^2012 eh inteiro, e este segundo cara
aqui deve ser bem pequenino. Se voce conseguir mostrar que este segundo
cara eh menor que 10^(-1000)

Abraco,
  Ralph


2013/6/16 marcone augusto araújo borges 

> o numero é (8 +65^1\2)^2012
>
> --
> Esta mensagem foi verificada pelo sistema de antivírus e
> acredita-se estar livre de perigo.
>

-- 
Esta mensagem foi verificada pelo sistema de antivírus e
 acredita-se estar livre de perigo.



Re: [obm-l] x^n = y^n => x = y

2013-06-15 Por tôpico Ralph Teixeira
Sim, mas olha a primeira linha:

"Em primeiro lugar, como n eh impar, note que x^n tem o mesmo sinal de x.
Assim, se x^n e y^n tem o mesmo sinal, x e y devem ter o mesmo sinal."

Assim eu jah havia concluido que x e y tem o mesmo sinal. Entao a partir
dali soh analiso os 3 casos em que x e y tem o mesmo sinal, porque o caso
y<0

> x e y terem o mesmo sinal indica que o segundo termo tem todas as parcelas
> de mesmo sinal e não inclui o caso y<0 e x>0.
>
>
> Em 14 de junho de 2013 19:24, Felipe  escreveu:
>
> O segundo termo sempre positivo não se verifica...
>> N = 4:
>> x⁴-y⁴ = (x-y)(x³+x²y+xy²+y³)
>> Se |y| > |x|, y<0 e x>0
>>
>>
>> Em 14 de junho de 2013 18:52, luiz silva 
>> escreveu:
>>
>> Ola Ralph,
>>>
>>> O termo é sempre positivo, mas temos números negativos tb :
>>>
>>> Ex n=3 :
>>>
>>> x3-y3 = (x-y)(x2+xy+y2)
>>>
>>> Supondo y<0 e x>0; [y]>[x]
>>>
>>> Então : x2 e y2 positivos, como xy>> positivo. Claro que se ambos são positivos ou [x]>=[y] o mesmo se verifica.
>>>
>>> Abs
>>> Felipe
>>>
>>>   --
>>>  *De:* Ralph Teixeira 
>>> *Para:* obm-l@mat.puc-rio.br
>>> *Enviadas:* Sexta-feira, 14 de Junho de 2013 18:25
>>> *Assunto:* Re: [obm-l] x^n = y^n => x = y
>>>
>>> Tem um jeito de fazer que soh precisa de uma fatoracao esperta.
>>>
>>> Em primeiro lugar, como n eh impar, note que x^n tem o mesmo sinal de x.
>>> Assim, se x^n e y^n tem o mesmo sinal, x e y devem ter o mesmo sinal.
>>> Assim, ha apenas 3 casos a considerar:
>>>
>>> i) x=0 e y=0
>>> ii) x>0 e y>0
>>> iii) x<0 e y<0
>>>
>>> O caso (i) automaticamente implica que x=y. Vou fazer o caso (ii) (o
>>> (iii) eh analogo, trocando x e y por -x e -y).
>>>
>>> Entao, suponha que x e y sao positivos e que x^n=y^n. Entao:
>>> x^n-y^n=0 --> (x-y)(x^(n-1)+x^(n-2)y+x^(n-3)y^2+...+xy^(n-2)+y^(n-1))=0
>>>
>>> Agora o termo comprido soh tem numeros positivos, entao eh positivo.
>>> Assim, o unico jeito deste produto dar 0 eh se x-y=0, isto eh, x=y.
>>>
>>> Abraco,
>>>  Ralph
>>>
>>> 2013/6/14 Pedro Chaves 
>>>
>>> Caros Colegas,
>>>
>>> Como provar que a igualdade x^n = y^n implica x = y, sendo x e y números
>>> reais quaisquer e n natural ímpar?
>>> --
>>> Esta mensagem foi verificada pelo sistema de antivírus e
>>>  acredita-se estar livre de perigo.
>>>
>>>
>>> =
>>> Instruções para entrar na lista, sair da lista e usar a lista em
>>> http://www.mat.puc-rio.br/~obmlistas/obm-l.html
>>> =
>>>
>>>
>>>
>>> --
>>> Esta mensagem foi verificada pelo sistema de antivírus e
>>> acredita-se estar livre de perigo.
>>>
>>>
>>>
>>> --
>>> Esta mensagem foi verificada pelo sistema de antivírus e
>>> acredita-se estar livre de perigo.
>>>
>>
>>
>>
>> --
>> Felipe de Souza Araujo
>>
>
>
>
> --
> Felipe de Souza Araujo
>
> --
> Esta mensagem foi verificada pelo sistema de antivírus e
> acredita-se estar livre de perigo.
>

-- 
Esta mensagem foi verificada pelo sistema de antiv�rus e
 acredita-se estar livre de perigo.



Re: [obm-l] x^n = y^n => x = y

2013-06-14 Por tôpico Ralph Teixeira
Tem um jeito de fazer que soh precisa de uma fatoracao esperta.

Em primeiro lugar, como n eh impar, note que x^n tem o mesmo sinal de x.
Assim, se x^n e y^n tem o mesmo sinal, x e y devem ter o mesmo sinal.
Assim, ha apenas 3 casos a considerar:

i) x=0 e y=0
ii) x>0 e y>0
iii) x<0 e y<0

O caso (i) automaticamente implica que x=y. Vou fazer o caso (ii) (o (iii)
eh analogo, trocando x e y por -x e -y).

Entao, suponha que x e y sao positivos e que x^n=y^n. Entao:
x^n-y^n=0 --> (x-y)(x^(n-1)+x^(n-2)y+x^(n-3)y^2+...+xy^(n-2)+y^(n-1))=0

Agora o termo comprido soh tem numeros positivos, entao eh positivo. Assim,
o unico jeito deste produto dar 0 eh se x-y=0, isto eh, x=y.

Abraco,
 Ralph

2013/6/14 Pedro Chaves 

> Caros Colegas,
>
> Como provar que a igualdade x^n = y^n implica x = y, sendo x e y números
> reais quaisquer e n natural ímpar?
> --
> Esta mensagem foi verificada pelo sistema de antivírus e
>  acredita-se estar livre de perigo.
>
>
> =
> Instruções para entrar na lista, sair da lista e usar a lista em
> http://www.mat.puc-rio.br/~obmlistas/obm-l.html
> =
>

-- 
Esta mensagem foi verificada pelo sistema de antivírus e
 acredita-se estar livre de perigo.



[obm-l] Fwd: [obm-l] questão bacana(quase me tira o sono)

2013-06-13 Por tôpico Ralph Teixeira
-- Forwarded message --
From: Ralph Teixeira 
Date: 2013/6/13
Subject: Re: [obm-l] questão bacana(quase me tira o sono)
To: obm-l@mat.puc-rio.br


Que tal assim -- pense numa maneira de colocar os pesos como uma fila de
pesos (na ordem em que eles serao colocados) E TAMBEM um bando de post-its,
um pregado em cada peso, com as letras D ou E dizendo onde aquele peso vai.

Entao, seja F(n) o numero de maneiras de montar uma fila com os N pesos
2^0,...,2^(N-1) satisfazendo as condicoes do enunciado (isto eh, numa ordem
e escolhendo os post-its de forma que, ao colocar os pesos da fila nos
pratos o lado esquerdo nunca pese mais que o direito). Note que o numero de
maneiras de colocar os pesos 2^1,...,2^N na maneira do enunciado tambem eh
F(N) (afinal eu soh multipliquei todos os pesos por 2).

Para encontrar uma maneira de colocar os pesos 2^0, 2^1,...,2^N, voce vai
ter que fazer o seguinte:
i) Decida a ordem e o lado onde voce vai colocar 2^1, 2^2, ..., 2^N. Ha
F(N) maneiras de fazer isto.
ii) Agora voce vai ter que encaixar o 2^0 em alguma posicao -- sao N+1
posicoes, a saber "antes do 1o" ou "entre o 1o e o 2o" ou ... "em ultimo".
Mas, se voce colocar ele no inicio da fila, teria que ser com o post-it D
(direito); em qualquer outra das N posicoes, vale D ou E. Entao sao 2N+1
opcoes para encaixar o peso 2^0 na fila.

Assim, para cada 1 maneira de botar os N pesos 2^1, 2^2, ..., 2^N,
ha exatemente 2N+1 maneiras (distintas!) de botar os N+1 pesos 2^0,
2^1,..., 2^N.

Note que esta correspondencia eh biunivoca no seguinte sentido: se voce me
der uma maneira de botar os N+1 pesos, eu jogo fora o peso 2^0 e sigo a sua
ordem (e post-its) para colocar os N pesos 2^1, 2^2, ..., 2^N dum jeito
valido!

Entao F(N+1)=(2N+1).F(N). Como F(1)=1, vem F(2)=3, F(3)=3x5, F(4)=3x5x7,
etc..

Em suma, F(N)=1x3x5x7xx(2N-1) e F(101)=1x3x5x7x...x201 como o Lucas jah
tinha dito.

Abraco,
Ralph

2013/6/11 marcone augusto araújo borges 

> São dados uma balança de 2 pratos e os pesos 2^0,2^1,2^2,...2^100 gramas
> os pesos são colocados um a um de modo que o prato esquerdo nunca seja
> mais pesado que o prato direito.De quantos modos isso pode ser feito
>
> não consegui e agradeço a quem ajudar
>
> --
> Esta mensagem foi verificada pelo sistema de antivírus e
> acredita-se estar livre de perigo.
>

-- 
Esta mensagem foi verificada pelo sistema de antivírus e
 acredita-se estar livre de perigo.



Re: [obm-l] Duvidas

2013-06-03 Por tôpico Ralph Teixeira
Formalize usando indução finita.

i) Dados 3 pontos no plano, não colineares, é bem claro que são 3 retas
distintas.

ii) Suponha que a afirmação é verdadeira para N=k (isto é, k pontos no
plano não-colineares determinam pelo menos k retas distintas).
Tome k+1 pontos não-colineares. Escolha um deles, digamos P, e olhe para os
k restantes.
-- Se os k restantes forem colineares, não dá para usar a hipótese de
indução, mas não precisa -- o argumento do Guilherme mostra que temos k+1
retas distintas por estes pontos.
-- Senão, são k pontos não colineares; por hipótese de indução, eles
determinam pelo menos k retas distintas!
 Se ALGUMA das k retas não passar por P, pegue um ponto desta reta,
ligue-o a P e monte assim a k+1-ésima reta.
 Se as k retas passarem por P, putz grila, você é muito azarado! Mas
tudo bem -- pegue duas dessas retas (ok pois k>=2), pegue um ponto (dos k
"antigos") em cada uma e voilá!, temos uma nova reta ("nova" pois ela não
passa por P).

Obs.: "Determinam" aí em cima significa "tem pelo menos dois pontos desses
na reta".

Abraço,
 Ralph


2013/6/2 Guilherme Sales 

> Uma ideia:
>
> na pior das hipóteses, N-1 pontos estão na mesma reta (N-1 >= 2) e o
> N-ésimo está fora (pela hipótese de que não são todos colineares).
>
> Você tem então: a reta com os N-1 pontos e N-1 retas distintas que cada um
> desses determina com o que ficou de fora; então há pelo menos N retas
> distintas passando por pelo menos dois pontos.
>
> Acho que formalizar os detalhes não deve ser difícil.
>
> Guilherme
>
>
> Em 2 de junho de 2013 22:05, Cláudio Thor escreveu:
>
>  Alguém poderia me ajudar neste problema.
>>
>> Dados N pontos no plano, com N maior ou igual a 3, com a restrição de que
>> nem todos estão na mesma linha, demonstre que o conjunto das linhas que
>> passam por pelo menos dois pontos tem tamanho maior ou igual a N.
>>
>> Agradeço de já.
>>
>> Claudio
>>
>
>


[obm-l] Re: [obm-l] Duas perguntas(teoria dos números)

2013-05-27 Por tôpico Ralph Teixeira
1) Suponha, por contradicao, que 2^(x1)+2^(x2)+...+2^(xn)=2^A para
x1,x2,...,xn naturais distintos (suponha s.p.d.g. que x1=2).

Por um lado, A>B, porque o lado esquerdo eh claramente maior que 2^B; entao
A>=B+1.
Por outro lado, mesmo que voce use TODAS as potencias de 2 ateh 2^B, nao
chega a 2^A. De fato, o lado esquerdo eh menor ou igual a:
1+2+2^2+...+2^B=2^(B+1)-1<2^(B+1)<=2^A (usei soma dos termos da P.G. ali).

Entao realmente nao tem como valer a igualdade.

2) Pois eh, como voce viu, m-48=2^a e m+48=2^(a+b) onde b>0 (pois
m+48>m-48). Para mostrar que seu par eh o unico que presta, faca a
subtracao, como voce disse, e fatore:

2^a (2^b-1) = 96

Agora, 2^a SOH TEM FATORES 2, enquanto 2^b-1 eh impar e portanto NAO TEM
FATORES 2. Entao, 2^a tem que ser TODA a "parte potencia de 2" na fatoracao
de 96=(2^5) * 3, enquanto 2^b-1 tem que ser TODA a parte impar (isto eh,
3). Entao, a unica solucao eh a=5 e b=2, como voce havia dito.

Abraco,
  Ralph

2013/5/27 marcone augusto araújo borges 

> 1) Gostaria de saber se a soma de duas ou mais potencias de base 2
> distintas pode ser uma potencia de base 2.
>
> Acredito que não e escrevendo esses números na base 2 talvez se possa
> mostrar isso.
>
> 2) Desconfio que   2304 + 2^n é um quadrado perfeito para um único valor
> de n.
>
> Eu fiz 2^n = (m + 48)(m - 48)
> m + 48 e m - 48 devem ser potencias de base 2
> As únicas potencias de base 2 cuja diferença é 96 são 128 e 32
> Dai o único valor de n seria 12
> Um esclarecimento seria muito bem vindo
>
> Desde já agradeço
>
>
>


Re: [obm-l] Soma igual ao produto

2013-05-11 Por tôpico Ralph Teixeira
Pois eh, {1,2,3} eh bacana porque tem a propriedade e nao eh apelativo que
nem o meu montao de 1's...

Outro "problema" eh que, NOS REAIS, voce sempre pode tomar
x305=(x1+x2+...+x304)/(x1x2x304 - 1). Se o produto x1x304 for maior
que 1, o conjunto {x1,,x305} vai ter a propriedade pedida. Entao o
problema nao eh tao bacana nos reais, tem respostas demais que nao sao tao
especiais...

Entao me parece que a pergunta BACANA eh:

"Quais sao as n-uplas (x1,...,xn) (com x1<=x2<=...<=xn) de numeros NATURAIS
cuja soma eh igual ao produto e que tem NO MAXIMO um numero 1?"

(Versao 2, mais facil: SEM nenhum 1?)

Estas eu jah vi em algum lugar -- dah para atacar o problema, e nao tem
muitas respostas nao. Basicamente, o produto vai ser MUITO maior que a
soma, exceto em uns "poucos" casos.

Abraco,
  Ralph


2013/5/11 Bernardo Freitas Paulo da Costa 

> 2013/5/11 Ralph Teixeira :
> > Bom, se voce deixar a pergunta assim, a resposta eh sim, montes deltes.
> >
> > Afinal, 1+1+1+...+1+x_1+x_2+...+x_n=1.1.1.1.1.1.1.x_1.x_2.x_3x_n
> se
> > voce botar o numero certo de 1's ali...
> >
> > Entao a pergunta bacana eh...?
>
> Poxa, eu achei 1 + 2 + 3 = 1 * 2 * 3 tão bacana!
> --
> Bernardo Freitas Paulo da Costa
>
> =
> Instruções para entrar na lista, sair da lista e usar a lista em
> http://www.mat.puc-rio.br/~obmlistas/obm-l.html
> =
>


Re: [obm-l] Soma igual ao produto

2013-05-11 Por tôpico Ralph Teixeira
Bom, se voce deixar a pergunta assim, a resposta eh sim, montes deltes.

Afinal, 1+1+1+...+1+x_1+x_2+...+x_n=1.1.1.1.1.1.1.x_1.x_2.x_3x_n se
voce botar o numero certo de 1's ali...

Entao a pergunta bacana eh...?


2013/5/11 Paulo Argolo 

> Caros Colegas,
>
>
> Sabemos que 2 + 2 = 2.2  e 1+ 2 + 3 = 1.2.3
>
> Minha dúvida: Existem outros números reais positivos (dois ou mais,
> distintos ou não) cuja soma seja igual ao produto?
>
>
> Abraços do Paulo Argolo
>
> __
> =
> Instruções para entrar na lista, sair da lista e usar a lista em
> http://www.mat.puc-rio.br/~obmlistas/obm-l.html
> =
>


Re: [obm-l] Ajuda

2013-05-05 Por tôpico Ralph Teixeira
Oi, Marcelo.

Esse caiu na Primeira Olimpiada Iberoamericana. De uma olhada em
http://www.mail-archive.com/obm-l@mat.puc-rio.br/msg48192.html

Achando a area, eh facil achar o lado.

Abraco,
 Ralph


2013/5/5 Marcelo de Moura Costa 

> *Tenho certeza de que alguém da lista já se deparou com esse problema e
> sua solução:*
>
> Um ponto interno de um triângulo equilátero dista 5cm, 7cm e 8cm dos
> vértices do triângulo.?
>
> *Solução:*
>
> *3(p^4 + q^4 + t^4 + a^4) = (p^2 + q^2 + t^2 + a^2)^2.
>
> p = 5
> q = 7
> t = 8
> *
> *a=lado do triângulo equilátero. *
> *
> *
> *Alguém já viu a demonstração ou conhece?*
> *Agradeceria a informação.*
> *
> *
> *Abraços e boa semana.*
> *
> *
> *Marcelo*
>


Re: [obm-l] Matrizes

2013-04-26 Por tôpico Ralph Teixeira
Tem um Tao (de Terence Tao) que tem umas ideias sobre isso:

http://arxiv.org/abs/math/0501313

2013/4/26 Athos Cotta Couto :
> Seja M uma matriz nxn, onde aos elementos dessa matriz são atribuidos
> aleatoriamente os valores 0 ou 1. Qual a probabilidade que essa matriz seja
> inversível?

=
Instruções para entrar na lista, sair da lista e usar a lista em
http://www.mat.puc-rio.br/~obmlistas/obm-l.html
=


[obm-l] Re: [obm-l] Re: [obm-l] Divisibilidade(questão simples)

2013-04-18 Por tôpico Ralph Teixeira
Ou, para evitar totalmente congruências e coisas assim, note que
n^2+1=(n+2)(n-2)+5. Então:

n^5-n=(n-1)n(n+1)(n^2+1)=(n-2)(n-1)n(n+1)(n+2)+5(n-1)n(n+1)

O primeiro termo tem 5 números consecutivos, então é divisível por 2, 3 e
5. O segundo tem 3 números consecutivos e aquele fator 5, então também é.

Abraço,
Ralph


2013/4/18 Nehab 

>  Oi, Mauricio,
>
> Apenas uma obs para evitar congruências (em seu argumento de
> divisibilidade por 5) e, assim, tornar a questão accessível para quem não
> aprendeu este conteúdo:
>
> A partir de sua fatoração n(n^4 - 1), por exemplo, eu usaria o seguinte
> argumento:
>
> - O último algarismo de n^4 possui periodicidade 1, 2 ou 4, qqs o último
> algarismo final de n (fácil de mostrar para a garotada através de uma
> tabelinha)...
> - Tais potências (expoente 4) sempre terminarão em 1, 5 ou 6; logo, se n
> não terminar em 5, tal último algarismo, menos 1 será 5...
>
> Este tipo de argumento resolve vários problemas olímpicos mais simples de
> forma mais intuitiva.
>
> Abraços
> Nehab
>
> On 18/04/2013 14:00, Mauricio de Araujo wrote:
>
>  fatorando: n5-n = n(n4-1) = n(n2+1)(n+1)(n-1)...
>
>  temos 3 números consecutivos => multiplo de 2 e 3
>
>  note agora que n(n4-1) é ´multiplo de 5 pois:
>
>  ou n é múltiplo de 5 ou
> n4-1 mas n4-1 é múltiplo de 5 sempre que n não o for... use
> congruencia...
>
>  n=1 (mod5) => n4=1(mod5);
> n=2(mod5) => n2=-1(mod5) => n4=1(mod5);
> n=3(mod5) => n2=-1(mod5) => n4=1(mod5);
> n=4(mod5) => n4=1(mod5)...
>
>  Logo n5-n é múltiplo de 2, 3 e 5 ou seja, múltiplo de 30
>
>  CQD.
>
>
> 2013/4/18 marcone augusto araújo borges 
>
>>  Mostrar que  m = n^5 - n é divisível por 30
>>
>>  Fatorando,dá pra ver que m é múltiplo de 3.
>> Como o algarismo das unidades de n^5 é igual ao algarismo das
>> unidades de n,temos que m termina em zero,ou seja,é múltiplo de 10,e ai
>> acaba.
>> Fui tentar por indução também e ai complicou.
>> Alguém resolveria por indução?
>>
>>
>> --
>> Esta mensagem foi verificada pelo sistema de antivírus e
>> acredita-se estar livre de perigo.
>>
>
>
>
>  --
>  Abraços
>
> oɾnɐɹɐ ǝp oıɔıɹnɐɯ
> *momentos excepcionais pedem ações excepcionais.*
>  *A primeira vez é sempre a última chance.*
>
> --
> Esta mensagem foi verificada pelo sistema de antiv�rus e
> acredita-se estar livre de perigo.
>
>
>
> --
> Esta mensagem foi verificada pelo sistema de antivírus e
> acredita-se estar livre de perigo.
>

-- 
Esta mensagem foi verificada pelo sistema de antiv�rus e
 acredita-se estar livre de perigo.



[obm-l] Re: [obm-l] BINÔMIO DE NEWTON

2013-01-25 Por tôpico Ralph Teixeira
Em primeiro lugar, analise o triangulo de Pascal modulo 2. Fica algo assim:

1
11
101

10001
110011
1010101


Entao, provar que a linha 2^n-1 eh toda impar, isto eh, 111...1,
eh o mesmo que provar que a linha 2^n eh do tipo 10...0001.

Agora, o terence tinha provado isso numa mensagem anterior. Era algo assim:

i) Tomando coeficientes modulo 2, tem-se (z+1)^2=z^2+1. Entao
(x+1)^4=(x^2+1)^2=x^4+1, e (x+1)^8=(x^4+1)^2=x^8+1, etc. Em suma,
(x+1)^(2^p)=x^(2^p)+1. Entao quando n=2^p, a linha eh 10...0001 e
a linha n+1 eh 1...111.

ii) Por outro lado, seja n um inteiro qualquer nao potencia de 2.
Escreva-o em base 2, assim: n=p1+p2+...+pn onde os p1:
> Caros amigos, já apareceu na lista, mas não me convenceu. Se alguém tiver
> uma solução, agradeço!
>
> Seja n um inteiro positivo. Demonstrar que todos os coeficientes do
> desenvolvimento do binômio de Newton (a+b)^n são ímpares se, e somente se, n
> é da forma 2^s - 1.

=
Instruções para entrar na lista, sair da lista e usar a lista em
http://www.mat.puc-rio.br/~obmlistas/obm-l.html
=


Re: [obm-l] Probabilidade

2013-01-13 Por tôpico Ralph Teixeira
Do jeito que eu vejo o problema faltam dados. Voce precisaria ter uma
ideia do seguinte:

i) Supondo que os filhos sao (h,h), quao frequentemente o casal
responderia deste jeito "sim, o mais velho eh homem"? Quao
frequentemente responderia "sim, o mais NOVO eh homem", ou
simplesmente "sim" ou qualquer outra coisa?
ii) E se fossem (h,m)? Quao frequentemente eles diriam "sim, o mais
velho eh homem" versus outras coisas?

E, convenhamos, estas probabilidades nao sao obvias, dependem mais de
psicologia do que de matematica Por isso que, nos problemas
originais, a gente limitava as respostas a "sim" ou "nao", e imaginava
que os casais nunca mentiam -- ai nao precisava de nada disso, porque
o que o casal respondia era o que voce sabia, e nada mais.

(Alias, note-se: nos problemas originais, se o casal mente de vez em
quando, voce teria que (i) ter uma ideia de quao frequentemente os
casais mentem e (ii) fatorar essa informacao no problema, o que pode
modificar a resposta!)

Abraco,
Ralph

2013/1/13 Bruno Rodrigues :
> Oi Ralph,não sei se está certo,mas vou escrever aqui meu raciocínio sobre
> seu desafio.
> Pelo meu raciocínio,o espaço amostral seria parecido com o do último
> exemplo,sendo dessa vez (x,y,z),onde (x,y) são os filhos,e z o filho mais
> velho.O espaço amostral então
> seria:{(h,m,h),(m,h,h),(m,h,m),(h,m,m),(m,m,m),(m,m,m),(h,h,h),(h,h,h)},onde
> a probabilidade de cada subconjunto acotencer é de 1/8.
> Seria assim que eu responderia a sua pergunta?
>
> Abraços,
> Bruno
> Por probablidade condicional: P(ter 2 filhos h | filho + velho é h),que é
> equivalente a perguntar: qual a probabilidade de ter 2 filhos homens sabendo
> que o filho mais velho é homem.
> Resolvendo: P(ter 2 filhos h | filho + velho é h)=P(ter 2 filhos h ∩ filho +
> velho é h) /P(filho + velho é h)=0,25/0,50=1/2.
>
> Em 11 de janeiro de 2013 21:45, Ralph Teixeira  escreveu:
>>
>> Oi, Heitor e Bruno.
>>
>> Pois eh, este problema eh famoso... vejam aqui:
>>
>> http://en.wikipedia.org/wiki/Boy_or_Girl_paradox
>>
>> O espaco amostral razoavel eh aquele mesmo omega que o Bruno pos. Do
>> jeito que eu interpreto probabilidade (sou Bayesiano) nao precisa
>> supor infinitos casais -- mas eh necessario fazer as hipoteses usuais
>> de que filhos e filhas sejam igualmente provaveis e de que o sexo dos
>> filhos sao independentes um do outro. Neste caso, a distribuicao de
>> probabilidade em omega eh 1/4 para cada um dos 4 eventos elementares.
>> Entao:
>>
>> A) Pr( (h,h) | {(h,m),(m,h),(h,h)})=(1/4)/(3/4)=1/3. (interpretei como
>> "pelo menos um filho homem")
>> B) Pr( (h,h) | {(h,m),(h,h)} )=(1/4)/(2/4)=1/2.
>>
>> Eh isso mesmo: em linguagem coloquial imprecisa, 1/3 dos (casais que
>> tem pelo menos um filho homem) tem dois filhos homens; mas 1/2 dos
>> casais (cujo filho mais velho eh homem) tem dois filhos homens.
>>
>> Agora, cuidado -- a probabilidade depende um bocado de COMO voce
>> descobriu que pelo menos um filho eh homem.
>>
>> -- Se voce perguntou explicitamente ao casal "pelo menos um dos seus
>> filhos eh homem" e soh deixou eles responderem "sim" ou "nao", neste
>> caso, a probabilidade eh 1/3. Em linguagem imprecisa, 1/3 dos casais
>> que responderem "sim" terao dois filhos homens.
>>
>> -- Agora, se voce perguntar ao casal "pense aleatoriamente em uma de
>> suas 2 criancas. Pensou? Eh um homem?" e eles responderem "sim", agora
>> a resposta eh 1/2, mesmo que voce nao saiba nada da crianca pensada
>> alem de ela ser homem! Sim, isto eh BEM DIFERENTE da situacao
>> anterior, onde voce faz o casal pensar em AMBOS os filhos antes de
>> responder -- aqui eles soh pensaram em um deles! Para fazer este aqui,
>> voce teria que aumentar o espaco amostral para incluir em que filho
>> eles pensaram. Ficaria algo assim: (x,y,z) onde (x,y) sao os filhos e
>> z eh o filho que eles escolheram, supostamente com probabilidade 1/2.
>> O espaco amostral seria:
>> {(h,h,h),(h,h,h),(h,m,h),(h,m,m),(m,h,m),(m,h,h),(m,m,m),(m,m,m)} onde
>> cada elemento tem 1/8 de probabilidade (ou junte aqueles (h,h,h) e
>> aqueles (m,m,m) cada um com 1/4). Entao
>>
>> Pr( (h,h,?) | (?,?,h) ) = 2/4=1/2.
>>
>> Isto eh equivalente a perguntar "o mais velho eh homem?" e receber um
>> "sim" de resposta!
>>
>> ---///---
>>
>> Para complicar, aqui vai o terceiro problema: voce pergunta ao casal
>> "pelo menos um de seus filhos eh homem?" e o casal responde "sim, o
>> mais velho eh homem". Qual eh a probabili

Re: [obm-l] Valores de a, b e c

2013-01-12 Por tôpico Ralph Teixeira
Estou sem tempo agora, mas acho que jah mostramos que nao ha solucoes positivas.

Afinal, se houvesse, seria 0<=S<=1/3. Mas S=x+y=(a+b)/c, entao
teriamos c/(a+b)>=3.

Como isso tambem valeria para a/(b+c) e b/(a+c), acho que teriamos um absurdo.

(Deve ter algum jeito MUUUITO mais rapido de ver isso -- ou entao eu errei algo)

Abraco,
   Ralph

2013/1/12 Rhilbert Rivera :
> Muito obrigado pelas informações Ralph, vou  verificar com cuidado. Mas, era
> como você desconfiou, eu queria os valores INTEIROS, rsrsrs... vamos
> aguardar.
>
> Abraços
>
>> Date: Thu, 10 Jan 2013 13:02:01 -0200
>> Subject: Re: [obm-l] Valores de a, b e c
>> From: ralp...@gmail.com
>> To: obm-l@mat.puc-rio.br
>>
>> CASO 1: c=0.
>> Neste caso, temos a/b+b/a=1. Entao x=a/b teria que ser positivo
>> Mas x+1/x>=2 para todo x real positivo, entao nao ha solucoes no caso
>> 1.
>>
>> CASO 2: c<>0
>> A ideia eh notar que a equacao eh homogenea: se (a,b,c) eh solucao,
>> entao (Ka, Kb, Kc) tambem eh (para K<>0) Entao tomando K=1/c, a
>> gente ve que (x,y,1) tem que ser solucao (onde x=a/c e y=b/c). Melhor
>> ainda, botando tudo em funcao de x e S=x+y, vem:
>>
>> x/(S-x+1) + (S-x)/(x+1) + 1/S = 1
>>
>> Abrindo tudo, organizando como uma quadratica em x, se eu nao errei
>> contas, fica (3S-1)x^2-(3S^2-S)x+(S^3+1)=0.
>>
>> Mais contas, achei o discriminante como D=-(3S-1)(S+2)(S^2-S+2). Para
>> isto ser positivo, devemos ter (3S-1)(S+2)<0, isto eh, -2<=S<=1/3. Por
>> outro lado, dado S ai, certamente temos solucao:
>>
>> x=((3S^2-S)+-raiz(D))/2(3S-1)
>>
>> Entao voces tem infinitas solucoes da forma
>> a=c[((3S^2-S)+-raiz(D))/2(3S-1)]
>> b=c(S-x)
>> c=c
>> onde S eh um real arbitrario em [-2,1/3] e c eh outro real arbitrario.
>>
>> Abraco,
>> Ralph
>>
>> P.S.: Ou voce queria apenas valores INTEIROS de a, b e c? Ai eh OUTRO
>> problema
>>
>> 2013/1/9 Rhilbert Rivera :
>> > Buscando uma ajuda no problema a seguir.
>> >
>> > Se a/(b+c) + b/(c+a) + c/(a+b) =1, quais os possíveis valores de a, b e
>> > c?
>> >
>> > Obrigado
>>
>> =
>> Instruções para entrar na lista, sair da lista e usar a lista em
>> http://www.mat.puc-rio.br/~obmlistas/obm-l.html
>> =

=
Instruções para entrar na lista, sair da lista e usar a lista em
http://www.mat.puc-rio.br/~obmlistas/obm-l.html
=


Re: [obm-l] Probabilidade

2013-01-11 Por tôpico Ralph Teixeira
Oi, Heitor e Bruno.

Pois eh, este problema eh famoso... vejam aqui:

http://en.wikipedia.org/wiki/Boy_or_Girl_paradox

O espaco amostral razoavel eh aquele mesmo omega que o Bruno pos. Do
jeito que eu interpreto probabilidade (sou Bayesiano) nao precisa
supor infinitos casais -- mas eh necessario fazer as hipoteses usuais
de que filhos e filhas sejam igualmente provaveis e de que o sexo dos
filhos sao independentes um do outro. Neste caso, a distribuicao de
probabilidade em omega eh 1/4 para cada um dos 4 eventos elementares.
Entao:

A) Pr( (h,h) | {(h,m),(m,h),(h,h)})=(1/4)/(3/4)=1/3. (interpretei como
"pelo menos um filho homem")
B) Pr( (h,h) | {(h,m),(h,h)} )=(1/4)/(2/4)=1/2.

Eh isso mesmo: em linguagem coloquial imprecisa, 1/3 dos (casais que
tem pelo menos um filho homem) tem dois filhos homens; mas 1/2 dos
casais (cujo filho mais velho eh homem) tem dois filhos homens.

Agora, cuidado -- a probabilidade depende um bocado de COMO voce
descobriu que pelo menos um filho eh homem.

-- Se voce perguntou explicitamente ao casal "pelo menos um dos seus
filhos eh homem" e soh deixou eles responderem "sim" ou "nao", neste
caso, a probabilidade eh 1/3. Em linguagem imprecisa, 1/3 dos casais
que responderem "sim" terao dois filhos homens.

-- Agora, se voce perguntar ao casal "pense aleatoriamente em uma de
suas 2 criancas. Pensou? Eh um homem?" e eles responderem "sim", agora
a resposta eh 1/2, mesmo que voce nao saiba nada da crianca pensada
alem de ela ser homem! Sim, isto eh BEM DIFERENTE da situacao
anterior, onde voce faz o casal pensar em AMBOS os filhos antes de
responder -- aqui eles soh pensaram em um deles! Para fazer este aqui,
voce teria que aumentar o espaco amostral para incluir em que filho
eles pensaram. Ficaria algo assim: (x,y,z) onde (x,y) sao os filhos e
z eh o filho que eles escolheram, supostamente com probabilidade 1/2.
O espaco amostral seria:
{(h,h,h),(h,h,h),(h,m,h),(h,m,m),(m,h,m),(m,h,h),(m,m,m),(m,m,m)} onde
cada elemento tem 1/8 de probabilidade (ou junte aqueles (h,h,h) e
aqueles (m,m,m) cada um com 1/4). Entao

Pr( (h,h,?) | (?,?,h) ) = 2/4=1/2.

Isto eh equivalente a perguntar "o mais velho eh homem?" e receber um
"sim" de resposta!

---///---

Para complicar, aqui vai o terceiro problema: voce pergunta ao casal
"pelo menos um de seus filhos eh homem?" e o casal responde "sim, o
mais velho eh homem". Qual eh a probabilidade de ambos serem homens?
Que outros dados voce precisaria, ou que hipoteses voce faria para
calcular isso? :) :) :) :)

Abraco,
   Ralph

2013/1/10 Bruno Rodrigues :
> Você é da turma de probabilidade do leonardo?
> Ele passou esse exercício lá,mas disse que ia alterar pq da pra ser
> subentendido que existem infinitos casais com 2 filhos e vc teria que
> escolher 1 entre os infinitos,com probabilidade 1/infinito de cada casal ser
> escolhido.Lá ele me explicou (o que eu entendi de sua explicação) que o
> espaço amostral omega seria omega={(h,m),(m,h),(h,h),(m,m)} , onde x=irmão
> mais velho e y=irmão mais novo,e a partir daí vc calcula A e B,mas a próxima
> parte ele não explicou pra começar um assunto novo,e seria legal ouvir a
> opinião de outras pessoas também a respeito desse exercício.
> Bom,esse foi o jeito que o professor disse na sala,também tive dúvidas
> absurdas nela =) , espero ter ajudado.
> Saudações
> Bruno
>
> Em 10 de janeiro de 2013 19:47, Heitor Bueno Ponchio Xavier
>  escreveu:
>
>> Numa cidade são catalogados todos casais que tenham 2 filhos e que não
>> sejam gêmeos. Um casal é escolhido ao acaso dessa lista. Calcule a
>> probabilidade condicional de esse casal ter dois filhos homens, sabendo-se
>> que:
>> A) O casal tem um filho homem.
>> B)O filho mais velho do casal é homem.
>
>

=
Instruções para entrar na lista, sair da lista e usar a lista em
http://www.mat.puc-rio.br/~obmlistas/obm-l.html
=


Re: [obm-l] Valores de a, b e c

2013-01-10 Por tôpico Ralph Teixeira
CASO 1: c=0.
Neste caso, temos a/b+b/a=1. Entao x=a/b teria que ser positivo
Mas x+1/x>=2 para todo x real positivo, entao nao ha solucoes no caso
1.

CASO 2: c<>0
A ideia eh notar que a equacao eh homogenea: se (a,b,c) eh solucao,
entao (Ka, Kb, Kc) tambem eh (para K<>0) Entao tomando K=1/c, a
gente ve que (x,y,1) tem que ser solucao (onde x=a/c e y=b/c). Melhor
ainda, botando tudo em funcao de x e S=x+y, vem:

x/(S-x+1) + (S-x)/(x+1) + 1/S = 1

Abrindo tudo, organizando como uma quadratica em x, se eu nao errei
contas, fica (3S-1)x^2-(3S^2-S)x+(S^3+1)=0.

Mais contas, achei o discriminante como D=-(3S-1)(S+2)(S^2-S+2). Para
isto ser positivo, devemos ter (3S-1)(S+2)<0, isto eh, -2<=S<=1/3. Por
outro lado, dado S ai, certamente temos solucao:

x=((3S^2-S)+-raiz(D))/2(3S-1)

Entao voces tem infinitas solucoes da forma
a=c[((3S^2-S)+-raiz(D))/2(3S-1)]
b=c(S-x)
c=c
onde S eh um real arbitrario em [-2,1/3] e c eh outro real arbitrario.

Abraco,
 Ralph

P.S.: Ou voce queria apenas valores INTEIROS de a, b e c? Ai eh OUTRO
problema

2013/1/9 Rhilbert Rivera :
> Buscando uma ajuda no problema a seguir.
>
> Se   a/(b+c) + b/(c+a) + c/(a+b) =1, quais os possíveis valores de a, b e c?
>
> Obrigado

=
Instruções para entrar na lista, sair da lista e usar a lista em
http://www.mat.puc-rio.br/~obmlistas/obm-l.html
=


[obm-l] Re: [obm-l] RE: [obm-l] Re: [obm-l] Espaços

2012-11-19 Por tôpico Ralph Teixeira
Soh para dar a minha opiniao:

OFICIALMENTE, R^(n-1) nao eh subespaco de R^n -- o problema eh que R^(n-1)
nao eh nem SUBCONJUNTO de R^n, jah que os elementos de R^(n-1) sao
"completamente" diferentes dos de R^n (acho que foi isso que o Artur falou).

Isto dito Para mim, existe uma identificacao bem natural, quase
canonica, de R^(n-1) com o espaco dos elementos de R^n cuja ultima
coordenada eh zero, como o Rafael falou. Com esta identificacao, eh natural
pensar no R^(n-1) como subespaco de R^n (COM ESTA IDENTIFICACAO!!!). Assim,
se alguem falar que R^(n-1) eh subespaco de R^n, provavelmente eh isso
que estah se pensando.

Alias, eh mais que um homeomorfismo, eh um isomorfismo de R^(n-1) com um
subespaco do R^n Tah, isso nao significa nada, afinal quaisquer dois
espacos reais de mesma dimensao finita sao isomorfos... Mas eh MAIS do que
um isomorfismo, porque, repito, a identificacao eh muito natural.

Abraco,
 Ralph

P.S.: Para quem acha que a identificacao nao eh natural -- eu nao a defini
explicitamente, mas aposto que voce sabe qual eh! :) :) :)
2012/11/19 Rafael Chavez 

>  Na realidade R^(n-1) é homeomorfo a um subespaço de R^n que pode ser por
> exemplo o espaço das n-uplas com a última coordenada sendo zero.
> Topologicamente homeomorfo significa ter as mesmas propriedades
> topológicas, i. e., topologicamente eles são iguais, mas só topologicamente.
>
> Rafael
>
> --
> Date: Sun, 18 Nov 2012 12:24:01 -0300
> Subject: [obm-l] Re: [obm-l] Espaços
> From: steinerar...@gmail.com
> To: obm-l@mat.puc-rio.br
>
> Não. Rn é compisto por n- tuplas. R(n-2) por (n-1)tuplas. Eles tem
> dimensões diferentes
> Artur
> Artur Costa Steiner
> Em 17/11/2012 13:39, "Athos Couto"  escreveu:
>
>  Boa tarde pessoal.
> Rn-1 está contido em Rn?
>
> Caso a resposta seja sim, por que Rn-1 não é um subespaço de Rn?
>
> OBS: denotei o conjunto dos números reais por R
> Obrigado pela ajuda.
> Att.
>
> Athos Cotta Couto
>
>


[obm-l] Re: [obm-l] Valor máximo

2012-11-09 Por tôpico Ralph Teixeira
H Você quer a,b,c positivos?

Então eu aposto (por simetria) que o máximo é quando a=b=c=1/3, quando dá
1/81.

Aliás, roubei um pouquinho aqui: botei no Wolfram Alpha:

"maximize a^2b^2c^2/(a^3+b^3+c^3) subject to a+b+c=1"

e ele também acha que é (1/3,1/3,1/3).

Link:
http://www.wolframalpha.com/input/?i=maximize+a%5E2b%5E2c%5E2%2F%28a%5E3%2Bb%5E3%2Bc%5E3%29+subject+to+a%2Bb%2Bc%3D1

E agora?

Abraço,
  Ralph




On Fri, Nov 9, 2012 at 6:17 PM, Athos Couto  wrote:

>  Sendo a+b+c = 1:
> Qual o valor máximo de:
> (abc)²/(a³+b³+c³) ?
> E quais as tríades de (a,b,c) que atingem esse valor?
>
>
>


<    1   2   3   4   5   6   7   8   >